PAeasy Pulmonary

Ace your homework & exams now with Quizwiz!

A 31-year-old HIV-positive woman presents for ongoing care. She was diagnosed with HIV 2 years ago, and she began antiretroviral therapy. Her CD4 T cell count is 400 cells/mL, and she has a history of oral candidiasis. As part of evaluation, her physician decides to perform a tuberculin skin test (TST) using 5 TU of purified protein derivative (PPD). The test site is examined 48 hours later and the skin reaction is measured. What is the minimum diameter of induration at which this test result should be considered positive in this patient? Answer Choices 1 2 mm 2 5 mm 3 10 mm 4 15 mm 5 20 mm

5 mm Persons with HIV should be tested yearly for tuberculosis using the purified protein derivative (PPD) skin test, also referred to as the Mantoux test. In those with HIV, and in certain other cases (refer to the table), an induration of more than, or equal to 5mm, is considered positive. Preventive therapy should be prescribed for all patients having a positive PPD. Those with a positive skin test or high-risk exposure should undergo prophylaxis. A common regimen consists of isoniazid (INH) and pyridoxine daily, usually for at least 1 year. CDC recommends INH (300mg/day) for a period of 12 months to any HIV-infected persons with positive TST (> 5 mm), along with supplemental pyridoxine (25 - 50mg/day), to prevent peripheral neuropathy. Both isoniazid-resistant and multidrug-resistant strains of Mycobacterium tuberculosis are becoming more prevalent. Classifying positive TST reactions: Interpretation of the Tuberculin Skin Test* Induration (dia.) Positive in Persons With > 5mm - HIV infection - People with chest X-ray findings consistent with prior TB - Close contacts of a person with infectious TB - Patients with organ transplant and other immunosuppressed persons > 10mm - Medical risk factors such as chronic renal disease, diabetes, gastrectomy, and silicosis - Residents/employees of high-risk congregate settings (jails, nursing homes, hospitals and other long-term facilities for elderly) - IV drug users - Mycobacteriology laboratory personnel > 15mm - Healthy persons without known risk factors * 5 TU PPD

What is the most appropriate age to administer the pneumococcal vaccine in healthy individuals with no other comorbid diseases? A Under age 16 B After age 16-34 C > Age 35 D > Age 50 E > Age 65

> Age 65 E The Advisory Committee on Immunization Practices of the Centers for Disease Control and Prevention recommend that the most appropriate age to administer the pneumococcal vaccine in healthy individuals with no other comorbid diseases is over the age of 65.

A 43 year-old woman is brought to the emergency department in critical care secondary to a traumatic brain injury. Which of the following best describes her Cheyne-Stokes respiratory pattern? A Alternating periods of shallow and deep breathing B Difficult or labored breathing C Difficult of labored breathing while supine D Periods of absent breathing E Sudden awakening due to shortness of breath

A Cheyne-Stokes respirations are characterized by shallow breaths that increase in rate and depth followed by periods of apnea (A). Dyspnea is difficult or labored breathing (B) and if it occurs while supine (C) is termed orthopnea. Apnea is noted periods of absent breathing (D). Sudden awakening due to shortness of breath (E) is paroxysmal nocturnal dyspnea.

What exposure is linked to brucellosis

Brucellosis can be caused by exposure to hogs, cattle, or goats.

What is the most common blood gas abnormality in patients with a pulmonary embolism? A Respiratory alkalosis B Metabolic acidosis C Metabolic alkalosis D Respiratory acidosis E Compensated respiratory acidosis

Respiratory Alkalosis Respiratory alkalosis occurs as a result of the hyperventilation.

A 3-month-old male infant presents with history of noisy breathing since birth; the noise is gradually increasing. There is no history of fever, cough, or running nose. Physical examination reveals a low-pitched, inspiratory wheeze; it is more prominent over the central airways and loudest over the trachea. Wheezing increases during crying, feeding, and when the infant is laid in supine position. There is no cyanosis, subcostal or intercostal retraction, or hoarseness of voice. Wheezing has not shown any response to bronchodilators. Question What is the most likely diagnosis? Answer Choices 1 Bronchiolitis 2 Congenital subglottic stenosis 3 Tracheomalacia 4 Tracheoesophageal fistula 5 Vocal cord paralysis

Tracheomalacia Tracheomalacia is a common cause of persistent wheezing in early infancy, with male preponderance in the ratio of 2:1. In primary tracheomalacia, there is insufficient cartilage to maintain the patency of the airway throughout the respiratory cycle. It commonly occurs in premature infants. Secondary tracheomalacia occurs when trachea is compressed by structures like vascular rings or deficient cartilage due to tracheoesophageal fistula. The dominant finding is a monophonic, low-pitched wheeze most prominent over central airways. There is persistent wheeze even in the absence of any viral respiratory infection, and it is loudest over the trachea. Subcostal retractions are absent unless there is asthma or some other cause of small airway obstructions. Wheeze does not respond to bronchodilators. Diagnosis is confirmed by flexible or rigid bronchoscopy. Bronchiolitis is usually caused by viral infection. More than 50% of cases are caused by respiratory syncytial virus (RSV). Other viruses include parainfluenza virus, adenovirus, and mycoplasma. It commonly occurs in male infants who have not been breast-fed. The infant first develops a mild upper respiratory tract infection with sneezing and rhinorrhea, which is accompanied by fever up to 101 - 102 degrees Fahrenheit. Respiratory distress, cough, and wheezing appear gradually; symptoms may interfere with feeding. Apnea is more common in infants less than 2 months of age. The infant is tachypneic with nasal flaring and chest retractions. Auscultation may reveal fine crackles and wheeze. A trial with bronchodilators, albuterol, or epinephrine may be considered and continued only if there is a clinical improvement with epinephrine used in an inpatient setting. Congenital subglottic stenosis is the 3rd most common congenital anomaly of the larynx causing stridor, which is usually biphasic or primarily inspiratory. Recurrent or persistent croup occurs in these infants. The 1st symptom often occurs during an episode of respiratory infection, as edema and thick secretions cause narrowing of the already compromised airway. Esophageal atresia is a frequent congenital anomaly affecting 1/4000 neonates. Of these, over 90% have associated tracheoesophageal fistula. It typically manifests with frothing and bubbling at the mouth and nose and episodes of coughing, respiratory distress, choking, and cyanosis. These symptoms are exacerbated by feeding. Regurgitation of gastric contents through the distal fistula causes more damaging pneumonia than aspiration of pharyngeal secretions from the blind upper pouch of esophagus. Vocal cord paralysis is the 2nd most common cause of neonatal stridor. It is often associated with central nervous system lesions like meningomyelocele, Arnold-Chiari malformation, and hydrocephalus. Bilateral vocal cord paralysis typically produces airway obstruction manifested by high-pitched inspiratory stridor. Unilateral paralysis causes aspiration, coughing, and choking. Cry is weak. Other symptoms of airway obstruction are less common. Diagnosis of vocal cord paralysis is made by awake flexible laryngoscopy with ultrasonography, which is a useful adjunctive examination.

What exposure is linked to Tularemia

Tularemia is associated with contact with rabbits, other rodents, and biting arthropods.

What would be tactile remits be in a pleural effusion

Decreased

What would the tactile fremitis be in a Pneumothorax/hemothorax?

Decreased

A newborn boy is evaluated in the delivery room at 1 minute of life after an emergency C-section due to late decelerations. He is limp, pale, and unresponsive; he has a heart rate of 78 and a slow, irregular respiratory rate. What is his Apgar score? Answer Choices 1 0 2 10 3 6 4 2 5 5

2 The correct response is 2. The Apgar score is a system of assessing newborns immediately after birth to help identify those requiring resuscitation, and to predict survival in the newborn period. The 1-minute score may signal the need for immediate resuscitation, and the 5-minute, 10-minute, and subsequent scores may indicate the probability of success. The score is based on the observation of 5 signs including heart rate, respiratory effort, muscle tone, reflex response, and color. Each receives an individual score between 0 and 2. The highest possible score is 10. This patient would receive 1 point each for heart rate and respiratory effort, and no points for muscle tone, reflex response, or color. Immediate resuscitation efforts would be initiated due to the low score.

A 24-year-old man undergoes a routine medical check-up to become a volunteer in the ER. PPD skin test shows a diameter of 9 mm. Sputum and chest X-ray were negative for tuberculosis. He is otherwise healthy and shows no systemic effect of Mycobacterium tuberculosis infection. What is the most appropriate explanation for the doubtful tuberculin test in this patient? Answer Choices 1 Sputum and chest X-ray report could be wrong 2 Adequate sputum samples were not supplied 3 He was given BCG vaccination earlier in life 4 He may suffer from chronic cavitary lesion in the lung 5 He has an active tubercular infection

3 He was given BCG vaccination earlier in life The Mantoux tuberculin skin test (TST) is performed to determine whether the person is infected with Mycobacterium tuberculosis. The TST is performed by injecting 0.1 ml of tuberculin purified protein derivative (PPD) into the inner surface of the forearm and when placed correctly it should produce a wheal of 6 - 10 mm in diameter. The skin test results should be read 48 - 72 hours after administration of the PPD. Generally any palpable induration measuring 10 mm or more is considered a positive reaction. In the case of tuberculosis suspects, or close contacts of individuals with tuberculosis, an induration of 5 mm or more should be interpreted as a positive reaction. Induration measuring 5-9 mm is considered a doubtful reaction. In case of doubtful reaction, the possibility of skin sensitivity due to previous immunization or atypical mycobacterium should be considered. Though prior BCG vaccination increases the risk of a reactive PPD, this effect is known to be inconsistent. Studies have shown that reactions >10 mm should not be attributed to prior BCG vaccine. Positive tuberculin test indicates exposure of the immune system to tuberculous protein, either in the form of BCG vaccineor an active tuberculous infection or a chronic tuberculous infection. Proper vaccination history could have unmasked the cause of doubtful reaction to tuberculin test in this patient. Other causes, such as a wrong sputum and chest X-ray report, inadequate sputum samples, chronic cavitary lesion, or active tubercular infection are not the cause of doubtful reaction in this patient.

A 60-year-old man presents with a 1-year history of increasing dyspnea. The patient used to work as a construction worker, and he is a chronic smoker. A chest radiograph shows diaphragmatic pleural plaques and interstitial lung disease. What is the likely diagnosis? Answer Choices 1 Silicosis 2 Siderosis 3 Byssinosis 4 Berylliosis 5 Asbestosis

Asbestosis The radiological changes in asbestosis are usually confined to the lower 2/3 of the lung-fields. Pleural plaques are a frequent finding in a patient with asbestosis. The classical clinical feature in asbestosis is increasing exertional breathlessness due to pulmonary fibrosis. Digital clubbing is usually present, and inspiratory crepitations are audible over the lower zones of both lungs. Silicosis produces a pattern of silicotic nodules in the lung. Silica is a fibrogenic dust and causes the development of hard nodules, which coalesce as the disease progresses. The radiological changes of silicosis are more marked in the upper zones of the lung. Siderosis is a disorder due to increased iron. Increased iron in the lung is an uncommon pneumoconiosis. It is more likely to occur with hemochromatosis with iron overload. It occurs in occupations like arc welding. Byssinosis produces asthma-like symptoms. Berylliosis produces sarcoid-like granulomatous disease (chronically) and an acute pneumonitis (acutely). Patient will have history of exposure to beryllium.

A 40-year-old man presents with a 5-day history of cough and purulent sputum. The patient has had recurrent attacks of cough with sputum production since his childhood. There are no other systemic complaints. The patient is febrile and has grade III finger clubbing. Rales are present all over the chest on auscultation. The chest X-ray shows a characteristic honeycomb appearance. What is the most likely diagnosis? Answer Choices 1 COPD 2 Tuberculosis 3 Bronchiectasis 4 Pneumonia 5 Cystic fibrosis

Bronchiectasis The clinical picture along with clubbing and the honeycomb appearance on chest X-ray is diagnostic of bronchiectasis. Bronchiectasis is a disease caused by irreversible dilatation of the bronchial tree. Pathogenesis can be obstruction, congenital disorder, and infection. It can develop as a result of severe bacterial infections in childhood, often as a complication of whooping cough or measles. It can be cylindrical, varicose, or cystic, and it corresponds to the severity of degree of bronchiectasis. Typically, the patient has recurrent respiratory infections, usually cough with lots of sputum production. There will be clubbing of fingers and cyanosis (depending on the severity), and it presents at a later age compared to cystic fibrosis. Chest X-ray shows a typical honeycomb appearance. Diagnosis can be confirmed by CT scan or bronchography. Treatment is by antibiotics and assisting postural drainage. Cystic fibrosis presents with associated symptoms of malabsorption and infertility. COPD is associated with a history of smoking. It will usually have a normal X-ray, as in bronchitis, or be seen with associated features of emphysema (e.g., bullae and hyper-translucent lung field with loss of peripheral vascular markings). Pneumonia will have an acute history, and recurring infections are not characteristic. X-ray will be diagnostic. Tuberculosis will also be diagnosed on the X-ray. Bronchiectasis may develop secondary to a tuberculous hilar lymph node obstructing a major bronchus.

A woman is leaving with her 1-year-old son to go grocery shopping. She lives in a condominium with a small covered garage. As she starts her car, her telephone rings. She runs into the house to answer the phone, leaving her son safely buckled into his car seat and the car engine still running. When she returns to her car, she finds her son still buckled into his car seat. He is unconscious. She calls the paramedics. They tell her that the boy was exposed to noxious fumes. Question What will the child's mother notice about his skin? Answer Choices 1 Cherry-red (hyperemic) 2 White as a ghost (anemic) 3 Slate gray (pigmented) 4 Yellow (jaundiced) 5 Blue (cyanotic)

Cherry-red (hyperemic) The child inhaled carbon monoxide from the exhaust fumes of the car. A car running in a small covered garage will produce dangerous levels of carbon monoxide in a short period of time. Carbon monoxide has a much higher affinity for hemoglobin than oxygen (200-300 times). This leads to cellular hypoxia. The cherry red color is secondary to the carboxyhemoglobin. Paleness, a slate gray skin color, jaundice, and cyanosis are not seen with carbon monoxide poisoning.

A 1 day-old boy develops progressing abdominal distension, bilious vomiting and failure to pass a meconium stool. Abdominal radiographs show dilated loops of small bowel. Which of the following is the most likely diagnosis in this patient? A Cystic Fibrosis B Hypothyroidism C Imperforate anus D Intussusception E Pyloric Stenosis

Cystic Fibrosis The patient presents with a meconium ileus consistent with a diagnosis of cystic fibrosis (A). Pyloric stenosis (E) typically presents between 3 and 6 months of age, while intussusception (D) presents later (6 to 24 months). Imperforate anus (C) presents at birth, but infants typically lack acute abdominal distention and bilious vomit. Other common causes of intestinal obstruction in a newborn include meconium plug syndrome, Hirschsprung Disease, Intestinal Atresia, and Midgut Volvulus.

A 42-year-old woman presents complaining of shortness of breath, three days of fever as high as 103F, and a cough productive of green sputum. On physical examination, you hear crackles in her lungs. A chest radiograph reveals a consolidation in the left lower lobe. What tactile fremitus findings do you expect to observe in the left lower lobe region? A Absent B Decreased C Hyperresonant D Increased E Normal

D increased Tactile fremitus is increased (D) by processes that cause consolidation within the lung parenchyma and decreased (B) by processes that insulate the lung parenchyma from transmitting vibration (e.g., pneumothorax, hemothorax) or airway obstruction.

Which of the following treatments is first-line therapy for sputum culture-positive Legionnaire's pneumonia in an immunocompetent patient? A ampicillin/sulbactam B erythromycin C ceftriaxone D vancomycin E clindamycin

Erythromycin First-line therapy for legionella pneumonia (mild to moderate) in the immunocompetent host is erythromycin 500 mg to 1 g IV qid or 500 mg po qid for 14 to 21 days. Another option for first-line therapy is doxycycline 200 mg IV or po once daily for 14 to 21 days. Alternatives include levofloxacin 500 mg IV or po q day for 7 to 10 days or azithromycin 500 mg IV or po q day for 3 days. Severe infection or treatment in the immunocompromised patient is levofloxacin or azithromycin.

A 1 day-old boy develops progressing abdominal distension, bilious vomiting and failure to pass a meconium stool. Abdominal radiographs show dilated loops of small bowel. Which of the following genetic mutations should the patient be evaluated for? A ΔF508 B G551D C Q1412X D R117H E W1282X

G551D Mutations in CFTR protein function resulting from genotype G551D mutations are amenable to treatment with ivacaftor (B). Approximately 5% of CF patients have the G551D mutation and all patients should be assessed for potential ivacaftor therapy. ΔF 508 (A) is the most common genotype occurring in 60-66% of all CF patients and is not amenable to ivacaftor therapy.

A 76 year-old woman with steroid dependent chronic obstructive pulmonary disease is hospitalized with fever, chills, and a productive cough. The sputum gram stain shows many WBCs and small, pleomorphic gram-negative rods. Which of the following is the most likely causative agent? A Chlamydia pneumoniae B Haemophilus influenzae C Mycoplasma pneumoniae D Staph aureus E Strep pneumoniae

H influenzae Haemophilus influenzae (B) is a gram-negative pleomorphic coccobacillus. Strep pneumonia (E) and Staph aureus (D) are gram positive organisms. Mycoplasma pneumonia (C) and Chlamydia pneumoniae (A) aren't visible on gram stain.

A 41-year-old woman presents due to worsening symptoms. She was diagnosed with idiopathic pulmonary hypertension about 2 years prior to presentation; she is on home oxygen therapy. She has longstanding fatigue and dyspnea, but she is now experiencing profound dyspnea with exertion; swelling in her ankles; some discomfort in her right, upper abdomen; and the inability to breathe well when lying down. She has always been thin, but her weight has increased by 10 pounds in the last month. She denies fever and chills. She recently had an electrocardiogram (ECG), but she has not seen a healthcare provider to discuss the results. The ECG report indicates peaked p waves, right axis deviation, and tall R wave in V1. Question Based on this patient's history and test results, what physical exam findings would be expected? Answer Choices 1 Abdominal bruit 2 Absent breath sounds in right lung 3 Dry mucus membranes and reduced skin turgor 4 Hepatojugular reflux 5 Virchow's node enlargement

Hepatojugular reflux This patient is presenting with a progression of a primary pulmonary disease (pulmonary hypertension) into cor pulmonale, which is also known as pulmonary heart disease. Cor pulmonale, when moderate to severe, will present with signs and symptoms of right heart failure, such as hepatojugular reflux, peripheral edema, prominent P2, hepatomegaly, cyanosis, wheezing, and ascites. The ECG findings indicate right ventricular hypertrophy, which is a complication of progressing primary lung disease. Depending on the location in which the bruit was auscultated, an abdominal bruit might indicate atherosclerosis in the renal arteries, aorta, or iliac arteries. Atherosclerosis is typically seen in much older patients. This patient's pulmonary hypertension is not directly associated with atherosclerosis, and bruits would not be expected with her physical exam. Absent breath sounds are a finding associated with several pulmonary conditions, such as pneumothorax or a severe pneumonia; they are also seen in patients with a history of pneumonectomy. A pneumothorax or pneumonia might have some aspects in common with this patient's history (such as dyspnea), but they would not be associated with the fluid retention and ECG changes. Dry mucus membranes and reduced skin turgor are indicators of severe dehydration. Nothing about this patient's history is consistent with dehydration. On the contrary, this patient is in a fluid-overload state. Virchow's node enlargement indicates left-sided supraclavicular lymphadenopathy. This finding is a harbinger for malignancy of the thorax. The patient's symptoms do not suggest malignancy. If malignancy was present, depending on the primary type of cancer, the patient might exhibit unexplained weight loss and possibly night sweats.

A 72-year-old man presents to the outpatient clinic in follow-up for his dyspnea and cough. He reports shortness of breath, especially with activity, and a cough, which is non-productive. Symptoms have been present for 1 year, and they are getting worse. He initially went to the cardiologist for heart concerns, but no cardiovascular disease was found. A chest X-ray was ordered, and the patient reports it showed no masses in his chest. The patient denies any other symptoms, including fever, chills, night sweats, chest pain, and weight loss. The patient is a retired salesman; he fishes as a hobby. He lives at home with his wife; he denies use of tobacco, alcohol, and drugs. He denies any out of the country travel. On physical exam, the patient sits comfortably with normal respiratory effort. Auscultation of his lungs reveals fine crackles in both bases. A dry cough is noted a few times. Cardiovascular exam, including heart and extremities, is normal, except for clubbing of the fingers bilaterally. Question What test is the next most appropriate in evaluation of this patient's condition? Answer Choices 1 Echocardiogram 2 High-resolution computed tomography (CT) 3 Mantoux test or purified protein derivative (PPD) 4 Ultrasound of the thorax 5 Ventilation-perfusion (VQ) scan

High resolution CT This patient has a likely diagnosis of idiopathic pulmonary fibrosis (IPF). This is a relatively rare disease and can be difficult to diagnose. Patients typically have symptoms for 1 - 2 years prior to definitive diagnosis. IPF is a chronic, progressive restrictive pulmonary disease of the lung parenchyma. Unlike pneumoconiosis, there are no known occupational exposures (hence the term idiopathic). IPF presents with exertional dyspnea and a non-productive cough. A high-resolution computed tomography (CT) and/or a lung biopsy are the means for definitive diagnosis. The CT may show reticular opacities, honeycombing, and perhaps a 'ground glass' appearance in the lung tissue. There are heterogeneous areas of diseased lung interspersed with healthy tissue. An echocardiogram is not helpful in diagnosing IPF. It can be assumed that evaluation by a cardiologist would have determined if an echocardiogram was necessary. This patient was cleared by a cardiologist and has only pulmonary complaints (dyspnea and cough, with findings of lung crackles and clubbing). The CT should be done next, and if not consistent with IPF, further testing with an echocardiogram should be considered. The Mantoux test or purified protein derivative (PPD) is a skin test for tuberculosis. This patient's presentation is not consistent with tuberculosis (TB), so a PPD would not be recommended. Chronic cough, fever, night sweats, weight loss, along with TB risk factors (such as nursing home setting, foreign birth or travel, etc.) would indicate PPD. Ultrasound of the thorax would not be helpful in evaluating possible IPF. In fact, ultrasound of the thoracic structures (outside of echocardiography) is not often done. A plain chest film (X-ray) is a good starting point for pulmonary complaints, followed by a CT as the next appropriate test. A ventilation-perfusion scan is a test that is primarily used in the diagnosis of pulmonary embolism (PE). This patient is not presenting with a history consistent with a PE (which may include acute dyspnea and chest pain).

A 67-year-old man presents for a routine follow-up visit. His past medical history includes chronic obstructive pulmonary disease (COPD) for the past 3 years and hypertension (HTN) for the past 5 years. Daily medications include a tiotropium inhaler and hydrochlorothiazide. He has smoked a pack of cigarettes a day for 50 years, and he has no desire to quit at this time. Physical exam remains unchanged from 3 months ago, and the patient has no complaints. His last pneumococcal and influenza vaccinations were 2 years ago. Question What should be offered at this visit? Answer Choices 1 Bupropion 2 Influenza vaccine 3 Fluticasone and salmeterol 4 Nicotine patch 5 Pneumococcal vaccine

Influenza vaccine Chronic obstructive pulmonary disease (COPD) affects approximately 32 million Americans and is the 4th leading cause of death in the US (1). The main cause of COPD is smoking. For this reason, COPD is considered a preventable disease. "The Global Initiative for Chronic Obstructive Lung Disease (GOLD) guidelines define COPD as a disease state characterized by airflow limitation that is not fully reversible, is usually progressive, and is associated with an abnormal inflammatory response of the lungs to inhaled noxious particles or gases" (Mosenifar, 2011, Overview). Treatment of COPD is aimed at limiting exacerbations and progression of disease. Smoking cessation is an important component of the treatment of COPD, but it will only be effective if the patient is willing to attempt to quit. Nicotine patches or gum can be used as nicotine replacement therapy to aid in smoking cessation. The dose of nicotine is gradually decreased. Bupropion (Zyban) is an antidepressant that can be used in conjunction with behavioral therapy to help in smoking cessation (1). Pharmacological treatment of stable COPD includes treating the airflow restriction and inflammation. Inhaled bronchodilators, such as beta-agonists and anticholinergic agents, are the mainstay of therapy. "Tiotropium is the only long-acting muscarinic agent available at this time and has become a first-line therapy in patients with persistent symptoms" (Mosenifar, 2011, Treatment). Combination therapy of inhaled beta-agonist and corticosteroids or inhaled anticholinergic and corticosteroids are indicated when a patient's symptoms are not controlled by monotherapy alone (1). In this case, the patient is controlled on tiotropium, so the addition of fluticasone and salmeterol (corticosteroid and long-acting beta-agonist) is not warranted. Pulmonary infections can lead to a COPD exacerbation; therefore, patients should routinely receive pneumococcal and influenza vaccines. The CDC recommends adults receive 1 dose at age 65 years. In the case presented, the patient had his vaccine at age 65 and does not need another one. The current recommendation for the influenza vaccine is that everybody receives an annual dose (2). Therefore, the patient in this case should be given an influenza vaccine at this visit.

You receive notice that one of your patients, a 2-month-old boy, was found dead in his parents' bed. Sudden Infant Death Syndrome (SIDS) is suspected. He was just seen for his 2-month well baby check and seemed healthy and thriving, although he was recovering from a mild upper respiratory infection. Everything looked consistent with a mild cold on his exam and he was afebrile. You recall that his mother mentioned she had returned to work part-time and found a family care home for him while she worked. You remember counseling about back to sleep care and other risks for SIDS. Although she is a smoker, she had cut back during her pregnancy and said she now only smokes outside. She seemed receptive, but she was also preoccupied with her 2 other small children present and seemed very tired. What major risk factor in almost every epidemiologic study of SIDS may have influenced this case? Answer Choices 1 Bedsharing 2 Care by secondary caregivers 3 Presence of soft objects in crib 4 Pacifier use 5 Maternal smoking during pregnancy

Maternal smoking during pregnancy SIDS is considered the sudden death of an infant under the age of 1 year that is unexplained by thorough case investigation, autopsy, and review of case history. The occurrence peaks between 2 - 3 months of age. Since 1994, when the Back to Sleep campaign began recommending placing infants in a supine position, there has been a consistent decrease in SIDS rate; however, SIDS is still responsible for more infant deaths in the United States than any other cause of death during infancy beyond the neonatal period. The predominant hypothesis concerning the etiology remains that certain infants may have a maldevelopment or delay in maturation of the brainstem neural network responsible for arousal, and this affects the physiologic response to life-threatening challenges during sleep. Independent risk factors for SIDS that have been consistently identified across studies are prone sleep position, sleeping on a soft surface, overheating, late or no prenatal care, young maternal age, preterm birth and/or low birth weight, male gender, and maternal smoking during pregnancy. Almost every epidemiologic study has shown maternal smoking during pregnancy to be a major risk factor. Smoking after birth has emerged as a separate risk factor, but separating this variable from maternal smoking before birth is difficult. Bed sharing is highly controversial. Although it facilitates breastfeeding and enhances the infant-maternal bond, it can be hazardous under certain conditions. SIDS risk seems to be particularly high with multiple bed sharers, especially if alcohol is consumed. It can be extremely hazardous when sleeping with an infant on a couch. The longer the duration of bed sharing is during the night, the higher the risk. There is, however, evidence that room sharing without bed sharing is the most protective sleep setting for the 1st 6 months and should be recommended. Approximately 20% of SIDS deaths occur while in the care of a non-parental caregiver. Many of these deaths have been associated with the prone sleep position, especially when the infant is accustomed to sleeping supine. This has shown to actually increase the risk of SIDS 18-fold. Some centers are unaware of the dangers of sleeping prone and/or are misinformed of the risks and benefits of sleep positions. Studies have shown that infants dying of SIDS were more likely to have used a pillow or soft mattress, and they were found with their nose and mouth completely covered by bedding or assumed a facedown posture. Soft objects such as stuffed toys, quilts, blankets, and loose bedding should be kept out of the crib. Blankets should be tucked in around the crib mattress. Several new studies have reported an actual protective effect of pacifiers on the incidence of SIDS. The mechanism is unclear, but lowered arousal threshold among users has been proposed. Others theorize that pacifier use may enhance an infant's ability to breathe through the mouth if the nasal airway becomes obstructed. Retroposition of the tongue can lead to obstructive apnea and asphyxiation; sucking of the pacifier requires forward positioning of the tongue, decreasing risk of oropharyngeal obstruction.. There has also been a concern raised about an approximate 1 - 2 fold increased risk of otitis media associated with pacifier use, but this incidence is generally lower in the 1st year of life, especially in the 1st 6 months, which is when the risk of SIDS is highest.

An 82-year-old man presents with difficulty breathing and chest pain. He admits to poor follow-up with his primary care physician; he has not been seen by his doctor in several years. When last seen, his health issues included hypertension, hyperlipidemia, diabetes mellitus type II, and being grossly overweight. His prescriptions ran out, so he has not been on any treatment for these issues for several years. Chest X-ray done at the bedside reveals significant bilateral effusions that are initially felt to be secondary to congestive heart failure based on various other findings. A pulmonologist is consulted and performs bilateral thoracentesis to help alleviate the patient's symptoms. The right side is significantly more affected than the left, with over 1000mL drained versus about 625mL on the left. Question During initial testing, what finding was most indicative of these thoracentesis results? Answer Choices 1 Bilateral decreased tactile fremitus 2 Mediastinum shifted to the right 3 Mediastinum shifted to the left 4 Pleural friction rub heard throughout 5 Bilateral inaudible breath sounds

Mediastinum shifted to the left Mediastinum shifted to the left is the correct answer. Patients with a pleural effusion that is over 1000mL can experience a shift in the mediastinum away from the side with the larger effusion. A pleural effusion is an accumulation of fluid within the pleural cavity, which is the area between the chest wall and the lung. The increased pressure within the affected side of the chest causes the mediastinum, including the trachea, to shift away from the side with the larger amount of fluid (and therefore larger amount of pressure). In this case, the mediastinum would shift towards the left, as the right side has almost twice the amount of fluid as the left. Decreased tactile fremitus is not the correct answer. Tactile fremitus is vibration felt on a patient's chest while the patient vocalizes at low frequency. The patient is asked to repeat the same phrase over and over while the examiner moves their palm throughout the various lung fields. Tactile fremitus can be decreased in cases of any pathology separating the lung tissue from the chest wall. Pleural effusion is a common cause of decreased tactile fremitus. It is incorrect in this case because a physical examination is very nonspecific. The examiner would not necessarily know from assessing tactile fremitus the amount of pleural effusion or the differentiation between the sides. Mediastinum shifted to the right is not the correct answer. If the patient's mediastinum was shifted toward the right in this case, then it would be shifted toward the affected side. In patients with large pleural effusions, the side with the higher amount of pressure pushes the shift away from the affected side. Patients with atelectasis, pleural fibrosis, or who are status post-pneumonectomy would experience a decreased pressure within the pleural cavity on 1 side; therefore, the mediastinum would be shifted toward that affected side. Pleural friction rub is not the correct answer. A pleural friction rub is heard on cardiac auscultation and is the result of the pleural linings rubbing together. In cases when the pleural linings are inflamed (such as in pneumonia, pleural effusion, or pulmonary embolism), a pleural friction rub can be heard. The finding is nonspecific in terms of the amount of fluid or differentiation between sides; therefore, it would not necessarily be indicative of these particular thoracentesis findings. Bilateral inaudible breath sounds is not the correct answer. Due to the increased fluid and pressure associated with pleural effusion, the breath sounds may not be heard during physical examination. However, not hearing the sounds on either side does not give any indication as to how severe the pleural effusion is (although this typically happens in the larger effusions) and certainly does not show the differentiation between sides.

An 82-year-old man presents with difficulty breathing and chest pain. He admits to poor follow-up with his primary care physician; he has not been seen by his doctor in several years. When last seen, his health issues included hypertension, hyperlipidemia, diabetes mellitus type II, and being grossly overweight. His prescriptions ran out, so he has not been on any treatment for these issues for several years. Chest X-ray done at the bedside reveals significant bilateral effusions that are initially felt to be secondary to congestive heart failure based on various other findings. A pulmonologist is consulted and performs bilateral thoracentesis to help alleviate the patient's symptoms. The right side is significantly more affected than the left, with over 1000mL drained versus about 625mL on the left. Question During initial testing, what finding was most indicative of these thoracentesis results? Answer Choices 1 Bilateral decreased tactile fremitus 2 Mediastinum shifted to the right 3 Mediastinum shifted to the left 4 Pleural friction rub heard throughout 5 Bilateral inaudible breath sounds

Mediastinum shifted to the left is the correct answer. Patients with a pleural effusion that is over 1000mL can experience a shift in the mediastinum away from the side with the larger effusion. A pleural effusion is an accumulation of fluid within the pleural cavity, which is the area between the chest wall and the lung. The increased pressure within the affected side of the chest causes the mediastinum, including the trachea, to shift away from the side with the larger amount of fluid (and therefore larger amount of pressure). In this case, the mediastinum would shift towards the left, as the right side has almost twice the amount of fluid as the left. Decreased tactile fremitus is not the correct answer. Tactile fremitus is vibration felt on a patient's chest while the patient vocalizes at low frequency. The patient is asked to repeat the same phrase over and over while the examiner moves their palm throughout the various lung fields. Tactile fremitus can be decreased in cases of any pathology separating the lung tissue from the chest wall. Pleural effusion is a common cause of decreased tactile fremitus. It is incorrect in this case because a physical examination is very nonspecific. The examiner would not necessarily know from assessing tactile fremitus the amount of pleural effusion or the differentiation between the sides. Mediastinum shifted to the right is not the correct answer. If the patient's mediastinum was shifted toward the right in this case, then it would be shifted toward the affected side. In patients with large pleural effusions, the side with the higher amount of pressure pushes the shift away from the affected side. Patients with atelectasis, pleural fibrosis, or who are status post-pneumonectomy would experience a decreased pressure within the pleural cavity on 1 side; therefore, the mediastinum would be shifted toward that affected side. Pleural friction rub is not the correct answer. A pleural friction rub is heard on cardiac auscultation and is the result of the pleural linings rubbing together. In cases when the pleural linings are inflamed (such as in pneumonia, pleural effusion, or pulmonary embolism), a pleural friction rub can be heard. The finding is nonspecific in terms of the amount of fluid or differentiation between sides; therefore, it would not necessarily be indicative of these particular thoracentesis findings. Bilateral inaudible breath sounds is not the correct answer. Due to the increased fluid and pressure associated with pleural effusion, the breath sounds may not be heard during physical examination. However, not hearing the sounds on either side does not give any indication as to how severe the pleural effusion is (although this typically happens in the larger effusions) and certainly does not show the differentiation between sides.

A 4-year-old girl, adequately immunized for her age, has a history of repeated pyogenic infections. She had normal antibody responses following childhood immunizations. A defect in what cells best explains the cause of her infections? Answer Choices 1 B-lymphocytes 2 T lymphocytes 3 Eosinophils 4 Macrophages 5 Neutrophils 6 Basophils

Neutrophils Pyogenic infections are generally caused by bacteria. The entry of bacteria into the body triggers a number of responses: activation of the complement pathway and phagocytosis by neutrophils and macrophages. Macrophages play a major role in chronic granulomatous infections, and neutrophils predominate in acute pyogenic infections. An adequate response to childhood immunizations indicates an adequate capability of antibody production, and recovery from viral infections like measles indicates appropriate cell-mediated immunity. Therefore, the defect must lie in the neutrophils

A 29-year old patient presents with a 9-month history of recurrent hemoptysis. The patient suffered from cavitary tuberculous infection 5 years ago and was effectively treated with antituberculous drugs. A thin-walled cavity was seen over the right upper lobe when first diagnosed. At a later date, a progressive opacification of tuberculous cavity was seen. What organism is most likely responsible for such radiologic changes of a tuberculous cavity? Answer Choices 1 Secondary growth of staphylococci 2 Growth of Candida albicans might have filled the cavity 3 Growth of Aspergillus fumigatus might have filled the cavity 4 The cavity filled with granulation tissue and subsequent fibrosis 5 The cavity filled with desquamated epithelial cells of alveoli

One of the complications of tuberculous cavity is the growth of Aspergillus fumigatus within the cavity (fungal ball). Diagnosis is made by isolation of Aspergillus hyphae in the sputum. IgG antibody against Aspergillus can be demonstrated in such patients.

What is the laboratory test that has the best predictability for diagnosing pneumonia caused by adenovirus? A Nasal washings B Viral DNA test C Complete blood count with differential D Sputum gram stain E Polymerase chain reaction test (PCR)

PCR PCR testing is the best method for determining adenovirus pneumonia. It has a sensitivity of 90 to 100% and the specificity is >95%.

A patient recovering from hip surgery begins to ambulate for the first time about 2 hours postoperatively. Suddenly, she experiences shortness of breath. She becomes tachypneic and tachycardic; she experiences pain on inspiration. Prior to getting out of bed, her postoperative course was unremarkable. There is no swelling; there is no palpable thrill at the incision site. What is your initial diagnosis? Answer Choices 1 Postoperative pneumonia 2 Atelectasis 3 Arteriovenous fistula at surgical site 4 Pulmonary embolism 5 Abdominal aortic aneurysm rupture

PE The clinical picture is suggestive of a pulmonary embolism (PE). Pulmonary embolisms are a serious and potentially fatal complication of thrombi formation, usually in the lower extremities. Risk factors include venous stasis, hypercoagulability, and injury to the vessel wall; they are known as Virchow's triad. Venous stasis increases with immobility, in this case bed rest, especially postoperatively. Clinical findings include dyspnea, chest pain, syncope, hemoptysis, tachypnea, and pain on inspiration. Postoperative pneumonia usually occurs after 48 hours postoperatively, and the patient will present with fever on day 3 or 4 postoperatively. Atelectasis is a common postoperative complication (collapse of lung tissue), especially after chest or abdominal surgery. Arteriovenous fistula is an abnormal connection between an artery and a vein; a palpable thrill can be felt at the incision site. An abdominal aortic aneurysm rupture presents with severe pain, a palpable abdominal mass, and hypotension.

A 45-year-old man is on chemotherapy. He starts to notice that he is short of breath and has developed a nonproductive cough. Bronchial lavage confirms that he has an infection. What is the most likely pathogen? Answer Choices 1 Pneumocystis jiroveci 2 Giardia lamblia 3 Babesia microti 4 Trichophyton tonsurans 5 Trypanosoma cruzi

Pneumocystis jiroveci Pneumocystis jiroveci (formerly called Pneumocystis carinii) is a fungus that can cause pneumonia in an immunosuppressed host. It is transmitted by inhalation. Giardia lamblia is a protozoon that can cause gastrointestinal discomfort, malabsorption, and diarrhea. It is transmitted by the fecal oral route and by drinking infected water. Babesia microti is a protozoon that can cause fever. In a vulnerable host, it occasionally causes a more severe illness. It is transmitted by a tick. Trichophyton tonsurans is fungus that causes tinea capitis. This is a dermatophytosis of the scalp and other areas with hair. Trypanosoma cruzi is a protozoon that causes Chagas' disease. It is transmitted by bugs.

A 49-year-old man presents with chronic fatigue. He states that, despite obtaining 7 - 9 hours of sleep nightly, he wakes up feeling unrested and sometimes with a headache. Occasionally, he falls asleep in the middle of the day. He is not sure if he snores at night, but he has been told that he snores when he is napping. He reports no other changes in general condition. His other medical problems are hypertension and hypercholesterolemia, for which he takes metoprolol and simvastatin, respectively. Physical exam reveals a moderately overweight man in no apparent distress. Blood pressure, heart rate, and resting oxygen saturation are within normal limits. Question Based on the patient's symptoms, what diagnostic studies would be most useful? Answer Choices 1 24-hour Holter monitoring 2 Echocardiogram 3 Electrocardiogram 4 Electroencephalogram 5 Polysomnography

Polysomnography

A 72-year-old man presents with an 8-month history of progressive dyspnea, which has been accompanied by a dry and persistent hacking cough. While the dyspnea now occurs at rest, he denies fever, chills, palpitations, chest pain, or peripheral edema. He states that he has worked for many years at a local chemical plant. His physical exam is remarkable for digital cyanosis and clubbing, while his pulmonary exam reveals diffuse fine, dry inspiratory crackles. His cardiac exam was positive for a prominent pulmonary valve closure sound (P2) and an elevated jugular venous pressure of 6 cm. A chest x-ray noted small lung volumes, with increased densities in the lung periphery and a honeycombing pattern; pulmonary function testing measured reductions in TLC, FEV1, and FVC with a preserved FEV1/FVC ratio. Question What is the best treatment for this patient at this time? Answer Choices 1 Erythromycin 2 Furosemide 3 Lung transplantation 4 Prednisone 5 Cyclophosphamide

Prednisone This patient most likely has hypersensitivity pneumonitis (HP), also known as extrinsic allergic alveolitis. This is an inflammatory disorder of the lung involving alveolar walls and terminal airways that is induced by repeated inhalation of a variety of organic agents. The chronic form of HP typically results from low-grade or recurrent exposure over many months to years, and the lung disease may already be partially or completely irreversible. These patients are usually advised to avoid all possible contact with the offending agent. In addition to identification of the causative agent and its avoidance, institution of glucocorticoid treatment is indicated. Prednisone at a dosage of 1 mg/kg per day or its equivalent is continued for 7 - 14 days. It is then tapered to 0.25-0.5 mg/kg and is maintained at this level for an additional 4 - 12 weeks at a rate that depends on the patient's clinical status. While patients with chronic HP may gradually recover without therapy following environmental control, a trial of prednisone may be useful to obtain maximal reversibility of the lung disease. Following initial prednisone therapy (1 mg/kg per day for 2 to 4 weeks), the drug is tapered to the lowest dosage that will maintain the functional status of the patient. Many patients will not require or benefit from long-term therapy if there is no further exposure to the antigen. Improvement of lung function may continue over a few months to years. If the patient's condition continues to decline on glucocorticoids, a second agent should be introduced while lowering or maintaining the prednisone dose at 0.25 mg/kg per day. Glucocorticoid therapy is recommended for symptomatic interstitial lung disease (ILD) patients with eosinophilic pneumonias, cryptogenic organizing pneumonia, connective tissue diseases, sarcoidosis, hypersensitivity pneumonitis, acute inorganic dust exposures, acute radiation pneumonitis, diffuse alveolar hemorrhage, and drug-induced ILD. In organic dust disease, glucocorticoids are recommended for both the acute and chronic stages.

A 6-year-old boy presents with fever and cough. He has history of several episodes of pneumonia. A sweat test reveals an increased amount of chloride, indicating that he has cystic fibrosis. He is coughing up thick, greenish sputum. Temperature is 37.6° C. A Gram stain of the sputum reveals Gram-negative rods and a culture grows a Gram-negative rod that is oxidase-positive and produces a blue-green pigment. What is the most likely cause of the infection? Answer Choices 1 Pseudomonas aeruginosa 2 Haemophilus influenzae 3 Bordetella pertussis 4 Legionella pneumophila 5 Streptococcus pneumoniae

Pseudomonas Pseudomonas aeruginosa is a Gram-negative, oxidase-positive rod that does not ferment glucose, with a typical metallic sheen of the growth on TSI agar, coupled with the blue-green pigment on ordinary nutrient agar and a fruity aroma. P. aeruginosa produces 2 pigments: (1) pyocyanin which can color the pus in a wound blue, and (2) pyoverdin (fluorescein), a yellow-green pigment that fluoresces under ultraviolet light. P. aeruginosa is the only species of Pseudomonads that synthesizes pyocyanin. Pseudomonads are able to grow in water containing only traces of nutrients, and they have a remarkable ability to withstand disinfectants. Strains of P. aeruginosa isolated from cystic fibrosis patients have a prominent slime layer (glycocalyx), which gives their colonies a very mucoid appearance. P. aeruginosa is primarily an opportunistic pathogen that causes infections in hospitalized patients, those with chronic respiratory disease in whom the normal clearance mechanisms are impaired, immunosuppressed patients with neutrophil counts of less than 500/μL, and patients with indwelling catheters. P. aeruginosa can cause infections virtually anywhere in the body, but urinary tract infections, pneumonia (especially in cystic fibrosis patients) and wound infections (especially burns) predominate. Haemophilus influenzae is a small Gram-negative rod with a polysaccharide capsule. It is the leading cause of meningitis in young children, and it is an important cause of upper respiratory tract infections and sepsis in children; also, it causes pneumonia in adults, particularly those with chronic obstructive lung disease. Bordetella pertussis is a small, coccobacillary, encapsulated Gram-negative rod. It causes whooping cough (pertussis). Legionella pneumophila is Gram-negative rod that stain faintly with the standard Gram stain. Sputum Gram stains reveal many neutrophils, but no bacteria. The organism fails to grow on ordinary media in a culture of blood or sputum, but it will grow on medium supplemented with iron and cysteine. L. pneumophila causes atypical pneumonia, both in the community and in hospitalized immunocompromised patients. Streptococcus pneumoniae causes pneumonia, bacteremia, meningitis and infections of the upper respiratory tract. Pneumococci are Gram-positive, lancet-shaped cocci arranged in pairs or short chains.

A 33-year-old African-American woman with no significant past medical history, who is in her 38th week of a normal pregnancy, presents with a 1-hour history of shortness of breath. She does not recall any precipitating activities or events that may have provoked these symptoms and recalls that she was laying in her bed for several hours since waking when the symptoms developed. She admits to an associated sharp, nonradiating pleuritic chest pain, as well as lower extremity swelling, which she states has been "persistent throughout the course of her pregnancy". She denies palpitations, chest pressure, cough, sputum, fever, chills, changes in weight, rashes, diaphoresis, abdominal pain, nausea, or a history of allergies. Her physical exam is noteworthy for tachypnea and tachycardia; however, the rest of the vital signs are normal. Her lungs are clear to auscultation bilaterally, without wheezing, rhonchi, or crackles. Her lower extremities are remarkable for 2+ pitting edema up to the level of her knees; there is no calf tenderness, venous cords, or Homan's sign appreciated. Her skin and mucous membranes were without diaphoresis or cyanosis. A bedside EKG performed reveals sinus tachycardia at 120 bpm with prominent S waves in lead I and Q waves in lead III. Question What is the most likely diagnosis? Answer Choices 1 Acute myocardial infarction 2 Bacterial pneumonia 3 Pulmonary edema 4 Pulmonary embolism 5 Bronchial asthma

Pulmonary embolism This patient has a presentation most consistent with pulmonary embolism. Nearly all PEs arise from deep venous thrombosis (DVT) in the lower extremity or pelvic veins. Risk factors for DVT and PE are similar in children and adults and include conditions that impair venous return, conditions that cause endothelial injury or dysfunction, and underlying hypercoagulability disorders. Bed rest and confinement without walking, even for a few hours, are common precipitators. Specific risks include an age of greater than 60, atrial fibrillation, cigarette smoking (including passive smoke), estrogen receptor modulators, exogenous estrogens, and progestins (including oral contraceptives and estrogen therapy), extremity or pelvic trauma, heart failure, hypercoagulability disorders, immobilization, indwelling venous catheters, myeloproliferative disorders, nephrotic syndrome, obesity, pregnancy and postpartum states, sickle cell anemia, recent surgeries, and prior venous thromboembolism. Larger emboli cause acute dyspnea, pleuritic chest pain, or both. Dyspnea may be intermittent or occur only with exercise. Less common symptoms include cough and hemoptysis. The most common signs of PE are tachycardia and tachypnea. Less commonly, patients have hypotension, a loud 2nd heart sound (S2) due to a loud pulmonic component (P2), and crackles or wheezing. In the presence of right ventricular failure, distended internal jugular veins and a right ventricular heave may be evident, and right ventricular gallop (3rd and 4th heart sounds [S3 and S4]), with or without tricuspid regurgitation, may be audible. ECG most often shows tachycardia and various ST-T wave abnormalities, which are not specific for PE. An S1Q3T3 or a new right bundle branch block may indicate the effect of abrupt rise in right ventricular pressure on right ventricular conduction; these findings are moderately specific but insensitive for PE. Pulmonary edema typically causes severe dyspnea, the production of pink, frothy sputum, and diaphoresis and cyanosis. Rales are present in all lung fields, as are generalized wheezing and rhonchi. The chest X-ray usually indicates vascular redistribution, blurriness of vascular outlines, increased interstitial markings, and, characteristically, the butterfly pattern of distribution of alveolar edema. The patient's younger age, quality of chest pain, lack of gastrointestinal and skin findings, and absence of characteristic EKG findings make acute myocardial infarction a less likely diagnosis. The rapid onset of symptoms and denial of fever, cough, and sputum suggest a diagnosis other than bacterial pneumonia, as does the absence of an allergic history and adventitious lung sounds.

A 17-year-old boy presents with intermittent bouts of shortness of breath, coughing, and chest tightness. The symptoms most often occur during football practice, but sometimes they also occur when he is just standing outdoors in cold weather. He denies palpitations, fever, and chills. Past medical history is noncontributory. He is a nonsmoker. Physical exam is unremarkable. Vital signs are as follows: BMI 19 kg/m2; BP 116/70 mmHg; HR 80 bpm; and SpO2 99% on room air. His physician orders pulmonary function testing (PFTs) and a chest X-ray (CXR). Question What set of PFTs is most consistent with the diagnosis of asthma? Answer Choices 1 Reduced forced expiratory volume in 1 minute (FEV1), reduced FEV1/Forced vital capacity (FVC) ratio, and Reduced Residual Volume (RV) 2 Reduced FEV1, Reduced FEV1/FVC ratio, and Increased RV 3 Reduced FEV1, Increased FEV1/FVC ratio, and Increased RV 4 Reduced FEV1, Increased FEV1/FVC ratio, and Reduced RV 5 Increased FEV1, Increased FEV1/FVC ratio, and Reduced RV

Reduced FEV1, Reduced FEV1/FVC ratio, and Increased RV Asthma is a chronic condition that affects pulmonary function. The pathophysiologic process underlying asthma is inflammation. In response to specific triggers, an immune-mediated cascade occurs that leads to recruitment of inflammatory mediators and airway constriction. Although this is a normal response to airway irritants, the airway of patients with asthma is hyperresponsive. Bronchial constriction increases the work of breathing, particularly the exhalation phase. This is manifested by reduced FEV1 (volume of air that can be forcibly exhaled in the first second of exhalation after maximal inspiration) and reduced FEV1/FVC ratio (FVC = volume of air that can be forcibly blown out after a maximal inspiration). Inability to adequately exhale leads to air trapping, which causes an increase in residual volume. Common treatments for asthma include inhaled beta-2 agonists, which promote relaxation of airway smooth muscle as well as inhaled corticosteroids that minimize inflammatory-mediated bronchoconstriction.

What is the most common cause of pneumothorax in a healthy patient? A Traumatic B Infectious C Ruptured bleb D Surfactant abnormality E Malignancy

Ruptured bleb

A 53-year-old man with a 40-pack/year smoking history presents with a 10-month history of intermittent cough with productive sputum. He admits to progressive exertional shortness of breath, which recently has limited his activity to climbing 1 flight of stairs or walking 3 city blocks. He denies diaphoresis, fever, chills, chest pain, palpitations, audible wheezing, pleurisy, peripheral edema, hemoptysis, abdominal pain, reflux, regurgitation, diarrhea, melena, or hematochezia. He also denies travel, sick contacts, and drug or alcohol use. His general survey reveals an overweight male with an odor of smoke and nicotine staining of his fingernails. His nails also demonstrate digital clubbing. His pulmonary exam reveals prolonged expiratory phase expiratory, barrel chest, poor diaphragmatic excursion, and wheezing to auscultation. Pulmonary function testing shows airflow obstruction with a reduction in FEV1 and FEV1/FVC ratio; an increase in total lung capacity, functional residual capacity, and residual volume were noted. Question What clinical intervention has been demonstrated to influence the natural history of this patient's illness? Answer Choices 1 Intravenous diuretics 2 Inhaled anticholinergic agents 3 Smoking cessation 4 Long-acting inhaled -agonists and glucocorticoids 5 Antibiotic prophylaxis

Smoking cessation This patient's diagnosis is most consistent with chronic obstructive pulmonary disease. Only 3 interventions: smoking cessation, oxygen therapy in chronically hypoxemic patients, and lung volume reduction surgery in selected patients with emphysema have been demonstrated to influence the natural history of patients with COPD. All other current therapies are directed at improving symptoms and decreasing the frequency and severity of exacerbations. Diuretics are not indicated in the management of COPD. Long-acting inhaled -agonists (e.g., salmeterol) and anticholinergic agents, such as tiotropium or ipratropium bromide, have been shown to improve symptoms and reduce exacerbations, but have failed to demonstrate that either influences the rate of decline in FEV1. Despite the frequent implication of bacterial infection in COPD exacerbations, chronic suppressive, or "rotating" antibiotics, are not beneficial in patients with COPD.

A thin 26-year-old woman presents with sudden onset of dyspnea, non-productive cough, and vague chest pain radiating to the left shoulder. She is a soccer player, and her symptoms initially occurred 24 hours ago during her usual 5-mile training run. PMH is unremarkable. EKG shows sinus tachycardia, 105 bpm. Respiratory rate equals 30 breaths per minute. Decreased breath sounds and hyperresonance are noted on the left thorax; otherwise, the physical exam is unremarkable. What is the most likely diagnosis? Answer Choices 1 Myocardial infarction 2 Spontaneous pneumothorax 3 Exercise-induced asthma 4 Dissecting aortic aneurysm 5 Atypical pneumonia

Spontaneous pneumothorax is more likely to occur in young, tall, thin individuals; a history of smoking is commonly present. Initial symptoms include sudden onset of chest pain, dyspnea, and cough; symptoms are usually associated with exertion. Chest X-rays will show air in the pleural space along the lung border on the affected side, without evidence of pleural effusion. Myocardial infarction is less likely in this age group, but it cannot be ignored and must be ruled out. In addition to the symptoms mentioned in this case, exercise-induced asthma typically presents with diffuse expiratory wheezing and prolonged expiratory phase. Dissecting aortic aneurysm may occur in individuals with Marfan's syndrome, which is also seen in tall, thin individuals. However, the classic presentation is that of sudden onset of severe chest pain, radiating to the back, with or without symptoms of hypovolemic shock. The lack of fever, prodromal period, fatigue, malaise, or other constitutional symptoms makes an infectious agent unlikely.

Which of the following is a major contraindication for surgical resection of a lung carcinoma? A Chest wall invasion B Non-malignant pleural effusion C Superior vena cava syndrome D Unilateral endobronchial tumor E Vagus nerve involvement

Superior Vena Cava syndrome Surgical resection of lung carcinoma is contraindicated in cases of superior vena cava syndrome, extrathoracic metastases, heart, pericardial or great vessel involvement, recurrent laryngeal or phrenic nerve involvement, esophageal or carina involvement, malignant effusion, or contralateral mediastinal lymph nodes. Other contraindications are patient and staging dependent.

A 56-year-old man presents with fatigue, fever, chills, and a productive cough. Symptoms began 3 days ago. The patient is drenched in sweat and has shaking chills. The patient has a 2 pack/day smoking habit. He has dyspnea (respiratory rate of 23/min) and pleuritic chest pain to his left side. Crackles are heard over the right middle, left middle, and left lower lung fields. X-rays are significant for right lower lobe, left lingular, and left lower lobe infiltrates. The patient has an elevated body temperature of 40.0° C. Sputum is collected for culture and Gram stain that is brown-green in color. A CBC and blood cultures are also ordered. The CBC results are significant for an elevated WBC of 14.0 x 109/L with a neutrophilic left shift on the differential. The sputum Gram stain contains >25 WBCs per lower-power field with no epithelial cells and many Gram-positive diplococci (refer to the image). The next day, the culture grows 4+ of an alpha hemolytic organism that is catalase negative and bile salt soluble. Blood cultures also turn positive that next day for Gram-positive diplococci. What is causing the pneumonia with septic complications? Answer Choices 1 Haemophilus influenzae 2 Streptococcus pneumoniae 3 Chlamydia pneumoniae (TWAR) 4 Candida albicans 5 Moraxella catarrhalis

Streptococcus pneumoniae is a Gram-positive diplococcus; it is catalase-negative, alpha hemolytic on BAP, PYR-negative, P-disc (optochin) sensitive, and bile salts soluble. The organism has an external capsule. It is a major cause of lobar pneumonia. It is also an important cause of a host of other diseases, such as otitis media, septicemia, sinusitis, and meningitis. Patients with pneumococcal pneumonia usually have an appearance that is described as grayish and noxious. They usually present with a body temperature of 102° -103° F, an elevated pulse rate, and a respiratory rate of 20-24 per minute. Chest radiographs usually reveal an area of infiltration involving less than a full segment; several areas may also be involved. Consolidation that involves a full segment or lobe is detected in the majority of bacteremic cases, and is more common when pneumococcal pneumonia occurs in otherwise healthy young adults. There is diminished respiratory excursion (splinting) on the affected side because of pain. Dullness to percussion is present in half of all cases. Crackling sounds are heard on careful auscultation in nearly all cases. WBC is elevated. In cases where there is no fever present and there is a decrease in WBC and the patient is a young adult, the pneumonia can progress rapidly with grave results.

A 24-year-old man presents with a 1-week history of shortness of breath and a nonproductive cough. On physical exam, he is tachycardic, tachypneic, and febrile. He has lost weight without a change in dietary habits. Auscultation of this chest reveals bibasilar crackles. A chest X-ray is ordered and demonstrates diffuse interstitial infiltrates. You collect an arterial blood gas, and the results show moderate hypoxemia. A metabolic panel is ordered, and the only abnormality is an isolated elevated lactate dehydrogenase (LDH) enzyme. Question What is the most likely diagnosis? Answer Choices 1 Bowen's disease 2 Streptococcal pneumoniae 3 Mycoplasma pneumoniae 4 Stevens-Johnson syndrome 5 Pneumocystis jiroveci

The clinical picture is suggestive of Pneumocystis jiroveci pneumonia. Signs and symptoms include fever, dyspnea, and a nonproductive cough. Chest examination reveals bibasilar crackles on auscultation, and X-rays reveal diffuse interstitial infiltrates. Isolated elevated LDH is often a finding on serum chemistries. The infectious agent is Pneumocystis jiroveci. This type of pneumonia is common in AIDS patients. Bowen's disease is a form of early squamous cell carcinoma. Lesions can appear as solitary or multiple, and they are pink or red in color with a slightly scaling surface, small erosions, and possible crusting. Streptococcal pneumoniae and/or Mycoplasma pneumoniae pneumonia are community-acquired pneumonias that present similarly to Pneumocystis jiroveci pneumonia, except bibasilar crackles are not present and isolated LDH is not a finding. Steven-Johnson syndrome is a mucocutaneous drug-induced or idiopathic reaction of the skin that is characterized by skin tenderness and erythema of skin and mucosa, followed by extensive cutaneous and mucosal epidermal necrosis and sloughing.

A 60-year-old man with a past medical history of hypertension, hyperlipidemia, and prostate cancer is being evaluated in the emergency room. He has complained of new-onset acute severe left-sided pleuritic chest pain over the course of the last 2 hours. The pain is associated with feelings of anxiety, hemoptysis, shortness of breath, and nausea. He "feels warm" but denies chills, palpitations, wheezing, cough, edema, vomiting, abdominal pain, abnormal bowel habits, or dietary intolerances. He endorses a 40 pack-year smoking history, but denies drug or alcohol use. Upon physical exam, he is found to be febrile, hypotensive, diaphoretic, and in acute painful distress. There is perioral cyanosis and a pleural friction rub to the left lung fields. There is no peripheral edema, and the remainder of the exam is normal. A bedside electrocardiogram revealed the attached image. S1Q3T3 Question What is the most appropriate initial intervention for this patient? Answer Choices 1 Surgical embolectomy 2 Oral warfarin 3 Unfractionated heparin 4 Recombinant tissue plasminogen activator 5 Inferior vena caval filter placement

Unfractioned Heparin Unfractionated heparin is the standard initial treatment for isolated, unconfirmed PE. The FDA has not approved any low molecular weight heparin (LMWH) for primary symptomatic PE without associated DVT. Patients at high risk for an adverse outcome with anticoagulation alone should be considered for embolectomy if they have contraindications to thrombolytic therapy. Clinical success has been reported, but randomized trials are lacking. This vitamin K antagonist prevents carboxylation activation of coagulation factors II, VII, IX, and X. The full effect of warfarin requires at least 5 days, even if the prothrombin time, used for monitoring, becomes elevated more rapidly. If warfarin is initiated as monotherapy during an acute thrombotic illness, a paradoxical exacerbation of hypercoagulability can increase the likelihood of thrombosis rather than prevent it. Pulmonary embolism thrombolysis with Tissue plasminogen activator (tPA) remains controversial because large clinical trials using survival as an end point have not been conducted. Tissue plasminogen activator (tPA) may be considered in radiographically-confirmed pulmonary embolism. There are 2 indications for placing an inferior vena caval filter for patients with acute PE: (1) concomitant major bleeding, requiring transfusion or any intracranial hemorrhage that precludes the use of anticoagulant therapy, or (2) recurrent PE despite prolonged intensive anticoagulation.

A 4-year-old boy presents with a 3-day history of fever with chills, cough, and fast breathing. His parents report decreased oral intake and increased difficulty breathing with retractions over the last 24 hours. His initial vital signs are: heart rate 144 bpm; respiratory rate 32/min; temperature 101.3 degrees Fahrenheit (38.5 degree Celsius); and oxyhemoglobin saturation 89% on room air. He is immediately started on supplemental oxygen, and his oxyhemoglobin saturation improves to 95%. Subsequent evaluation, including a chest X-ray, is suggestive of right middle and lower lobe pneumonia. Question What is the mechanism for the low oxyhemoglobin saturation in this patient? Answer Choices 1 Hypoventilation 2 Ventilation/Perfusion mismatch 3 Diffusion Block 4 Shunt 5 Increased dead space in lungs

Ventilation/Perfusion mismatch Of all the causes of hypoxemia in children, ventilation perfusion mismatch is the most common cause in most clinical situations. In a child with pneumonia, the middle and lower lobes of the right lung will continue to receive pulmonary blood flow, but the exchange of gases will be impaired due to poor air entry in those parts of the lung. This generates a mismatch between the ventilation (which is reduced) and perfusion (which is maintained), causing most of the blood from those areas to return to the left atrium in a relatively deoxygenated state. This leads to an overall drop in the percentage of hemoglobin molecules that are saturated with oxygen, thereby manifesting as a drop in the measured oxyhemoglobin saturation. Hypoventilation occurs in patients with neuromuscular weakness and can cause significant elevation of carbon dioxide levels before it causes hypoxemia. Diffusion block is associated with lung disorders, such as idiopathic pulmonary fibrosis and interstitial lung diseases, which increase the thickness of the alveolar basement membrane, thereby reducing the exchange of gases. Shunt causes hypoxemia due to the mixing of deoxygenated blood with oxygenated blood when the former has bypassed the lung completely (such as right to left shunt in a patient with Tetralogy of Fallot). A patient with pneumonia will not have an increase of dead space in the lungs.

A 12-year-old girl, diagnosed using a quick antigen test as having a pharyngeal infection due to Streptococcus pyogenes, also presents with a rash that is on the upper part of the chest and trunk. This rash is caused by Streptococcus pyogenes producing what? Answer Choices 1 Exfoliative toxin 2 Elastase 3 Enterotoxin 4 Erythrogenic toxin 5 Streptolysin

erythrogenic toxin Erythrogenic toxin (streptococcal pyrogenic exotoxin) is produced by Streptococcus pyogenes. The toxin is responsible for the rash of "scarlet fever". The toxin has been shown to exhibit pyrogenicity and cytotoxicity. It usually appears at the 2nd day of infection, on the upper part of the chest, spreading to the rest of the trunk out towards the rest of the body, with the palms, soles, and face being spared. Exfoliative toxin, produced by Staphylococcus aureus, is responsible for scalded skin syndrome (SSS), wherein the patient appears to have acquired a burn of the skin. There is extensive scalding and flaking desquamation of the epidermis. The syndrome is especially common in infants and small children. Elastase is an extracellular protease that is produced by Pseudomonas aeruginosa. The protease is associated with the organism's virulence due to tissue destruction and bacterial invasion. Elastase is necrotizing to the skin, cornea, and lung; it is capable of producing hemorrhage. Enterotoxin is produced by various bacteria. In Clostridium perfringens, the production of this toxin causes the symptoms of food poisoning. Clostridium perfringens type A is most associated with this toxin production. Enterotoxin is thought to act as a superantigen, which causes a massive release of inflammatory mediators and induces a calcium ion-dependent breakdown of permeability. Exotoxin (diphtheria toxin) is produced by Corynebacterium diphtheriae. The toxin inhibits protein synthesis in mammalian cells, but not bacteria. It affects all cells in the body; the heart, nerves, and kidney are impacted the most.

A 9-year-old girl developed erythema of the cheeks; subsequently, an erythematous maculopapular rash started spreading from her arms to her trunk and legs. She complains of itching. Her medical history is non-contributory, and the results of physical examination are otherwise unremarkable. What is the appropriate therapy? Answer Choices 1 Tetracycline 2 Vidarabine or acyclovir 3 Orally administered penicillin 4 Chloramphenicol or tetracycline 5 Supportive antipruritics

supportive antipruritics The pattern of rash progress indicates erythema infectiosum, a mild, self-limited systemic disease which is caused by a human parvovirus. The usual therapy consists only of supportive measures, such as antipruritics, to relieve itching. Antibiotics are irrelevant, and the mild, self-limited nature of the disease makes antiviral therapy unnecessary in immunocompetent patients.

A 67-year-old man presents with a 3-week history of increasing shortness of breath; it occurs even while he is at rest. The patient was diagnosed with congestive heart failure in the past year, and he has been well controlled on oral medication. He has no history of tobacco use. He has gained 10 pounds since his last exam 2 months prior to presentation. On physical exam, there are diminished breath sounds and decreased tactile fremitus bilaterally at the base of the lungs. Dullness to percussion is also noted in the same area. He has 3+ bilateral pitting lower extremity edema. Question Based on the patient's physical exam and history, what is the most likely diagnosis? Answer Choices 1 Lung malignancy 2 Tuberculosis 3 Empyema 4 Spontaneous pneumothorax 5 Pleural effusion

the correct answer is pleural effusion. This patient has a history of congestive heart failure, which is one of the most common causes of transudative pleural effusions. Pleural effusions in the setting of heart failure are usually right-sided or bilateral; in this patient's case, physical exam findings point to bilateral pleural effusion. Dyspnea is a common symptom seen in patients with a pleural effusion. Based on his pitting edema on exam, it is clear that this patient has uncontrolled heart failure. The physical exam findings are also common with pleural effusion.

A 55-year-old man is found on routine X-ray to have a growing lesion in the right middle lung field. He is a heavy smoker. The physical examination is within normal limits. Tuberculin test is negative. What is the management of choice? Answer Choices 1 Thoracotomy 2 Antibiotics 3 Reassurance 4 Observation 5 Chemotherapy

thoracotomy Because of the high percentage of carcinoma in heavy smokers, the excision of a growing lesion is indicated. The small mass in the lung is often discovered on X-ray examination and without any pulmonary symptoms. The patient's history of smoking suggests that the carcinoma is most likely a presenting complaint; as a result, lobectomy or excision of the growing lesions is indicated.

You transfuse 3 units of whole blood to a gun shot victim. Unknown to you, there was a shortage of blood, and the units you gave to your patient were from the bottom of the freezer and about to expire. Within 2 days, the transfused cells are breaking down, and your patient's ECG is beginning to show high T waves and an increased PR interval. You recognize that he is becoming hyperkalemic and that the excess potassium is affecting the electrical conduction system of the heart. What best describes the resting membrane potential of a sinus nodal fiber under normal conditions? Answer Choices 1 -124 mV 2 -91 mV 3 -85 mV 4 -55 mV 5 -25 mV

-55 mV Normal resting potential for most excitable cells is -70 mV. In order for cells of the SA node to be even more negative (-124 mV), they would either have a greater intracellular concentration of potassium or equally less potassium outside the cell. Increasing the intracellular concentration by that amount would result in severe osmolarity problems. -91 mV is the equilibrium potential for potassium. This value excludes the influence of potassium and other ions on resting membrane potential. To achieve a resting membrane potential of -85 mV, the cells would either have a greater intracellular concentration of potassium or equally less potassium outside the cell. Increasing the intracellular concentration by that amount would result in severe osmolarity problems. This significant decrease in the negativity of the membrane potential is caused by a decrease in permeability of the membrane of these cells to potassium. Therefore, sodium ions have a greater influence on resting membrane potential, bringing it closer to sodium's equilibrium potential (61 mV). If resting membrane potential of the SA nodes reaches -25 mV, the cells would depolarize, but not repolarize. Threshold for these cells is -40 mV. The membrane must return below threshold in order to be able to be depolarized again.

A 45-year-old woman presents with worsening symptoms. She was diagnosed with idiopathic pulmonary hypertension about 2 years ago, and she is on home oxygen therapy. She has longstanding fatigue and dyspnea; she is now experiencing worsening dyspnea with exertion, some swelling in her ankles, and the inability to breathe well when lying down. Although she has always been thin, she notes her weight has increased by 10 pounds in the last month. She denies fever and chills. The patient's vitals are shown in the table. Weight- 152 lbs Height- 66" Pulse- 95 Blood pressure- 132/86 Temperature- 98.4 °F/36.9°C Pulse oximetry- 91% On physical exam, the patient is in no apparent distress; she is using her portable oxygen by nasal cannula. Her ankles have visible edema, and her neck veins are mildly distended. Lung sounds are diminished bilaterally, but no rales or crackles are heard. A gallop is auscultated on her cardiac exam. Question What is the next most appropriate intervention for this patient? Answer Choices 1 Call an ambulance for immediate transport to emergency department 2 Initiate diuretics and refer to a pulmonologist this week 3 Initiate a macrolide antibiotic and cough syrup; follow-up in 2 days 4 Prescribe an albuterol inhaler for use as needed when shortness of breath occurs 5 Send the patient to the hospital for immediate admission

2 Initiate diuretics and refer to a pulmonologist this week This patient with pulmonary hypertension (which may be progressing into cor pulmonale) needs specialist care. It would be most appropriate to initiate diuretics and refer her to a pulmonologist this week. Treatment medications for pulmonary hypertension are complex; they can include endothelin receptor antagonists, phosphodiesterase inhibitors, intravenous prostacyclins, and others. This patient has presented to the outpatient clinic and is stable. If left untreated, her condition may deteriorate, but her current state is not an acute emergency requiring an ambulance for immediate transport to the emergency department. If the patient had pneumonia (you would expect a history of cough, fever, chills, and physical exam findings suggesting infection), it would be appropriate to initiate a macrolide antibiotic and cough syrup as well as follow-up in 2 days. However, this patient's pulmonary condition is not secondary to infection. If the patient's dyspnea was caused by asthma, it would be reasonable to prescribe an albuterol inhaler for use as needed when shortness of breath occurs. However, albuterol (a short-acting beta-agonist) will not address her pulmonary hypertension or the cor pulmonale. As with the discussion above regarding emergency transport, this patient does not present with an urgent or emergent condition that requires inpatient management. If there were indicators that the patient would not be compliant with outpatient care, inpatient admission could be considered. However, this patient describes prior specialist care, and it can be presumed she has been compliant with appointments, testing, and her home oxygen.

A 69-year-old man presents with a 2-hour history of right upper extremity hemiparesis; the hemiparesis resolved upon presentation to the hospital. A cerebral MRI demonstrates an isolated intracranial lesion, which is consistent with metastatic disease. A chest X-ray demonstrates a right hilar mass with right lower lung atelectasis. A CT is shown below that demonstrates right lower and middle lobe atelectasis secondary to an endobronchial cancer without lymphadenopathy. PET scan demonstrates uptake in the right lung and brain alone. Bronchoscopy demonstrates complete occlusion of the right lower lobe secondary to adenocarcinoma. The tumor encroaches upon the right middle lobe but does not occlude the orifice. The patient undergoes resection of the cranial metastasis uneventfully. Question What should the next treatment include? Answer Choices 1 Radiation therapy alone 2 Lung resection including the middle and lower lobe 3 Lung resection including a pneumonectomy 4 Lung resection of the lower lobe only 5 Photodynamic therapy alone

2 Lung resection including the middle and lower lobe Occasionally, patients present with resectable localized lung cancer and evidence of solitary metastasis. These patients should be considered for combined approach in which the primary tumor and solitary metastasis are resected. Locations that are amenable to a combined approach include adrenal, brain, lung, and bone. Most brain metastases are histologically adenocarcinoma in this case. Cranial resection is done first and thoracotomy thereafter, unless symptoms from the primary tumor site dominate the clinical picture. The best surgical treatment for this particular lung cancer is lower and middle lobectomy. Lower lobectomy alone would leave a cancerous middle lobe and offer no survival advantage. Pneumonectomy would be an unnecessary extensive resection in view of the bronchoscope findings. Also, a pneumonectomy would have a higher mortality rate without the benefit of a higher cure rate. For patients who are unable or unwilling to undergo surgery, radiation can be used with curative intent. The long-term survival rate for patients with early stage resectable disease is from 17%-33%. Results of curative photodynamic therapy are usually for microinvasive carcinoma, defined as lesions less than 10 mm. In 1997, a report documented 9 of 21 patients had no evidence of disease at a mean of 68 months. Currently, there is a limited role for photodynamic therapy with primary respectable lung cancer due to higher survival rates with surgical removal.

A 70-year-old man presents with difficulty breathing with a persistent cough and hemoptysis. He had a left radical nephrectomy 8 years ago for clear cell carcinoma of the kidney. See the attached image. The chest X-ray shows several large peripheral and hilar masses along with an interstitial infiltrate, which is characteristic of bronchoalveolar spread of cancer. A noninvasive chest CAT scan is useful to delineate the actual presence and extent of the tumor in the chest and mediastinum. If it appears that there is proximal bronchial involvement by cancer, then bronchoscopy and biopsy would be helpful in confirming the diagnosis. If the bronchoscopy is non-diagnostic, what is the next diagnostic test? Answer Choices 1 Thoracotomy 2 Transthoracic CAT scan guided needle biopsy 3 None. Treat the patient based upon the X-ray findings 4 Positron emission tomography 5 Mediastinoscopy

2 Transthoracic CAT scan guided needle biopsy Generally, in patients with clinical suspicion of recurrent or metastatic cancer, histologic confirmation is obtained in the least invasive manner possible. CAT scan-guided needle biopsy is a common method of obtaining tissue diagnoses without a surgical procedure.

A 55-year-old man presents with a 4-hour history of severe left-sided chest pain. His wife tells you that they went out to dinner earlier in the evening, but the patient felt ill shortly after returning home. He had several episodes of vomiting, followed by severe pain. On exam he appears ill; he is cool and diaphoretic. Blood pressure is 90/60, heart rate is 126, respiratory rate is 32, and temperature is 39.8o C. There is no JVD. Cardiac exam is tachycardic, but there is no murmur. Lung sounds are decreased in the left base, with dullness to percussion. The abdominal exam reveals tenderness, but there is no guarding or rebound. Laboratory studies reveal a white blood cell count of 14,000, a hemoglobin of 14 gm/dL, and a hematocrit of 44 gm/dL. EKG reveals a sinus tachycardia without any Q waves. Chest X-ray reveals a left pleural effusion. The described case scenario represents a patient with Boerhaave's syndrome, which is a spontaneous esophageal rupture after forceful vomiting. Once the diagnosis is confirmed, what is the optimal definitive treatment? Answer Choices 1 Intravenous antibiotics 2 Tube thoracostomy 3 Surgical exploration 4 Observation 5 Pulmonary function tests

3 Surgical exploration A gastrografin swallow will usually confirm the diagnosis and delineate the site of rupture. If this is unsuccessful, a barium swallow should follow. It is important to delineate the site of rupture since this will impact the surgical approach. Chest X-ray findings that are more suggestive of the diagnosis include hydropneumothorax, pneumomediastinum, and mediastinal widening. The most definitive treatment is surgical exploration, identification, and repair of the perforation. Since the time frame is relatively short in this case, suture repair with an omental or pleural flap should suffice. Wide chest tube drainage is necessary to prevent postoperative sequelae. Antibiotic therapy should be also initiated, but will never be the definitive treatment alone. The mortality rate from this condition is directly related to the time delay. In patients where the perforation is repaired within 24 hours, the mortality rate is 10 - 15%; the mortality rate is over 50% in patients where the therapy is delayed beyond 24 hours.

A 77-year-old female has been treated for the last two weeks for a community-acquired pneumonia. While on oral azithromycin, the patient continues to develop fevers, some as high as 103 0 F. Her oral intake has decreased, and her effort to breathe continues to be labored. On examination, the patient continues to have rhonchi and some mild rales that are best appreciated in the anterior right lung region. A follow-up chest x-ray reveals a consolidated infiltrate of the right middle lobe. A CT of the chest reveals a loculated, fluid-filled area of the right middle lobe with no evidence of a foreign body. Based on these new findings, what is the most likely pathogen causing this ongoing infection? A Staphylococcal aureus B Pseudomonas aeurginosa C Hemophilus influenza D Klebsiella pneumoniae E Chlamydia pneumoniae

A Staph Aureus This patient is having a history and physical exam that is consistent with an empyema. This loculated collection of fluid will harbor bacteria, the most common pathogen being Staphylococcus aureus. Intravenous antibiotics as well as surgical drainage are warranted.

A patient who resides in northern Arizona presents with signs and symptoms that are consistent with pneumonia. He is afebrile and appears non-toxic. His physical exam is unremarkable, and blood work is within normal limits. A chest x-ray is ordered and reveals bilateral upper lobe infiltrates. Based on this information, what is the best medication treatment for this patient? A Itraconazole B Erythromycin C Oseltamivir D Doxycycline E Amoxicillin

A itraconazole Patient is presenting with clinical signs and symptoms of fungal pneumonia (coccidiomycosis). The presentation of being afebrile, along with upper lobe infiltrates bilaterally in a region of the country that consistently has more fungal pneumonia's would lead the clinician with the reasoning that a fungal process is present. The azole's are the best treatment for fungal pathogens.

A 28-year-old man with a history of crack cocaine abuse was rushed in to the emergency room. His mother found him in his apartment. He was blue and severely short of breath. No other history is available. His examination revealed a young man in severe respiratory distress with temperature 99.2° F, pulse 102/min, respiration 40/min, BP165/95 mm Hg, and a pulse oximetry of 66%. He was intubated at the site by the EMS personnel because of lack of improvement on 100% non-rebreathing mask. His chest X-ray shows bilateral alveolar and interstitial infiltrates. The arterial blood gas on the mechanical ventilation is set at a rate of 12 cycles/min, tidal volume 500 mL, and PAO2/Fi02 ratio = 100 mmHg; PH 7.52, PCO2 30, PO2 55, and O2 saturation 88%. What is the most likely diagnosis? Answer Choices 1 Acute respiratory distress syndrome 2 Bacterial pneumonia 3 Myocardial infarction 4 Pulmonary embolism 5 Viral pneumonia

Acute respiratory distress syndrome Acute respiratory distress syndrome (ARDS) is an acute lung injury in which the ratio of PaO2/FIO2 is 200 or less and there is bilateral interstitial infiltrates with no evidence of raised left atrial pressure. Common etiologies are aspiration, sepsis, pancreatitis, toxic inhalation burns, multiple blood transfusions, drugs, and trauma. It is characterized by dyspnea, tachypnea, rales, bilateral infiltrates, marked hypoxemia that is not corrected by O2, and leukocytosis. Multi-organ failure may also be present. The patient in question has all of these features, and his PaO2/FIO2 ratio is 100. Treatment of ARDS includes identifying and treating the underlying condition as well as cardiopulmonary support. No pharmacological agents have been shown to be beneficial. Adding positive end-expiratory pressuremay improve oxygenation by opening the fluid-filled alveolar units (alveolar recruitment). Other modalities that may improve oxygenation include prone positioning and inverse ratio ventilation. Bacterial and viral pneumonia presents as cough with or without sputum production, fever, and leukocytosis. It does not present with this degree of shunt hypoxemia. It also does not lead to such rapid deterioration. Myocardial infarction may be precipitated by cocaine use. For it to have caused such infiltrates, however, there would have also been signs of right heart failure and hypotension; it would not have caused so much shunt hypoxemia. Pulmonary embolism can present acutely. It has to be massive to present this way, in which case the patient would be hypotensive, and CXR would not show bilateral infiltrate. Increasing the tidal volume or respiratory rate will not improve his oxygenation since he has very high minute ventilation. An emergent thoracentesis will not improve the patient's status.

A 72-year-old man presents due to worsening shortness of breath, orthopnea, and chest pain; symptoms have been occurring for the past few weeks. The patient admits to some chronic heart problems as well as fatigue, dyspnea, and a non-productive cough; however, he feels like symptoms have worsened recently. He denies fever, chills, and a productive cough. On physical exam, the man has mildly increased respiratory effort, but he does not appear in distress. He is barrel-chested. His breath sounds are diminished bilaterally, with dullness to percussion over the right and left lower lungs. No pleural friction rub is noted. On cardiovascular exam, an S3 gallop and mild tachycardia (110 bpm) are noted. Clubbing of the fingers, dependent edema in the lower extremities, and jugular venous distention are also noted. His cardiac enzymes and electrocardiogram demonstrate no acute cardiac pathology. Pleural fluid and cardiomegaly are found on the chest X-ray, and a thoracentesis is performed. The pleural fluid is generally clear in color, testing negative for chylomicrons and triglycerides. It has low levels of red blood cells, white blood cells, protein, and lactate dehydrogenase (LDH). Question What is the likely underlying mechanism for the pleural effusion in this patient? Answer Choices 1 Chylothorax from disruption of the thoracic duct 2 Empyema from infection in the pleural space 3 Exudates from local inflammation in capillary beds 4 Hemothorax 5 Transudates from increased hydrostatic pressure or decreased oncotic pressure

All of the listed choices are possible mechanisms for pleural effusions. According to this patient's history and physical exam, as well as the characteristics of the pleural fluid, it should be apparent that his pleural effusion stems from transudates from increased hydrostatic pressure or decreased oncotic pressure. He presents with typical chronic congestive heart failure symptoms. The pleural effusion occurs not from diseased pleura, but from the underlying illness. Transudative pleural effusions are the most common type, and heart failure is the most common underlying cause of pleural effusions. Chylothorax from disruption of the thoracic duct is relatively uncommon. The primary distinguishing characteristics are seen in the milky appearance of pleural fluid, with a large amount of chylomicrons and triglycerides. Empyema from infection in the pleural space might have been a cause if this patient had pneumonia or other severe pulmonary infection. This patient's clinical history is not suggestive of infection. The pleural fluid of empyema may be grossly purulent or milky white. If the milky white fluid is centrifuged, showing clear supernatant with a pellet of white cells below, empyema should be considered. Exudates from local inflammation in capillary beds is a mechanism for pleural effusions secondary to pulmonary embolism, malignancy, as well as some infections, such as pneumonia. Exudates can be distinguished from transudates by a combination of history, appearance, and characteristics of the pleural fluid (using Light's criteria). Simplified, the pleural fluid with exudates may be cloudy in appearance, with relatively high levels of protein and LDH. Hemothorax is blood in the pleural space. It can occur with trauma to the chest wall or other structures of the thorax. Typically, there would be a history of trauma. Hemothorax is unlikely to be bilateral, except in cases of significant trauma. Additionally, the pleural fluid may appear grossly bloody and demonstrate high numbers of red blood cells on analysis.

A 62-year-old woman with a long-standing history of hypertension presents with severe headache; it started this morning and is rapidly worsening. During the interview, she suddenly collapses. Your brief examination shows that she responds with extensor posturing on external stimuli. Her deep tendon reflexes are 3, and you elicit Babinski bilaterally. You also notice that her breathing has a peculiar pattern: deep inspiration with a pause at full inspiration, followed by a brief insufficient release and the end-inspiration pause. Question How do you best describe her respiratory pattern? Answer Choices 1 Cheyne-Stokes 2 Apneusis 3 Ataxic 4 Cluster 5 Central neurogenic hyperventilation

Apneusis In the hypertensive patient with sudden loss of consciousness and decerebrate response (extensor posturing), you should consider brain stem hemorrhage. Abnormal breathing patterns can be observed in both pontine and medullary lesions; they sometimes can be prognostic. Her breathing pattern is apneustic. Apneustic breathing pattern characterizes deep, gasping inspiration with a pause at full inspiration followed by a brief, insufficient release and the end-inspiration pause before expiration. Lesion in the pons or upper medulla causes the removal of input from the vagus nerve and the pneumotaxic center. Normally, apneustic center of the lower pons promotes inspiration by stimulation of the dorsal respiratory center in the medulla to delay the 'switch off' signal of the inspiratory ramp provided by the pneumotaxic center of pons. Therefore, athe pneustic center controls the intensity of breathing. Apneusis is an ominous sign, with a generally poor prognosis. Cheyne-Stokes respiration is an abnormal pattern of breathing characterized by repetitive progressively deeper and sometimes faster breathing, followed by a gradual decrease in breathing that results in temporary apnea. Cycles usually take 30 seconds to 2 minutes. Increased CO2 during the period of apnea causes compensatory hyperventilation. Hyperventilation in turn causes the decrease in CO2, which causes apnea and the cycle to restart. Causes include CNS dysfunction, cardiac failure with low cardiac output, sleep, hypoxia, or profound hypocapnia. Ataxic breathing (Biot's respiration) is an abnormal pattern of breathing characterized by groups of quick, shallow inspirations followed by regular or irregular periods of apnea. It is generally a poor prognostic sign. Biot's respiration is caused by the medullary lesion due to strokes or trauma or by pressure on the medulla due to uncal or tentorial herniation. Kussmaul breathing is a deep, labored, and gasping breathing pattern seen often in severe metabolic acidosis (diabetic ketoacidosis, renal failure). In metabolic acidosis, breathing is first rapid and shallow, but later on, as acidosis worsens, breathing gradually becomes Kussmaul breathing. Central neurogenic hyperventilation is a very deep and rapid pattern of breathing, usually seen in the lesions of the midbrain and upper pons. Respirations are generally regular, and the PACO2 decrease due to the hyperventilation. Cluster breathing is irregular breathing with periods of apnea that occurs at irregular intervals. It is generally seen in lesions in the low pons or upper medulla. It differs from Cheyne-Stokes pattern because there is no increasing and decreasing depth of respirations.

During a routine X-ray examination for employment insurance purposes, the radiologist notices a lesion on the right upper lobe of the pulmonary X-ray of a middle-aged man. The patient was treated for pulmonary cavitary tuberculosis (TB) 2 years ago; he has completed treatment, and he has not had any problems since. Refer to the image. What late complication of TB is seen in this patient? Answer Choices 1 Fibrothorax 2 Aspergilloma 3 Broncholithiasis 4 Reactivation of TB 5 Bronchiectasis

Aspergilloma Aspergilloma is a solid mycelial mass caused by the growth of Aspergillus species within a cavity, usually within the lung. This condition is most commonly a result of a previous tuberculous infection. Although the patient may be asymptomatic, cough is frequent, and a severe life-threatening hemoptysis may occur. Patients present with hemoptysis, or it can be an incidental diagnosis with a routine X-ray. Previous TB history and anti-TB treatment are helpful for the diagnosis. The diagnosis is confirmed by the demonstration of a mobile intracavitary mass on the radiograph and air-crescent sign around the mass. Although cultures are often negative, patients frequently have high titers of antibody to Aspergillus. If the patient is asymptomatic, the treatment is conservative. Surgical removal is the only reliable curative method indicated for uncontrollable hemoptysis. Spontaneous expectoration of the fungal mass may occur. Oral itraconazole results in partial or complete resolution of aspergillomas in 60% of patients. In this question, the radiologic picture has a typical Easter egg shaped aspergilloma in the right upper lobe old TB cavity. Lung destruction can be seen after recurrent TB, usually due to noncompliance. Patient's asymptomatic status and chest X-ray is suggestive of the late TB complication aspergilloma, not broncholithiasis, fibrothorax, reactivation of TB, or bronchiectasis.

A 40-year-old man who is a chronic alcoholic presents with cough productive of large amounts of purulent sputum. He developed the cough about 2 weeks ago, and it has gradually worsened over time. Now his sputum is foul-smelling. He has had high intermittent pyrexia for 4 days. The vitals are as follows: temperature of 38.9°C, pulse of 108BPM, respirations of 24/min, blood pressure of 140/80 mm Hg. On auscultation, there is pleural rub and diminished air entry on the right side. The chest radiograph shows a large dense opacity on the right side with a fluid level. The patient is diagnosed with lung abscess. Question What statement is true regarding lung abscess? Answer Choices 1 Only anaerobic organisms are responsible for this condition 2 Aspiration is the most common cause 3 Sputum culture is the best investigation 4 Computed Tomography (CT) has no role as an investigation in this condition 5 Treatment is with antibiotics for 3 days

Aspiration is the most common cause Explanation Lung abscesses are the result of microbial infection causing cavities containing necrotic pulmonary tissue to develop in the lung. Aspiration of oropharyngeal material is the most common cause. It is always secondary, due to infection reaching the lungs from elsewhere. About 65% of cases are caused by anaerobic bacteria and the rest by a mixture of both aerobic and anaerobic organisms. Symptoms include cough, foul-smelling sputum, fever, and sometimes hemoptysis. A chest X-ray can demonstrate a cavity with fluid in it. But other associated conditions, such as empyema or pulmonary infarction, are best demonstrated on CT scan. Knowledge of the underlying cause of the abscess can assist in deciding on the right antibiotic course. A sputum culture, although routinely done, is usually a contaminated specimen (by the organisms in the mouth) and hence not very useful. Although improvement is expected in 3-5 days, therapy is generally continued for 4-6 weeks, depending on further radiograph findings.

A 5-year-old boy presents with an erythematous skin rash. It is associated with intense itching. The boy's mother has noticed that her son's rash has been recurrent, with 3 to 4 episodes per year. The itching and rash increases after consumption of certain foods. On examination, erythematous raised papules are seen on the cheek, trunks, and upper arms; an example is shown. Question What is most likely associated with the boy's condition? Answer Choices 1 Gluten enteropathy 2 Asthma 3 Vitamin A deficiency 4 Psoriasis 5 Raised Dengue titers

Asthma Asthma is the correct answer. The vignette describes a child with atopic dermatitis, one of the most common childhood dermatological conditions. It usually begins before 2 years of age and displays a chronic relapsing course. Typical sites of involvements are the cheeks, flexural aspect of the elbows, natal cleft, and the hands. Clinical examination usually reveals xerosis, lichenification, and eczematous lesions. Atopic dermatitis is commonly associated with other atopic conditions, such as hay fever, extrinsic asthma, allergic conjunctivitis, and allergic rhinitis. A high IgE level and eosinophilia are common to all of these. Gluten enteropathy is incorrect. Gluten enteropathy or celiac disease may be associated with skin manifestations, namely dermatitis herpetiformis. The typical patient is middle aged and develops crops of fluid filled blisters resembling herpetic lesions (hence the name herpetiformis). The face, hairline, and shoulder are common sites. IgE levels are usually normal. The association with celiac disease is strong; associated symptoms of abdominal pain, nausea and bloating may be contributory findings. Vitamin A deficiency is incorrect. Vitamin A deficiency may result in dry, thickened 'toad skin'; it may also be associated with frequent skin infections. Eye manifestations, such as night blindness, corneal ulcers, and xerophthalmia, are more common. Psoriasis is incorrect. Psoriasis usually causes discrete scaly lesions and is more common in older age groups, except for Guttate psoriasis. It shows a predilection for extensor surfaces and the scalp, as opposed to atopic dermatitis, which favors the flexural aspects. Raised Dengue titers is incorrect. Dengue fever is an arboviral infection that may cause a fine maculopapular rash throughout the body. Dengue hemorrhagic fever may be associated with a low platelet count and petechial rashes. They are never associated with chronic discrete rashes.

A 6-year-old male child presents to the clinic for a cough that occurs only after he has been running, according to his mother. She says she first noticed this about 6 months ago, after he had had one of his usual winter colds, and his cough persisted for about a week. On the basis of this history, what is the most likely diagnosis? A airway foreign body B asthma C cystic fibrosis D laryngomalacia

Asthma Asthma, in this case exercise-induced, is the most likely cause of this problem. The symptoms commonly associated with acute exacerbations of asthma include wheezing, cough, dyspnea, and chest pain. Some symptoms that might be suggestive of asthma include exercise-induced cough, nighttime cough, cough after cold air exposure, and cough after laughing. Airway foreign bodies, though not common, are an acute problem that may present as sudden cough, choking, and wheezing. Cystic fibrosis (CF) is the most common, lethal, genetic disease affecting the Caucasian population. Up to 50% of patients with CF are diagnosed in infancy, but others may not be diagnosed until adolescence or adulthood. Chronic or recurrent cough should be an indicator for consideration of CF as a differential diagnosis. Laryngomalacia is the most common cause of stridor in infants. It is the incomplete development of the cartilaginous support of the laryngoglottic structures. This congenital condition is usually self-limiting and occurs most commonly in infants at or just after birth. The inspiratory collapse of the epiglottis or arytenoid cartilages is heard as stridor.

Following emergent appendectomy, a 58-year-old obese male develops a temperature of 102.4˚F, 18 hours postoperatively. His respiratory rate is 26 and his pulse is 116bpm. A physical exam reveals scattered fine rales. What is the most likely diagnosis? A Atelectasis B Aspiration pneumonitis C Pleural effusion D Pneumonia E Pulmonary embolus

Atelectasis A Pulmonary alveoli collapse, also known as atelectasis, occurs during operative procedures for a variety of reasons, including decreased clearance of secretions and decreased intra-alveolar pressure. Postoperatively, often due to pain, patients may not mobilize secretions appropriately, also contributing to atelectasis. Atelectasis is the most common postoperative pulmonary complication, and is often associated with emergent and prolonged surgeries, especially those of the thorax and abdomen. Atelectasis is associated with fever, an increased respiratory rate, an increased pulse, and lung exam findings ranging from normal to rales and decreased breath sounds. Symptoms usually present within the first 48 hours postoperatively. Pulmonary aspiration pneumonitis, although possible, is less likely due to appropriate preoperative and intraoperative measures being utilized to decrease risk. Pneumonia is also a common postoperative complication, due to the same contributing factors as atelectasis. Physical exam findings may also be similar. However, postoperative pneumonia is likely to become evident between 24 and 96 hours postoperatively. A postoperative pleural effusion may form, due to free peritoneal fluid as well as a complication of atelectasis, but has a lower incidence of occurrence than atelectasis alone. Patient symptoms will be based upon the size of the effusion, associated inflammation, and whether or not the effusion is infectious. Consideration must be given to pulmonary emboli for any post-surgical patient with tachypnea, tachycardia, and dyspnea. Pulmonary emboli may occur at any point postoperatively, but atelectasis remains a more common cause of postoperative fever and respiratory changes.

A 56 year-old woman develops a fever of 102.1 F on her first post-operative surgery day following cholecystectomy. Which of the following is the most likely diagnosis? A Atelectasis B Catheter-related phlebitis C Drug reaction D Urinary tract infection E Wound infection

Atelectasis Post-operative fever that occurs in the first 24 hours is most likely caused by atelectasis (A). Catheter-related phlebitis (B), drug reactions (C) and urinary tract infection (D) typically present between post-operative days 2 and 4, while wound infection (E) typically presents on or after post-operative day 5.

A couple presents to the office seeking genetic counseling advice regarding their child's potential risk of being born with Cystic Fibrosis. Both parents are identified as carriers for the cystic fibrosis gene mutation. Which of the following genetic inheritance patterns should be used to predict the probability of their child being affected by Cystic Fibrosis? A Autosomal dominant B Autosomal recessive C Mitochondrial inherited D X-linked dominant E X-linked recessive

Autosomal recessive Cystic fibrosis has an autosomal recessive (B) genetic inheritance pattern. In this case their children have a 25% chance of being affected, 50% chance of being a carrier, and 25% chance of lacking inheritance of the genetic trait.

A 35-year-old woman presents with 5-hour history of progressive shortness of breath, cough, and wheezing. This morning she felt that she was "catching a cold" because of sore throat and thin purulent rhinorrhea, for which she took aspirin. Her past medical history is significant for persistent rhinitis resistant to therapy. Question What should your patient do to prevent future asthma attacks? Answer Choices 1 Avoid aspirin 2 Take antihistamine 3 Inhale cromolyn 4 Take albuterol 5 Get influenza vaccine

Avoid aspirin Your patient most probably has aspirin-exacerbated respiratory disease. It is chronic sinusitis characterized by nasal polyposis, nonallergic induced asthma, and aspirin sensitivity. The condition is sometimes called aspirin triad. It commonly starts in the third decade. Clinical symptoms of aspirin-sensitive patients are characterized by mucosal inflammation and rhinitis, severe asthma precipitated by aspirin ingestion, and aggressive bilateral nasal polyposis. The rhinitis is persistent and difficult to manage, and the rhinorrhea is thin and nonpurulent (nonallergic rhinitis with eosinophilia syndrome). Asthma usually appears an average of 2 years after rhinitis, followed by the intolerance to aspirin and the co-occurrence of nasal polyps. Severe acute asthma attack can occur within a few minutes and up to 3 hours after ingestion of aspirin. Aspirin challenge can be used to confirm a diagnosis of aspirin sensitivity in these patients. Antihistamines are commonly used for the relief of allergies caused by intolerance of proteins. They will not help your patient. Inhaled cromolyn or nedocromil, inhaled corticosteroids, long-acting bronchodilators, and other drugs are used to control asthma, but they will not to prevent asthmatic attack in this patient. Albuterol and other quick relief medications used to relieve acute asthma exacerbations, to prevent exercise-induced asthma symptoms, and to speed recovery from acute exacerbations, but albuterol will not prevent aspirin-exacerbated respiratory disease. Influenza vaccine is useful to prevent influenza and the eventual need for the use of nonsteroidal anti-inflammatory drugs. It will not, however, prevent aspirin-exacerbated respiratory disease.

A 22 year-old male presents with a 2-week history of an upper respiratory infection that hasn't improved after taking amoxicillin for 6 days. He notes persistent sore throat, intermittent fever, and a worsening nonproductive cough. Physical examination reveals bilateral diffuse crackles. What is the most appropriate antibiotic to initiate after discontinuing the amoxicillin? A Amoxicilin and clavulanic Acid B Azithromycin C Cephalexin D Ciprofloxacin E Trimethoprim/Sulfamethoxazole

Azithromycin B The patient most likely has atypical pneumonia that responds best to macrolide antibiotics (B). Antibiotics that inhibit cell wall synthesis (A, C) are generally ineffective against these atypical organisms that are either intracellular or lack a cell wall.

A 30-year-old female presents with a five day history of a sore throat. She denies cough or nasal congestion. She also denies vomiting or diarrhea. On physical exam, her temperature is 101˚F, the pharynx is red with tonsillar exudates, and she has tender anterior cervical lymphadenopathy. What is your next step? A Perform culture and sensitivity B Perform rapid strep C Treat symptomatically with antipyretics D Begin oral penicillin E Begin oral ciprofloxacin

Begin Oral PCN The centor criteria include fever, tonsillar exudates, tender anterior lymphadenopathy and lack of cough. These signs and symptoms highly suggest group A beta hemolytic strep. Treatment with penicillin would be the most appropriate step, especially if cost is a concern to the patient. Penicillin v potassium, 250 mg three times per day, or 500 mg twice daily for 10 days, is highly effective. Some studies show a five-day regime to be as effective.

A 45-year-old woman develops a high fever, severe headache, cough, and muscle aches. She is quite upset about missing work because she owns her own pet store and has no employees to cover for her in her absence. She tries to work for a day despite her illness, but she is unable to do so. The next day, she seeks medical attention. On physical examination, she is found to be febrile. Based on her occupational history and her physical examination, you suspect psittacosis. From what animal did she probably acquire the psittacosis? Answer Choices 1 Squirrel 2 Bird 3 Dog 4 Cat 5 Rabbit

Bird

A 23-year-old male presents to the clinic complaining of left anterior neck pain that developed over the past week following recovery from an acute upper respiratory infection. On physical exam a tender mass is felt anterior to the left sternocleidomastoid muscle from the mandible inferiorly to the level of the cricoid cartilage. Which of the following is the most likely diagnosis? A Branchial cleft cyst B Dermoid cyst C Peritonsillar abcess D Salivary gland tumor E Thryoglossal duct cyst

Branchial cleft cyst The development of a neck mass in a young adult following URI is consistent with branchial cleft cyst (A) and thyroglossal duct cyst (E). The location of this mass away from the midline and anterior to the SCM is most consistent with branchial cleft cyst (A). The location of the mass and history are inconsistent with dermoid cysts (B), which are typically midline, peritonsillar abcesses (C), which would be located in the retropharyngeal space, and salivary gland tumors (D), which would be located in the parotid, submandibular, or submental salivary regions.

An 87-year-old woman presents with progressive shortness of breath. She has been in a wheelchair for 15 years due to paralysis of her lower extremities from unknown causes. At this time, she is unable to transfer from the chair to her wheelchair without having dyspnea. She is extremely tired, but denies chest pain, palpitations, cough, hemoptysis, dysphagia, hoarseness, or sick exposures. She has never smoked. Her past medical history is positive for hypertension (treated with enalapril), heart failure, chronic kidney disease, hepatitis C, breast cancer, s/p lumpectomy, and radiation treatment 10 years ago. You order a chest X-ray. Question What in her medical history would lead you to suspect an exudative pleural effusion? Answer Choices 1 Breast cancer 2 Chronic kidney disease 3 Cirrhosis 4 Heart failure 5 Hypertension

Breast Cancer A pleural effusion results from an abnormal accumulation of fluid in the pleural space. Transudative pleural effusions suggest the absence of localized pleural disease and are caused by increased hydrostatic pressure, as seen in heart failure, or decreased oncotic pressure, as seen in cirrhosis. Because there is no underlying pleural disease process, transudative pleural effusions are often bilateral. Exudative pleural effusions are indicative of an underlying pleural disease process, such as pneumonia or a malignancy. A diagnostic thoracentesis can help distinguish transudative from exudative pleural effusions. The fluid should be sent for a cell count, chemistry, and, if indicated, cytology or culture. A pleural effusion is diagnosed as exudative based on the chemistry when the pleural fluid meets 1 or more of the following criteria: "1) ratio of pleural fluid protein to serum protein greater than 0.5; 2) ratio of pleural fluid LDH to serum LDH greater than 0.6; and 3) pleural fluid LDH greater than 2/3 the upper limit of normal serum LDH." The patient presented in this case was treated for left-sided breast cancer with a lumpectomy and radiation therapy 10 years ago. She underwent a thoracoscopy and pleurodesis. During the procedure, 1500 cc of pleural fluid was removed. Cytology was positive for breast cancer. A patient with chronic kidney disease, heart failure, or cirrhosis is more likely to have bilateral transudative pleural effusions. Hypertension by itself is not a risk factor for a pleural effusion.

A 27-year-old woman presents with 3 days of fever, chills, headache, and a deep dry cough. She has been working at a pet store for the past month and thinks that one of the parakeets that came in 10 days ago may be sick. On examination, she has dullness to percussion of the right lung base and right-sided coarse crackles. The most likely diagnosis is A sarcoidosis B tularemia C psittacosis D brucellosis E listeriosis

C psittacosis The key piece of history in this question is the new exposure to parakeets. The symptoms and signs, including atypical pneumonia, are consistent with psittacosis but are not pathognomonic. Sarcoidosis is an illness of unknown cause. Listeriosis has been linked to exposures to contaminated food, particularly dairy products and hot dogs. Brucellosis can be caused by exposure to hogs, cattle, or goats. Tularemia is associated with contact with rabbits, other rodents, and biting arthropods.

What is the most common hematologic finding in a patient with pulmonary hypertension? A Anemia B Thrombocytopenia C Polycythemia D Leukocytosis E Elevated mean corpuscular volume (MCV)

C. Polycythemia Polycythemia is the most common finding. Hematocrits that are >60% usually require phlebotomy, to reduce the numbers and prevent a hypercoagulable state.

A 62 year-old woman with a 40 pack year smoking history presents with a history of progressive dyspnea of several months duration. She denies hemoptysis, cough, fever, or orthopnea. Sputum production is positive. On physical exam, she shows evidence of accessory muscle use when breathing, sits in a slightly bent forward position, uses pursed lips, and you notice she is very thin. Auscultation reveals decreased vesicular breath sounds with prolonged expiration. Which of the following is the most likely diagnosis? A CHF B COPD C Lung cancer D Pulmonary emboli E Pulmonary fibrosis

COPD The patient scenario is consistent with COPD (B) based on the history of progressive dyspnea and physical exam findings of obstructive lung disease (i.e., pursed lip breathing and decreased breath sounds with prolonger expiratory phase. The patient lacks physical exam findings of pulmonary edema due to CHF (A). She is at increased risk of lung cancer (C), but lacks red flag symptoms at this time. Pulmonary embolism (D) typically occurs in patients with venous stasis, hyper coagulable state, and vascular wall injury. Pulmonary fibrosis (E) would present with sign of restrictive lung disease (e.g., shallow breaths with rapid expiration)

A 58-year-old woman with a past medical history of hypertension, hyperlipidemia, and breast cancer is being evaluated in the emergency room after presenting with a 2-hour history of acute-onset severe left-sided pleuritic chest pain. The pain is associated with feelings of anxiety, hemoptysis, shortness of breath, and nausea. She "feels warm" but denies other complaints. She endorses a 30 pack-year smoking history. Upon physical exam, she is found to be febrile, normotensive, tachypneic, diaphoretic, and in acute painful distress. There is perioral cyanosis and a pleural friction rub to the left lung fields; the remainder of the exam is normal. A bedside electrocardiogram revealed the following image. Question What is the next correct step in the management of this patient? Answer Choices 1 Order a CT angiography of the chest 2 Refer the patient for an echocardiogram 3 Obtain magnetic resonance imaging of the chest 4 Prescribe an oral proton-pump inhibitor 5 Begin recombinant tPA infusion

CT angio This patient's presentation suggests a pulmonary embolism. EKG findings include an S1Q3T3 pattern, sinus tachycardia, T wave inversion in leads V1 - V3, Right Bundle Branch Block, and low amplitude deflections. Computed tomography angiography (CTA) is the initial imaging modality of choice for stable patients with suspected pulmonary embolism.The American College of Radiology (ACR) considers chest CTA to be the current standard of care for the detection of pulmonary embolism. The echocardiogram generally has limited accuracy in the diagnosis of pulmonary embolism. Few investigators have reported the feasibility of MRI in the evaluation of pulmonary embolism. However, the role of MRI is mostly limited to the evaluation of patients who have impaired renal function or other contraindications for the use of iodinated contrast material. Prescribing a proton pump inhibitor is not appropriate at this time, as the clinical presentation suggests pulmonary embolism. Thrombolytic therapy is not recommended for most patients. The role of thrombolytic therapy in the management of acute pulmonary embolism is controversial and has not demonstrated benefits in terms of reduced mortality rates or earlier resolution of symptoms when currently compared with heparin. The currently accepted indications for thrombolytic therapy include hemodynamic instability, such as systolic blood pressure less than 90 mmHg or systolic blood pressure drop of greater than 40 mmHg from baseline for at least 15 minutes, or right ventricular dysfunction demonstrated on echocardiogram. Thrombolytic therapy should not be used in patients with major contraindications due to bleeding risks.

A 67-year-old female with a history of oxygen dependent emphysema presents with a 4-hour history of increasing shortness of breath and pleuritic chest pain on the right side. Her resting oxygen saturation rate is 90%, and she is having pain on inspiration. On examination, the patient has decreased lung sounds with wheeze on the left and absent sounds on the right. There is also tympany to percussion on the right. Based on these findings, what is the best therapy for this patient? A Needle insertion to right chest wall B Supportive care C Increased oxygen delivery D Chest tube insertion E Nebulized albuterol

Chest Tube Insertion The treatment for this patient, who has a pneumothorax, is chest tube insertion and reinflation of the lung. Once the air leak has been eliminated and the lung appears reinflated on serial chest x-rays, the chest tube may be removed.

A 60-year-old man presents with a 2-year history of shortness of breath. He has been a heavy smoker for 40 years. He has a chronic cough; it produces at least ½ a cup of mucoid sputum a day. He was hospitalized 3 times last year for infective respiratory disease. He is currently on asthmatic medication; he is taking 5mg salbutamol, 250-mcg ipratropium 4 hourly by nebulizer, slow release theophylline 300-mg b.i.d., and prednisolone 10 mg daily. At present, he presents with history of worsening of cough and shortness of breath. On examination, the patient is cyanotic, T 100, P 100/m, and RR 25. What should the initial investigation be? Answer Choices 1 Pulmonary function test 2 Chest X-ray 3 Sputum culture 4 CBC 5 Blood culture

Chest Xray A chest X-ray can determine the extent of pulmonary consolidation. It will also exclude pneumothorax or carcinoma. Therefore, it is the best choice. A pulmonary function test is not done during the acute illness, and an estimation of blood gases is unlikely to influence the immediate management. CBC is not important in immediate management. Blood culture and sputum should be done afterward to determine further antibiotic treatment, but it would not influence immediate management.

A 53-year-old woman presents with a 2-year history of chronic cough. The cough produces large volumes of grossly purulent sputum. She has a history of recurrent respiratory infections; they resulted in 5 hospitalizations in the past year. She also had similar complaints during the previous year.. Shortness of breath limits her daily activity considerably. Upon pulmonary examination, bilateral breath sounds are audible, with inspiratory and expiratory crackles at the lung bases. Chest X-rays reveal increased lung volumes, flattened diaphragm, and tram track lines. What is the initial diagnosis? Answer Choices 1 Asthma 2 Emphysema 3 Pulmonary edema 4 Chronic bronchitis 5 Pulmonary fibrosis

Chronic Bronchitis The clinical picture is suggestive of chronic bronchitis. Chronic bronchitis is defined by a clinical history of productive cough for 3 months of the year for 2 consecutive years. Smoking is the leading cause. The principle pathologic feature is airway injury and narrowing, hypertrophy of the airway mucous glands, infiltrate of inflammatory cells, and loss of ciliated epithelium. The cough produces thick, often purulent sputum because of the ongoing local inflammation and the high likelihood of bacterial colonization and infection. The increased mucous production and defective mucociliary escalator function leads to inspiratory and expiratory crackles. On imaging, common findings are hyperinflation of lung volumes, depressed diaphragm, and parallel linear densities known as tram track lines. Asthma produces wheezing lung sounds. They are not heard in this patient. A productive cough is not a clinical manifestation of emphysema. Pulmonary edema may present with mild exertional dyspnea or a nonproductive cough, although a frothy or blood-tinged sputum may be seen. Pulmonary fibrosis is a restrictive lung disease with the clinical features of progressive dyspnea. It is typically accompanied by a dry, persistent hacking cough.

A man presents with a chronic respiratory infection; he is seeking the advice of an ear, nose, and throat specialist. A biopsy of his respiratory epithelium reveals an alteration in certain epithelial structures. What is most likely to be abnormal? Answer Choices 1 Microvilli 2 Desmosomes 3 Cilia 4 Hemidesmosomes 5 Stereocilia

Cilia In some individuals, a structural abnormality involving the absence of dynein arms in cilia (Kartagener syndrome, a hereditary disease) is associated with chronic respiratory difficulty (e.g., bronchitis and sinusitis). Ciliary motility is severely impaired or absent in these individuals. The epithelium in the respiratory passages does not possess microvilli. Desmosomes (maculae adherens) are cell-to-cell junctions attached to the lateral sides of 2 cells; it is the only attachment device present in epidermal cells. In other epithelia, especially those with cuboidal or columnar cells, desmosomes are found in conjunction with a zonula adherens. They are thought to play a role in dissipating physical forces throughout the cell from the attachment site. Hemidesmosomes, present on the basal surface of the cells of the immune system, occur where epithelia require stronger adhesion to the connective tissue; they are believed to confer a 'social identity' onto cells. Stereocilia (long microvilli) are limited to the epididymis of the male reproductive system. Stereocilia of the sensory epithelium of the ear are uniform in diameter and possess an internal filamentous structure. They serve as a receptor device rather than an absorptive structure.

A 52-year-old woman with a history of untreated bronchogenic carcinoma presents with a 1-week history of dyspnea, facial edema, and marked neck vein distention; the distention has progressed over the past 36 hours. A chest X-ray confirms a right hilar mass with a small pleural effusion in the right hemithorax. Which of the following steps is most appropriate? Answer Choices 1 Consult a cardiologist for pericardiocentesis 2 Consult an oncologist for radiation therapy 3Consult a cardiothoracic surgeon for a mediastinoscopy 4 Consult a cardiovascular surgeon for exploration thoracotomy 5 Prepare the patient for a pleurocentesis in the office

Consult an oncologist for radiation therapy The patient is presenting with signs and symptoms of Superior Vena Cava (SVC) syndrome, caused by obstruction of blood flow through the SVC due to compression, thrombosis, or infiltration from a right upper hilar malignancy. Obstruction of venous drainage at the upper thorax causes venous distention. Morbidity may result from irreversible thrombosis, central nervous system damage, or pulmonary complications. The most common cause of obstruction is malignant disease, with bronchogenic carcinoma at the top of the list. Signs and symptoms of SVC syndrome include swelling and cyanosis in the face and upper trunk, dyspnea, dysphagia, neck vein distension, and headache. Cerebral edema is a potential complication. In patients who present with serious airway or central nervous system systems, the preferred management strategies are SVC stenting and/or radiation therapy. Diuretics, steroids, and thrombolytics, are sometimes used as temporizing measures, but their effectiveness is questionable. The diagnosis of pericardial effusion is not supported by the history, physical exam, and chest film; therefore, pericardiocentesis is not indicated. Direct surgical interventions are ineffective in this case. The small pleural effusion present on chest x-ray is unlikely to account for the patient's symptoms, and an in-office pleurocentesis would not be advisable.

A 69-year-old man presents with dyspnea on exertion that has been slowly progressive over the course of the last year. He notes impairment in climbing stairs and walking short distances. His review of systems is positive for fatigue, palpitations, intermittent retrosternal chest pain, swelling of his lower extremities, dizziness, and "feeling faint;" his associated symptoms are also known to occur upon exertion. He denies any fever, chills, weight changes, cough, abdominal pain, early satiety, nausea, vomiting, diarrhea, changes to his urine color or odor, flank pain, hematuria, or dysuria. He denies any cigarette, alcohol, or drug use. His cardiac exam is remarkable for an increased pulmonic component of the second heart sound (P2), wide, inspiratory splitting of S2 over the cardiac apex, right-sided S3 and S4 gallops, a left parasternal lift, a loud diastolic murmur that increases with inspiration and diminishes with the Valsalva maneuver, prominent "A" waves in jugular venous pulsations, and increased JVD. He has an enlarged liver with hepatojugular reflux, peripheral edema, and ascites. A bedside EKG analysis revealed peaked P waves, rightward axis deviation, and prominent R waves in the early V leads. Question What is the most likely diagnosis? Answer Choices 1 Myocardial infarction 2 Cor pulmonale 3 Primary biliary cirrhosis 4 Left ventricular heart failure 5 Pulmonary embolism

Cor Pulmonale Myocardial infarctions occur at rest and most commonly in the early morning. The pain is similar to angina in location and radiation, but it may be more severe and builds up rapidly or in waves to maximum intensity over a few minutes or longer. Associated symptoms include diaphoresis, weakness, apprehension, and a feeling of impending doom; patients may move about, seeking a position of comfort, preferring not to lie quietly. Light-headedness, syncope, dyspnea, orthopnea, cough, wheezing, nausea and vomiting, or abdominal bloating may occur. Physical exam findings may include fever, anxiousness, diaphoresis, bradycardia or tachycardia, low cardiac output, or arrhythmia. There may be hypertension (in hypertensive patients) or low in patients with shock. Respiratory distress, jugular venous distention a Kussmaul sign, soft heart sounds, and atrial gallops (S4) or ventricular gallops (S3) usually indicate heart failure. Primary biliary cirrhosis is most common in middle-aged females and is characterized by fatigue, pruritus, hepatosplenomegaly, xanthomatous lesions on the skin, eyelids, and tendons, jaundice, and steatorrhea. Signs of portal hypertension are late findings. Other findings include orthostatic hypotension and cognitive dysfunction. Left-sided or forward failure may account for many of the clinical manifestations of heart failure, such as mental confusion from decreased cerebral perfusion, fatigue and weakness from decreased skeletal muscle perfusion, and sodium and water retention with secondary venous congestion from decreased renal perfusion. Isolated left-side heart failure is associated with dyspnea, fatigue, weakness, cough, paroxysmal nocturnal dyspnea, and orthopnea in the absence of peripheral edema, jugular venous distention (JVD), or hepatojugular reflux. Chest pain, dyspnea, and tachypnea are the most frequent signs and symptoms of pulmonary embolism. Other manifestations may include tachycardia, pleurisy, low-grade fever, apprehension, and productive cough with blood-tinged sputum. Massive PE may manifest as sudden collapse, crushing substernal chest pain, shock, diaphoresis, hypotension, distended neck veins, and loss of consciousness.

A 3-year-old boy presents with difficulty in breathing and a cough that sounds like a seal. On examination, the child has fever, a harsh barking cough, a respiratory rate of 38/minute, and minimal stridor on agitation. On lung auscultation, there are no rales or wheezing. On cardiac auscultation, there is tachycardia. Radiological examination reveals the so-called 'steeple sign'. What is the most likely diagnosis? Answer Choices 1 Bronchiolitis 2 Croup 3 Epiglottitis 4 Foreign body aspiration 5 Pneumonia

Croup Croup, which is caused by the parainfluenza virus, is an upper respiratory infection with a characteristic brassy cough, hoarseness, and respiratory stridor. The typical clinical picture has a variable onset, from a progressive fever, sore throat, and cough for a couple of days to a sudden midnight onset. The severity varies from stridor only (with mild agitation) to stridor at rest to airway obstruction. Treatment involves humidified air, inhaled or oral corticosteroids, and nebulized racemic epinephrine. Bronchiolitis, caused by the respiratory syncytial virus (RSV) and other respiratory viruses (influenza, parainfluenza, rhinovirus), is a severe infection in infants that can progress to respiratory failure. Premature infants and infants with heart and lung disease have severe forms of bronchiolitis. The patients have wheezing, rales, prolonged expiratory phase, rhinorrhea, nasal congestion, fever, tachypnea, and respiratory distress. Antigen testing for RSV from nasal secretions provides a rapid diagnosis. Ribavirin administered through aerosol may shorten the clinical course. Polyclonal and monoclonal antibodies are used for prophylaxis in wintertime. Epiglottitis, which is caused by Haemophilus influenzae, is characterized by inflammation and rapidly progressive edema of epiglottis and contiguous tissue. Children ages 2 to 7 years who missed some immunizations are prone to this infection during winter. Drooling, hoarseness, high fever, sore throat, the characteristic 'sniffing dog' position, and the rapidly progressive respiratory obstruction make the diagnosis a clinical one. Immediate intubation and intravenous ceftriaxone or cefuroxime are recommended for treatment. Foreign body aspiration presents with complete or partial airway obstruction localized at a different location in the respiratory tract. The acute presentation after a choking episode provides the diagnosis; however, without a witnessed episode of choking, clinical presentation may take a couple of days after the aspiration. Tracheal obstructions present with acute asphyxia in total obstruction and with stridor in partial obstruction. Localized signs, such as wheezing, rhonchi, and decreased breath sounds, are found in lower respiratory tract obstruction. The treatment of choice is extraction with rigid bronchoscope. Pneumonia in children may be a difficult clinical diagnosis. The immune status, the age, and risk factors may help in suspicion of the etiologic organism. Profuse crackles and rales after an upper respiratory infection may suggest a viral pneumonia. A localized consolidation syndrome is more characteristic for a bacterial origin of pneumonia. The characteristic findings on chest radiography also suggest the etiology of the pneumonia.

A 2-year-old boy presents with chronic respiratory infections, foul-smelling pale stools, weight loss, and wheezing. His mother reports using moist mist treatments repeatedly, and they offer some relief. A clinical exam shows delayed growth, an enlarged spleen upon palpation, and clubbing of fingers and toes. Auscultation of the lungs demonstrates pulmonary congestion. Lab results report a positive fecal fat test and sweat chloride test. What is the most likely diagnosis? Answer Choices 1 Cystic fibrosis 2 Pulmonary tuberculosis 3 Pulmonary histiocytosis 4 Pulmonary aspergilloma 5 Pulmonary nocardiosis

Cystic Fibrosis ystic fibrosis is an inherited disease that affects the respiratory and digestive systems. It affects the exocrine (mucus and sweat) glands of the body and is caused by a defective gene. It is the most common cause of chronic lung disease in children and young adults. Thick mucus is formed in the bronchial tree, which predisposes the person to chronic lung infections. Symptoms include no meconium stool in the first 24 hours of life, stools (pale or clay colored, foul-smelling, and floating), skin may taste salty (infants), persistent respiratory infections, coughing or wheezing, weight loss, clubbing of the fingers or toes, diarrhea, delayed growth, easy fatigue, and splenomegaly. Tests should include sweat chloride test, a fecal fat test (positive), bone X-ray, chemistry panel, upper GI, small bowel series, as well as immunoreactive trypsinogen. Treatments include antibiotics for respiratory infections and pancreatic enzymes to replace the missing enzymes. Mucomyst may be used on occasion to thin secretions. The use of ibuprofen has been shown to slow lung deterioration in some children. Other treatments include postural drainage, chest percussion, and other breathing treatments. Lung transplant may be considered in some cases. About 50% of patients with cystic fibrosis live beyond age 20. Few patients live beyond 35. Death occurs from pulmonary (lung) complications. Pulmonary tuberculosis is a contagious bacterial infection caused by Mycobacterium tuberculosis (TB). The lungs are primarily involved, but the infection can spread to other organs. The disease is characterized by the development of granulomas (granular tumors) in the infected tissues. Primary infection is usually asymptomatic. Symptoms may include initially but are not limited to minor cough and mild fever, fatigue, weight loss, coughing up blood, slight fever and night sweats, wheezing, rales, excessive sweating, joint pain, hearing loss, diarrhea, chest pain, breathing difficulty, positive Babinski's reflex, and clubbing of the fingers or toes. Pulmonary histiocytosis is characterized by inflammation of the small airways (bronchioles) and the small blood vessels in the lungs. This inflammation leads to stiffening (fibrosis) and destruction of the walls of the alveoli. Systemic involvement may cause rashes, pulmonary problems, enlargement of the spleen and liver, anemia, and death. In children, symptoms can include failure to thrive, weight loss, irritability, fever, seborrheic dermatitis of the scalp, generalized rash (petechiae or purpura), and chronically draining ears; bone pain may or may not be present. Pulmonary aspergilloma is a fungal mass that grows in pre-existing lung cavities (or can cause new lung cavities). The preexisting cavities may have been caused by a previous infection of histoplasmosis, tuberculosis, sarcoidosis, lung abscess, cystic fibrosis, or previous lung cancer. Symptoms include cough, coughing up blood, weight loss, and fever. Tests include bronchoscopy or bronchoscopy with lavage (BAL), chest X-ray, and sputum culture. The most effective therapy is a surgical resection (cutting out the fungus growth). If life-threatening bleeding occurs, emergency surgery may be the only choice of treatment. Pulmonary nocardiosis is an infection of the lung (pneumonia that is caused by a fungus-like bacterium). The causative organism is present throughout the world, and the infection is acquired through inhalation, causing a pneumonia-like illness. The disease can spread to any part of the body, but brain lesions and subcutaneous (under the skin) lesions are most common. Symptoms include cough with sputum production, progressive difficulty breathing, general discomfort, uneasiness or ill feeling (malaise), weight loss, fever (intermittent), night sweats, chest and joint pain, as well as liver and spleen enlargement (hepatosplenomegaly).

A 14-year-old boy presents with worsening shortness of breath; it most often occurs when he plays soccer. He often awakens in the middle of the night due to 'attacks': he starts to feel anxious because he feels like he cannot breathe, and he experiences chest tightness. He suffers from a dry cough, especially after playing sports. His mother has put a humidifier in his room and has him use his sister's inhaler, which seems to help temporarily; the boy has been using it 5 - 6 times daily. The family history is significant for asthma in his sister, father, and 3 other paternal relatives. The patient and his mother are not aware of any allergies. He denies fever, chills, and chest pain. In between 'attacks', he feels well and normal. The patient's past medical history is noncontributory. There are no known medical conditions; he has no drug allergies, and he has not had any surgeries. Other than the aforementioned inhaler, he does not take any medications. Question In addition to his own albuterol inhaler, what medication should be prescribed for this patient? Answer Choices 1 Burst of oral prednisone 2 Daily inhaled salmeterol 3 Daily low-dose inhaled budesonide 4 Daily oral zileuton 5 Subcutaneous injections of omalizumab

Daily low-dose inhaled budesonide This patient is exhibiting persistent asthma symptoms. Using the step-wise approach to treatment, the 2nd step (after using a short-acting beta-agonist, such as albuterol) is to add a low-dose inhaled corticosteroid. Alternatives include cromolyn, a leukotriene receptor antagonist, nedocromil, and theophylline. For this patient, daily low-dose inhaled budesonide would be most appropriate. He should then be monitored for response; if necessary, the medication should be adjusted. The patient and his mother should receive patient education on asthma, the use of peak flow meters, and the proper use of medications. A burst of oral prednisone can be useful in management of an acute exacerbation of asthma. However, due to the side effects of systemic steroids, its use is discouraged. This patient is not in acute distress and should start with inhaled steroids, which pose fewer side effects than oral/systemic steroids. Daily-inhaled salmeterol, a long-acting beta-agonist (or LABA), should be added to an inhaled corticosteroid if low-medium doses of the inhaled steroid alone are unable to control symptoms. The salmeterol is not recommended as a stand-alone therapy, and it should only be used with other asthma control medications. Daily oral zileuton, a 5-lipoxygenase inhibitor, is extremely expensive and is dosed 4 times daily. It should not be recommended initially when the preferred agent (low-dose inhaled corticosteroid) has not yet been tried. Additionally, compliance in teens can be difficult, so a 4-times-daily medication is not ideal. Subcutaneous injections of omalizumab, which is an immunomodulator, are an option in the step 5 and 6 patients, who have failed to achieve symptom control with multiple and high-dose medications. Omalizumab is not appropriate in this patient's case.

A 34-year-old trauma patient in the ICU is in a lot of pain, so you order 100mg of meperidine hydrochloride every 4 hours. What will the patient have to be closely monitored for? Answer Choices 1 Bowel function 2 Increase in blood pressure 3 Increase in heart rate 4 Increase in respiration 5 Decrease in respiration

Decrease in respiration Meperidine hydrochloride (Demerol) is a CNS depressant and could therefore decrease respiration and heart rate. It binds with opiate receptors, altering perception of and emotional response to pain through an unknown mechanism. If there were an effect on blood pressure, it would be hypotension, especially after rapid i.v. injection, and not an increase in blood pressure. As with all narcotic analgesics, there is the danger of constipation when administered over a longer period of time, but there is no need to monitor bowel function closely. Other side effects of meperidine are shown in the following table. Central nervous system: Sedation, euphoria, dizziness, tremor, headache, hallucination, seizures, light-headedness Gastrointestinal: Nausea, vomiting, dry mouth, constipation, ileus, biliary tract spasms Cardiovascular: Bradycardia, tachycardia, hypotension, cardiac arrest, shock Respiratory: Respiratory depression, respiratory arrest Urogenital: Urine retention Musculoskeletal: Twitching Skin : Pruritus, urticaria, increased perspiration Other: Physical dependence, pain at injection site, induration, increased pancreatic enzyme levels Meperidine is contraindicated in patients with known hypersensitivity to the drug or who have been treated with MAO-inhibitors within the past 2 weeks.

A 76-year-old Caucasian man with a 90 pack per year smoking history presents with progressive fatigue, tachypnea, exertional dyspnea, cough, and lower extremity edema. Inspection of his chest and abdomen reveals an increased chest diameter, labored respiratory efforts with retractions and cyanosis, left parasternal and subxiphoid heaves, hepatojugular reflux, and a pulsatile liver. Additionally, there is scattered wheezes and crackles in his lungs and bilateral lower extremity edema. Question What physical exam findings would be most consistent with the underlying diagnosis? Answer Choices 1 Distended neck veins with prominent a or v waves 2 Dullness to percussion and increased tactile fremitus of the lungs 3 Pericardial friction rub 4 Laterally displaced and enlarged point of maximal impulse 5 A systolic ejection murmur located at the aortic valve area

Distended neck veins with prominent a or v waves This patient's diagnosis is chronic cor pulmonale. Cor pulmonale is estimated to account for 6-7% of all types of adult heart disease in the United States, with chronic obstructive pulmonary disease (COPD) due to chronic bronchitis or emphysema the most common cause (the causative factor in more than 50% of cases). Cor pulmonale usually presents chronically; however, 2 main conditions can cause acute cor pulmonale: pulmonary embolism (more common) and acute respiratory distress syndrome (ARDS). In chronic cor pulmonale, right ventricular (RV) hypertrophy (RVH) generally predominates. Common symptoms are fatigue, tachypnea, exertional dyspnea, and cough. Other symptoms include anginal chest pain (due to right ventricular ischemia), hemoptysis, and, rarely, hoarseness due to compression of the left recurrent laryngeal nerve by a dilated pulmonary artery. Anorexia, right upper quadrant abdominal discomfort, and jaundice may occur due to passive hepatic congestion. Physical exam findings reflect the underlying lung disease or pulmonary hypertension, right ventricular hypertrophy (RVH), and RV failure. Expected signs include an increase in chest diameter, labored respiratory efforts with retractions of the chest wall, distended neck veins with prominent a or v waves, and cyanosis. Wheezes and crackles may be audible upon auscultation due to underlying lung disease. S2 heart sound splitting with an accentuated pulmonic component may be found early, while a systolic ejection murmur with sharp ejection click over the region of the pulmonary artery may be heard in advanced disease, along with a diastolic pulmonary regurgitation murmur. Other cardiac findings include third and fourth sounds and the systolic murmur of tricuspid regurgitation. RVH is characterized by a left parasternal or subxiphoid heave. Hepatojugular reflux and pulsatile liver are signs of RV failure with systemic venous congestion. On percussion, hyperresonance of the lungs may be a sign of underlying COPD; ascites can be seen in severe disease. Examination of the lower extremities reveals evidence of pitting edema. Dullness to percussion and increased tactile fremitus of the lungs suggests a lung consolidation, as in pneumonia, pulmonary edema, or pulmonary hemorrhage. Pericardial friction rubs suggest inflammation of the pericardial sac, while a laterally displaced and enlarged point of maximal impulse occurs with left ventricular hypertrophy, congestive heart failure, cardiomyopathy, and ischemic heart disease. The finding of a systolic ejection murmur located at the aortic valve area most likely suggests a diagnosis of aortic stenosis.

A 55-year-old man presents with a COPD exacerbation that is being managed with a ventilator. The patient's blood pressure drops and the ventilator alarm goes off. The only medication administered is amlodipine via nasogastric tube. This patient is afebrile, even though the hospital has been having problems with pseudomonas infection in ventilated patients. On examination, there is a middle-aged orally-intubated man with temperature 99.4° F, pulse 145/min, and BP 62/34 mm Hg; he breathes above the ventilator at a rate of 36 cycles/min even. His breath is shallow and he has diminished breath sounds in his right hemithorax. What is the most appropriate next step? Answer Choices 1 Add positive end-expiratory pressure 2 Do thoracotomy/chest tube placement 3 Start antibiotics 4 Give IV boluses 5 Start pressor agents

Do thoracotomy/chest tube placement The patient's diagnosis is right tension pneumothorax in a COPD patient on a ventilator. Features supportive of tension pneumothorax are hypotension, tachycardia, tachypnea, shallow breathing, and decreased breath sounds in the right hemithorax. Management of this condition is emergent thoracentesis through insertion of a large bore needle into the second intercostal space. Chest tube placement is indicated for definitive treatment. Positive end-expiratory pressure is the maintenance of positive pressure in the alveolar at the end of expiration. Its application will cause an increase in the mean and peak airway pressure and worsen the pneumothorax by forcing more air into the lungs. Antibiotics will be used if there is an infective process going on, but this is not evident. Since the hypotension is related to decreased venous return because of increased intrathoracic pressure leading to decreased cardiac output, IV fluids or pressors are unlikely to increase the blood pressure.

A 45-year-old man presents for a certification of disability. The man works in a nursery. He gives history of occasional breathlessness for which no medical record is present. Examination shows tachypnea, and scattered rhonchi are heard all over the chest. In what way is spirometry helpful in this case? Answer Choices 1 Done alone 2 Done once and repeated again with bronchodilator therapy 3 Done only with bronchodilator therapy 4 Spirometry is not important 5 Spirometry conducted alone or with bronchodilator therapy, does not matter

Done once and repeated again with bronchodilator therapy Spirometry is the most useful assessment of impairment caused by airway obstruction. It is used to determine the impairment leading to disability. It should be done initially and then repeated with bronchodilator therapy, because it proves that the 1st result was due to blockage of the airway.

A 15-year-old male presents complaining of a sore throat, headache, and mild cough that started 8 days ago and has progressed to include a worsening cough and increasing fatigue. His chest x-ray reveals bilateral hilar infiltrates, CBC is normal and a nasal secretions test positive for mycoplasma pneumoniae by PCR. What is the most appropriate therapy? A Amoxicillin B Cefuroxime C Clindamycin D Ciprofloxacin E Doxycycline

Doxycycline Mycoplasma pneumoniae is commonly treated with macrolides, doxycycline (E), or respiratory fluoroquinolones. Mycoplasma pneumonia doesn't respond to beta-lactam antibiotics (A, B) or non-respiratory fluoroquinolones (D).

A 42 year-old woman presents complaining of shortness of breath, three days of fever as high as 103F, and has a cough productive of green sputum. On physical examination, you hear crackles in her lungs. A chest radiograph reveals a consolidation in the left lower lobe. What do you expect to hear when you percuss this patient's left lower thorax? A Dull B Flat C Hyperresonant D Resonant E Tympanic

Dullness Consolidation leads to dullness (A) to percussion. Flat sounds (B) are noted with organ percussion (i.e., hepatic flatness), hyperresonant (C) sounds are noted in pneumothorax, normal lung parenchyma results in resonance (D), and percussion of the gastric air bubble leads to tympanic sounds.

A 64-year-old man with hypertension, coronary artery disease, and poorly-controlled left ventricular congestive heart failure presents with a 3-day history of insidious chest pain. Pain is made worse when he takes a deep breath in and when he coughs. He denies any relation of pain to position, activity, or food intake. He denies fever, chills, palpitations, sputum production, wheezing, abdominal pain, nausea, vomiting, diarrhea, or peripheral edema. His physical exam reveals a widespread friction rub upon inspiration, absent lung fremitus, and reduced lung sounds over the thoracic cavity. Question What additional physical exam finding would be most likely expected in this patient? Answer Choices 1 Vesicular breath sounds 2 Dullness to percussion 3 Tracheal shift to the affected side 4 Chest wall tenderness 5 Increased anteroposterior diameter

Dullness to percussion This patient's presentation is most consistent with a pleural effusion. This patient is demonstrating pleuritis, which is caused by damage and inflammation to the pleura of the lung. The most common cause of pleural effusion is left-ventricular failure. It usually causes a sharp, knife-like pain that is aggravated by inspiration or coughing. Vesicular breath sounds are the normal breath sounds, which occur in most of the lung fields. The trachea may deviate toward the side opposite a large pleural effusion. Chest wall tenderness is an expected finding in chest wall pain syndromes, such as costochondritis. An increased anteroposterior diameter, or barrel chest, may be observed in chronic obstructive pulmonary disease.

During the physical exam of a patient with a suspected pleural effusion, you ask the patient to make the sound "eee." You note on auscultation that the transmission is auscultated as "ay," suggestive of resonance through fluid. What is the name of this exam technique? A Bronchophony B Diaphragmatic excursion C Egophony D Tactile fremitus E Whispered pectoriloquy

Egophony Normal lungs transmit spoken sounds faintly and with indistinct syllables, except over main bronchi. An area of fluid, such as a pleural effusion, consolidation or atelectasis, and areas of fibrosis will increase sound transmission and alter the distinction of the sound. This occurs for both whispered and spoken sounds. The utilization of the spoken sound "eee," auscultating for a change to "ay" due to fluid within the lung fields, is termed egophony. The use of whispered sounds and generally spoken sounds to determine lung changes are termed whispered pectoriloquy and bronchophony respectively. Diaphragmatic excursion is performed to determine the thoracic diaphragmatic movement during respiration. Tactile fremitus assesses chest vibration during vocalization, with changes being noted in the presence of consolidation (increases fremitus) and pleural effusion (decreased or absent fremitus).

An immigrant worker presents with a chronic cough, blood-stained sputum, and night sweats. Physical examination reveals a wasted middle-aged man with bronchial breathings on the right upper lobe. His sputum examination under microscope shows acid-fast staining rods, and his PPD is 15 mm. Based on clinical findings, sputum examination, and PPD result, pulmonary tuberculosis is diagnosed. Treatment is started after sending the sputum for culture. The patient comes back 4 weeks later for a check up, and he mentions not being able to see clearly and not being able to distinguish the color blue from green. What drug is most likely causing this side effect? Answer Choices 1 Isoniazid 2 Pyrazinamide 3 Ethambutol 4 Streptomycin 5 Rifampicin

Ethambutol The correct response is ethambutol. Important side effects of TB drugs are: Ethambutol: optic neuritis and difficulty distinguishing blue from green Isoniazid: hepatitis, and peripheral neuropathy (pyridoxine is the treatment for peripheral neuropathy) Rifampicin: hepatotoxic, hypersensitivity reactions, and red coloring of the urine and other secretions (saliva, tears, stool) Pyrazinamide: hepatotoxic (rare), and hyperuricemia Streptomycin: ototoxic, and nephrotoxic.

A 30-year-old woman presents with a 2-day history of fever, cough with sputum, and chest pain; there is also a 1-day history of frank blood in sputum (75 cc in the last 24 hours). She is a non-smoker and does not give a history of any recent inhalation exposure or illicit drug use. There is no past history of malignancy or autoimmune disease. On examination, pulse is 92/min; BP is 106/70mmHg; and temperature is 101 F. Oxygen saturation is 97%. On auscultation, S1, S2 normal, decreased breath sounds, and increased tactile vocal fremitus are noted on the right middle lobe. There is no pallor, icterus, cyanosis, edema, or lymphadenopathy. Question What is the next best step in evaluation of her hemoptysis? Answer Choices 1 Computed Tomography (CT) scan of chest 2 Sputum for Gram stain 3 Plain X-ray of chest 4 Bronchoscopy 5 Complete blood count (CBC)

Explanation Plain X-ray of chest is the correct answer. It is the next step in the evaluation of hemoptysis to identify the source of bleeding1. If the chest X-ray does not help identify the source of bleeding, then a CT scan of the chest should be done1. CT scan helps to identify the source of bleeding, as well as gives information on mediastinal lymhadenopathy, which may support the diagnosis of thoracic malignancy1. Both Gram stain and CBC should be obtained for complete evaluation1. Gram stain helps evaluate for the presence of infection, and CBC is used to assess platelet count and hematocrit1. These investigations should be done following chest X-ray. If all these procedures are unrevealing, then bronchoscopy should be considered1.

A 56-year-old man presents for a routine follow-up regarding his positive HIV status. He is compliant with his medications and has been feeling well. In addition to his antivirals, he takes daily trimethoprim/sulfamethoxazole for Pneumocystis jiroveci pneumonia (PCP) prophylaxis. Question What detail of his history would warrant the prophylaxis? Answer Choices 1 CD4 cell count <400 2 History of previous PCP infection 3 CD4 cell count >200 4 History of any previous pneumonia infection 5 HIV viral load >100,000 copies/mL

Explanation History of previous Pneumocystis jiroveci pneumonia (PCP) infection is the correct answer. Patients who are HIV positive should take antibiotic prophylaxis for PCP if they have a history of PCP infection, if their CD4 cell count is below 200, or if they have evidence of immunodeficiency, such as oral candidiasis. Prophylaxis for PCP is typically daily trimethoprim/sulfamethoxazole double strength, but it can also be done with daily dapsone or monthly aerosolized pentamidine. If patients are on antiretroviral therapy and their CD4 cell count remains above 200 for at least 3 months, then they can stop prophylaxis. If their CD4 cell count drops below 200 again, they need to restart prophylaxis. CD4 cell count <400 is not the correct answer. Patients who are HIV positive need to be on PCP prophylaxis for various reasons, one of which is a CD4 count <200. A CD4 cell count between 200 and 400 would not warrant prophylaxis, until the point that the count drops below 200. CD4 cell count >200 is not the correct answer. A CD4 cell count over 200 would actually be a reason NOT to have the patient on PCP prophylaxis, as one of the criteria to start prophyhlaxis is a CD4 cell count below 200. In addition, when a patient has been on prophylaxis and antiretrovirals and their CD4 cell count is over 200 for 3 months, they can stop the prophylaxis. History of any pneumonia infection is not the correct answer. Only patients who have had previous PCP infections need to be treated with prophylaxis for PCP. Having had other types of pneumonia does not warrant PCP prophylaxis. HIV viral load >100,000 copies/mL is not the correct answer. HIV viral loads over 100,000 warrant antiretroviral treatment of HIV, but PCP prophylaxis is not given based on viral load. Rather, it is given based on CD4 cell count, previous PCP infection, or evidence of immunodeficiency.

A 4-year-old girl has always been below 3% for her height and weight. Her mother says she eats 3 meals a day with healthy snacks. The family tries to maintain a low-fat, high-fiber diet. Her parents and sisters are of above average height and average weight. Other than occasional upper respiratory infections and 1 episode of pneumonia last winter, the girl has been fairly healthy. Her failure to thrive workup is normal other than a sweat test, which reveals a high chloride concentration. Question What is an important modification to her diet in light of her probable diagnosis? Answer Choices 1 Fat-soluble vitamin supplements 2 Low-protein foods 3 Low-fat foods 4 High-calcium foods 5 Water-soluble vitamin supplements

Fat-soluble vitamin supplements A positive sweat test indicates cystic fibrosis. Positive results should be confirmed, and a negative result should be repeated if there is still a suspicion of cystic fibrosis. Patients with cystic fibrosis usually have loss of exocrine pancreatic function and inadequate digestion of proteins and fats. The optimal diet includes fat-soluble vitamin supplements (A, D, E, and K), pancreatic enzyme replacements, and high-protein and high-calorie foods. Zinc and iron supplements are also sometimes needed. A higher number of calories may be required due to the increased work of breathing and higher metabolic demands. Weight gain is usually not achieved unless lung infection is controlled.

A 48 year-old male presents to the clinic with a history of productive cough for the past 3 days. Which of the following factors if present best predicts the diagnosis of pneumonia in this patient? A Adenopathy B Double sickening C Dyspnea D Fever E Hoarseness

Fever Cough, fever (D) and sputum production are classic symptoms of pneumonia. Adenopathy (A) and hoarseness (E) may coexist in some cases of viral pneumonia. Dyspnea (C) may develop as pneumonia progresses. Double sickening (B) is a clinical consideration in the differentiation of acute sinusitis from upper respiratory infection that typically occurs after approximately 7 days of illness.

A 68-year-old African-American woman with a past medical history of obstructive sleep apnea, hypertension, and COPD presents with chronic, progressive dyspnea, which occurred upon exertion initially, but is now noted at rest over the last 8 months. She is maintained on home oxygen for COPD. There is associated fatigue, substernal exertional chest pain, and 2 episodes of syncope, which has occurred during exertion. She denies other symptoms. Physical exam reveals an oxygen saturation of 90%, a left parasternal lift, narrow splitting of the second heart sound, accentuation of the pulmonary component of the second heart sound, an early systolic ejection click, and an S4 gallop. No murmurs are identifiable. There is also +1 pitting edema noted bilaterally to the lower extremities to the mid-calf level. An EKG and chest x-ray were performed, which revealed the results shown. Question What treatment is considered beneficial in the management of this patient at this time? Answer Choices 1 Propranolol 2 Warfarin 3 Furosemide 4 Digitalis 5 Vasopressin

Furosemide This patient has a presentation most consistent with cor pulmonale. Chronic obstructive pulmonary disorder is the most common cause of secondary cor pulmonale while primary pulmonary hypertension is idiopathic. The electrocardiogram demonstrates right axis deviation, right ventricular hypertrophy, and right atrial enlargement. The chest x-ray indicates cardiac enlargement, with prominence of the pulmonary artery, right atrium, and right ventricle. Diuretics such as furosemide are used in the management of chronic cor pulmonale, particularly when the right ventricular filling volume is markedly elevated and in the management of associated peripheral edema. These agents may result in improvement of the function of both the right and left ventricles. Propranolol is a nonselective beta adrenergic receptor blocker whose use is contraindicated in patients with bronchospastic disease (such as asthma), COPD and congestive heart failure. Beta-selective agonists, not antagonists, demonstrate the advantages of bronchodilator and mucociliary clearance effect in the treatment of cor pulmonale. Warfarin is recommended in patients at high risk for thromboembolism. The beneficial role of anticoagulation in improving the symptoms and mortality in patients with primary PAH has been demonstrated in several studies. The evidence of benefit, however, has not been established in patients with secondary PAH. Therefore, anticoagulation therapy may be used in patients with cor pulmonale secondary to thromboembolic phenomena and with underlying primary PAH. Digitalis should not be used during the acute phases of respiratory insufficiency when large fluctuations in levels of hypoxia and acidosis may occur. Patients with hypoxemia or acidosis are at increased risk of developing arrhythmias due to digitalis through different mechanisms, including sympathoadrenal stimulation. Vasopressin possesses both vasoconstrictor and antidiuretic properties, both of which contribute to elevating arterial pressure. Its use is not appropriate for cor pulmonale. Rather, vasodilators have been advocated in the long-term management of chronic cor pulmonale with modest results. For instance, calcium channel blockers (particularly oral sustained-release nifedipine and diltiazem) can lower pulmonary pressures.

A 55-year-old woman presents with a slight cough. She has had the cough for about a week. She is a non-smoker, and she does not remember having a fever or feeling sick. Auscultation of the chest reveals clear lung fields. A chest X-ray shows a subpleural 'coin lesion' in the right upper lobe. What is the most likely diagnosis? Answer Choices 1 Small cell anaplastic carcinoma 2 Bronchiectasis 3 Silicosis 4 Granuloma 5 Exogenous lipid pneumonia

Granuloma The most likely diagnosis is granuloma. The differential diagnosis of a solitary coin lesion includes: Lung carcinoma (most commonly an adenocarcinoma) Granuloma Hamartoma A small cell carcinoma (or oat cell carcinoma) tends to spread very quickly and not remain localized. Moreover, it is common with a history of smoking. Silicosis results from years of inhaling dust containing silica, and it produces diffuse scattered nodules in the lungs. Bronchiectasis results from inflammation, destruction, and dilatation of the bronchi; it does not produce a round discreet lesion. It gives a typical honeycomb appearance on chest X-ray. Exogenous lipid pneumonia is due to aspiration of a substance that is oily or contains lipids. It is not typically localized.

A 29-year old patient presents with a 9-month history of recurrent hemoptysis. The patient suffered from cavitary tuberculous infection 5 years ago and was effectively treated with antituberculous drugs. A thin-walled cavity was seen over the right upper lobe when first diagnosed. At a later date, a progressive opacification of tuberculous cavity was seen. What organism is most likely responsible for such radiologic changes of a tuberculous cavity? Answer Choices 1 Secondary growth of staphylococci 2 Growth of Candida albicans might have filled the cavity 3 Growth of Aspergillus fumigatus might have filled the cavity 4 The cavity filled with granulation tissue and subsequent fibrosis 5 The cavity filled with desquamated epithelial cells of alveoli

Growth of Aspergillus fumigatus might have filled the cavity One of the complications of tuberculous cavity is the growth of Aspergillus fumigatus within the cavity (fungal ball). Diagnosis is made by isolation of Aspergillus hyphae in the sputum. IgG antibody against Aspergillus can be demonstrated in such patients.

A 76 year-old woman with steroid dependent chronic obstructive pulmonary disease is hospitalized with fever, chills, and a productive cough. The sputum gram stain shows many WBCs and small, pleomorphic gram-negative rods. Which of the following is the most likely causative agent? A Chlamydia pneumoniae B Haemophilus influenzae C Mycoplasma pneumoniae D Staph aureus E Strep pneumoniae

Haemophilus influenzae Haemophilus influenzae (B) is a gram-negative pleomorphic coccobacillus. Strep pneumonia (E) and Staph aureus (D) are gram positive organisms. Mycoplasma pneumonia (C) and Chlamydia pneumoniae (A) aren't visible on gram stain.

A 43-year-old male farmer from the Southwest United States has been working in a very contaminated barn with rodent feces for the last week. He presents to your office with complaints of fever, non-productive cough, malaise, and decreased appetite. His physical exam reveals a temperature of 102 0 F, pulse rate of 98, blood pressure of 98/62, and O 2 saturation of 93%. Lung sounds have diffuse crackles throughout, and the rest of the exam is unremarkable. Based on the history and exam findings, what is the most likely pathogen for this type of illness? A Influenza pneumonia B Varicella pneumonia C Hantavirus pneumonia D Streptococcal pneumoniae E Cytomegalovirus

Hantavirus The history of the patient being exposed to the rodent feces is a typical presentation of a pneumonia caused by the hantavirus. There is no treatment for this type of pneumonia, only supportive care.

A 55-year-old female presents with a several-month history of increasing cough and dyspnea. She also has increased serum urea, nitrogen, and serum creatinine. A chest X-ray shows multiple bilateral small nodules. A renal biopsy shows a focal necrotizing vasculitis; her antineutrophil cytoplasmic autoantibody (ANCA) test is positive at 1:160. What additional finding would be most likely to occur? Answer Choices 1 Angina 2 Hemorrhagic pericarditis 3 Endocarditis 4 Hemoptysis 5 Hemothorax

Hemoptysis The history of upper airway involvement (cough with dyspnea), the nodular pattern on X-ray, and the renal biopsy showing focal necrotizing vasculitis suggests the diagnosis of Wegener's granulomatosis. Pulmonary (as well as upper respiratory tract) involvement with Wegener's granulomatosis commonly leads to hemorrhage, which manifests as hemoptysis. Although the etiology is unknown, it is believed to represent an antigen-triggered immunologic reaction. Pulmonary manifestations include cough, dyspnea, chest discomfort, and hemoptysis. In most cases, the extrarenal disease precedes the onset of renal disease. Although the heart may be involved with Wegener's granulomatosis, it does not usually lead to symptoms of ischemia (angina). The vasculitis does not typically involve the pericardium extensively; therefore, hemorrhagic pericarditis would not be a finding. The renal failure could lead to a fibrinous pericarditis. Endocarditis is not a feature of Wegener's granulomatosis. The vasculitis of Wegener's granulomatosis involves very small arteries and capillaries. Hemothorax is more typical of trauma.

A 58-year-old woman with a past medical history of hypertension, hyperlipidemia, breast cancer, hip fractures, and coronary artery disease is being evaluated for acute-onset, severe left-sided pleuritic chest pain over the course of the last 2 hours. The pain is associated with feelings of anxiety, hemoptysis, shortness of breath, and nausea. She "feels warm" but denies chills, palpitations, wheezing, cough, edema, vomiting, abdominal pain, abnormal bowel habits, or dietary intolerances. She endorses a 30 pack/year smoking history, but denies drug or alcohol use. Upon physical exam, she is found to be febrile, hypotensive, tachycardic, tachypnic, diaphoretic, and in acute painful distress. There is perioral cyanosis and a pleural friction rub to the left lung fields; the remainder of the exam is normal. Question What is the most appropriate therapeutic intervention for this patient at this time? Answer Choices 1 Indomethacin 2 Doxycycline 3 Heparin 4 Prednisolone 5 Albuterol

Heparin This patient's presentation is significant for a pulmonary embolism. Predisposing underlying conditions are almost always present; venous thrombosis may result from a generalized hypercoagulable state, venous endothelial injury, or local stasis (Virchow triad). Most commonly, the initial manifestations of pulmonary embolism include an abrupt dyspnea and chest pain. Tachycardia and hypoxia are the most common clinical signs. Associated manifestations include fever, hypotension, cyanosis, pleural friction rub, and findings consistent with pulmonary consolidation. Anticoagulation is the foundation for successful treatment of DVT and PE. Unfractionated heparin, low-molecular-weight heparin, or fondaparinux all achieve effective anticoagulation immediately.

Which of the following is the causative agent in patients with Fifth disease? A herpesvirus 6 (HHV-6) B human parvovirus B19 C paramyxovirus D varicella-zoster virus (VZV) E none of the above

Human parvovirus B19 The disease is caused by parvovirus B19. It appears sporadically, but often in epidemics in communities. Children are infectious during the prodromal stage, which is unapparent or mild and usually indistinguishable from an upper respiratory infection. The rash is an immune-mediated phenomenon that occurs after the infection, so children with the rash are not infectious and should not be restricted from school or other activities. Aka slapped cheeks disease

A 28-week-premature infant is noted to have increasing tachypnea and difficulty breathing, with diffusely decreased breath sounds on exam shortly after delivery. A chest x-ray reveals diffuse, bilateral atelectasis and air bronchograms. Which of the following is the most likely diagnosis? A Acute asthma exacerbation B Hyaline membrane disease C Meconium aspiration D Pleural effusion E Spontaneous pneumothorax

Hyaline membrane disease Premature infants, due to a lack of surfactant, develop marked atelectasis and decreased lung compliance, with acute respiratory distress. This is termed hyaline membrane disease and is the most common cause of respiratory distress in preterm infants. Meconium aspiration is more likely in full-term or near-term infants who experience fetal distress, and the x-ray may demonstrate hyperexpansion and irregular infiltrates. Spontaneous pneumothorax can occur at birth. Exam findings will include decreased breath sounds on the affected side, and x-ray findings should indicate the pneumothorax or pneumomediastinum. Pleural effusion may be present in hydropic infants, or it may be due to an underlying disorder or chylothorax. The x-ray would reveal opacity of the affected side, with blunting of the costophrenic recess. Asthma exacerbation should involve airway hyperresponsiveness in relation to a trigger exposure, and would demonstrate bilateral hyperinflation with diaphragm flattening on x-ray.

A 64-year-old man presents with progressive dyspnea, fatigue, chronic dry cough, and exercise intolerance. His symptoms have been progressing over the past several months to a year. As part of your work up, you obtain pulmonary function testing; it reveals a FEV1/FVC ratio of >0.7, a decreased total lung capacity, and a decreased residual volume. Question What is the most likely diagnosis? Answer Choices 1 Anemia 2 Asthma 3 Chronic obstructive pulmonary disease 4 Pulmonary embolism 5 Idiopathic pulmonary fibrosis

Idiopathic Pulmonary Fibrosis This patient in this case presents with dyspnea, fatigue, dry cough, and exercise intolerance, which are fairly common symptoms. The clue to making this diagnosis lies in his pulmonary function tests (PFT's). The findings of an increased FEV1/FVC ratio with a decreased total lung capacity and residual volume is indicative of a restrictive pathology.1 Idiopathic pulmonary fibrosis (IPF) is a slowly progressive fibrosing lung disease; it commonly affects people 50 - 70 years old, and there is a slight male predominance.1 In addition, physical exam may reveal late inspiratory crackles and digital clubbing.2 Anemia can cause similar symptoms, but this is due to decreased circulation of oxygenated blood and not a result of an underlying lung disorder; therefore, there will be no abnormality in PFT's. Both asthma and chronic obstructive pulmonary disease (COPD) are obstructive pathologies. PFT's would show an obstructive pattern, with an FEV1/FVC ratio of <0.7 as well as an increased total lung capacity and residual volume.1 Pulmonary embolism is an unlikely diagnosis in a patient who has been symptomatic for several months to a year.

On Thanksgiving day, a 5-month-old infant presents with wheezing, rapid respirations (>45 breaths/min), and chest retractions. The patient has a 2-day history of rhinorrhea and low-grade fever. Breath sounds are normal, and there is no cyanosis. What test can confirm the most likely diagnosis? Answer Choices 1 Chest X-ray 2 Immunofluorescence of nasal secretion 3 Gram stain of the sputum 4 Blood gas analysis 5 White blood cell count and differential

Immunofluorescence of nasal secretion The correct answer is Immunofluorescence of nasal secretion. The clinical presentation, time of year, and age of the child strongly suggest bronchiolitis caused by respiratory syncytial virus. A chest X-ray may indicate interstitial pneumonia, but it can be relatively normal. Gram stain is needed in case of suspected bacterial infection, and sputum usually is difficult to obtain from children. Blood gas analysis may detect abnormalities when cyanosis is absent, but it cannot confirm a specific diagnosis. The white blood cell count and differential are usually normal; however, immunofluorescence of nasal secretion can lead to rapid viral identification.

A 57-year-old man, who is well known to your practice, presents with a 6-month history of a daily productive cough. The patient is a nonsmoker, and he has worked in a local coal mine for the past 39 years. He very rarely comes in to see a healthcare provider. When asked if he is up to date with his vaccinations, he does not recall the last ones he received; he also does not recall when these may have been given. Question Given the most likely diagnosis at this time, what vaccinations would be recommended to help decrease any significant morbidity and mortality in this patient? Answer Choices 1 Influenzae and pneumococci 2 Influenzae and zoster 3 Varicella and pneumococci 4 Zoster and varicella 5 Influenzae

Influenzae and pneumococci It is very likely that the patient described above has an occupation pulmonary disease process, most likely coal worker's pneumoconiosis, or black lung disease. Most patients that develop this disease tend to have worked around coal dust for several years, if not decades. For this reason it is commonly diagnosed in patients who are over the age of 50. Smoking does not increase the risk of developing pneumoconiosis. The chronic ingestion of inhaled coal dust by macrophages in the alveoli leads to the formation of coal macules, usually 2 - 5 mm in diameter. These areas then appear on a chest radiograph as opacities, especially in the upper lung. Simple coal work's pneumoconiosis commonly is asymptomatic; at times a cough or even varying degrees of shortness of breath may be present. The patient will typically not have any significant abnormal pulmonary function results. More severe cases of this pathology are usually referred to as progressive massive fibrosis. There is no cure for coal worker's pneumoconiosis; therefore, prevention of excessive and long-term exposure is necessary. Once a patient has developed symptoms of this disease, such as the patient above, the healthcare provider must ensure the patient is up to date with all potential vaccinations to avoid any pulmonary co-morbidities. This should be viewed as a critical component of health maintenance. The influenzae and the Streptococcivaccines would be crucial for this patient to receive; consequences of these infections could be extremely dangerous. A zoster vaccination may be considered, but would not necessary be the best answer. This patient receiving a varicella vaccine would not be recommended at this time.

You are currently on an inpatient hospitalist team in a local pediatric hospital. First thing this morning, your team is called in to evaluate an infant born at 27 weeks gestation 50 minutes ago. Upon initial inspection of the newborn, you observe rapid, labored grunting respirations, flaring nostrils, and retractions that are present above and below the breastbone. Auscultation reveals diminished air movement, and a chest radiograph reveals a ground glass appearance in the lung fields bilaterally. The patient is diagnosed with respiratory distress syndrome (RDS). Question What pharmacological agent should be initiated as soon as possible to help alleviate the patient's signs and symptoms? Answer Choices 1 Intravenous antivirals 2 Hepatitis B immunoglobulin 3 Intravenous antibiotic 4 Inhaled surfactant replacement 5 Inhaled corticosteroids

Inhaled surfactant replacement Respiratory distress syndrome (RDS) is also known as hyaline membrane disease (HMD) and is a condition that causes infants to need extra oxygen and assistance in breathing. It is one of the most common problems seen in premature infants; the more premature the baby, the higher the risk and the more severe the HMD. Many times HMD typically worsens over the first 48 to 72 hours after birth and then improves with treatment; more than 90 percent of babies with HMD survive. HMD occurs when there is not enough surfactant in the newborn's lungs. Surfactant is made by the cells in the airways and consists of phospholipids and protein. It begins to be produced in the fetus at about 24 to 28 weeks of pregnancy and is found in amniotic fluid between 28 and 32 weeks. By about 35 week's gestation, most babies have developed adequate amounts of surfactant. In healthy lungs, surfactant is released into the lung tissues to help lower surface tension in the airways and this helps keep the lung alveoli open. When there is not enough surfactant, the tiny alveoli collapse with each breath. As the alveoli collapse, damaged cells collect in the airways and this makes it even harder to breath. The newborn works harder and harder to breath with each breath, trying to re-inflate the collapsed airways. This vicious cycle can eventually lead to a build-up of carbon dioxide, eventual acidosis, and/or eventual physical exhaustion from trying to attempt to take breaths. The most common symptoms of HMD include difficulty breathing at birth that gets progressively worse, cyanosis, flaring of the nostrils, tachypnea, grunting sounds, and chest retractions. Chest radiographs will reveal a characteristic "ground glass" appearance". Treatment of HMD consists of placing an endotracheal tube, supplemental oxygen, continuous positive airway pressure (CPAP), and inhaled surfactant replacement with artificial surfactant. This treatment has been shown to reduce the severity of HMD and is most effective if started within the first 6 hours of birth. Artificial surfactant comes as a powder that is mixed with sterile water and given through the ET tube. RDS stemming from HMD is not a viral or bacterial infection; therefore, intravenous antivirals or antibiotics would be an inappropriate choice. This also is not specifically an issue with Hepatitis B infection, therefore, the Hepatitis B immunoglobulin answer is incorrect. Inhaled corticosteroids may give some minor symptomatic relief, but they are not an indicated or efficient treatment for patients with hyaline membrane disease or respiratory distress syndrome.

A 43 year-old woman with a past medical history of HIV infection presents to the clinic due to a 3-month history of dry, non-productive cough and progressive dyspnea. The patient is subsequently prescribed trimethoprim-sulfamethazole. What is the mechanism of action of this sulfamethazole? A Binds to the 30S ribosomal subunit to inhibit protein synthesis B Binds to the 50S ribosomal subunit to inhibit protein synthesis C Inhibits dihydropteroate synthase and folate production D Inhibits DNA replication by binding to DNA gyrase and topoisomerase IV E Inhibits the transpeptidation reaction

Inhibits dihydropteroate synthase and folate production Sulfamethoxazole inhibits dihydropteroate synthase and folate production (C). Tetracycline binds to the 30S ribosomal subunit to inhibit protein synthesis (A). Macrolide antibiotics bind to the 50S ribosomal subunit to inhibit protein synthesis (B). Ciprofloxacin inhibits DNA replication by binding to DNA gyrase and topoisomerase IV (D). Beta lactam antibiotics inhibit the transpeptidation reaction leading to cell wall destruction (E).

A 23 year-old male with cystic fibrosis inquires about the availability of treatments that can help improve his lung function. Which of the following treatments is most effective at reversing the pulmonary effects of cystic fibrosis? A Albuterol B Azithromycin C Inhaled hypertonic saline D Inhaled levofloxacin E Ivacaftor

Ivacaftor Ivacaftor (E) is the only treatment that restores function of the CFTR protein in cystic fibrosis patients with a G551D mutation thereby reversing the effects of the disease, approximately 5% of all cystic fibrosis patients have the G551D mutation. Albuterol (A) and hypertonic saline (C) are indicated to improve lung function and mucous clearance. Azithromycin (B) and inhaled levofloxacin (D) are used to treat chronic infection/colonization with pseudomonous.

What exposure is linked to Listerosis

Listeriosis has been linked to exposures to contaminated food, particularly dairy products and hot dogs.

An 8-month-old infant presents with shortness of breath, wheezing, intercostal retractions, and respiratory rate of 50. What would further suggest a diagnosis of bronchiolitis? Answer Choices 1 Wheezing improved after nebulizer 2 Peak flow 300 prior to treatment 3 Liver and spleen palpable 4 Clubbing of the finger

Liver and spleen palpable Enlargement of the liver and spleen occurs due to hyperinflation of the lungs with bronchiolitis. With this diagnosis, a nebulizer that improves wheezing is not a specific criterion, but it is more so with asthma. A peak flow of 300 is good; therefore, it would likely be lower with bronchiolitis. Clubbing of the fingers is seen with prolonged COPD in adults, not in the pediatric population.

Which of the following is the most prevalent fatal cancer in the United States? A Cervical B Colon C Esophageal D Liver E Lung

Lung Lung cancer continues to lead as the major cause of cancer deaths in both men and women in the United States, with cigarette smoking causing greater than 90% of cases. Despite educational campaigns highlighting the risks of smoking, lung cancer continues to kill more individuals that colorectal, breast, and prostate cancers combined.

A 68-year-old man presents to your pulmonology practice for long-standing dyspnea and non-productive cough. The patient has had the cough and progressively worsening dyspnea for about 1.5 years. He denies other symptoms, and he is not a smoker. He denies fevers, chills, and night sweats. He denies unusual travel, hobbies, and occupational exposures. The rest of his review of systems is negative. His past medical history is unremarkable. He had a workup through his family practice, and he was then sent to a cardiologist, who ruled out cardiovascular causes of his dyspnea. He was given trials of various antibiotics, inhalers, and steroids; there was no improvement in symptoms, despite good compliance. Several screening blood tests (including blood count, metabolic panel, HIV, and autoimmune markers) are done, revealing no abnormalities. A chest X-ray shows a few reticular opacities. A high-resolution computed tomography (HSCT) scan shows some scattered areas of reticular opacities and honeycombing. Your office runs a pulmonary function test (PFT), which shows some restrictive impairment and reduced perfusion of carbon monoxide. Physical exam is significant for fine inspiratory bibasilar crackles and clubbing in the fingers. An occasional dry cough is noted. Question What intervention is recommended for this patient's condition? Answer Choices 1 Avoidance of exertion 2 Daily inhaled steroids 3 Long-term oral steroid use 4 Lung transplant 5 Ventilation-perfusion scan

Lung Transplant This patient is presenting with idiopathic pulmonary fibrosis (IPF), and the only intervention currently associated with improvements in mortality is lung transplant. IPF is a chronic, progressive restrictive pulmonary disease of the lung parenchyma. IPF presents with exertional dyspnea and non-productive cough; it also presents with the imaging characteristics described in this patient. Avoidance of exertion may prevent the exertional dyspnea in this patient, but it is not recommended. Respiratory therapy to improve exercise tolerance is recommended in cases of IPF. Daily inhaled steroids and long-term oral steroid use are not proven as effective in IPF. The pathophysiologic basis for IPF is much more a disorder of fibroblastic disease, in which there is an abnormal accumulation of extracellular matrix, and not inflammatory in nature. Anti-inflammatory medications do not alter the course of IPF. A ventilation-perfusion scan is a test that is primarily used in the diagnosis of pulmonary embolism (PE). This patient is not presenting with a history consistent with a PE (which may be acute dyspnea and chest pain). The imaging that has already been done (high-resolution CT) is sufficient for diagnosis of IPF. No further studies are recommended. If the provider needs further confirmation, a lung biopsy should be the confirmatory test of choice.

A 69-year-old man presents with a 7 - 10 day history of increasing dyspnea and inspiratory chest pain. He has a 2-pack-per-day smoking history and abuses alcohol. He has hypertension, diabetes, coronary artery disease, and chronic kidney disease. Upon further questioning, you discover that he was discharged from the hospital 2.5 weeks ago; he was diagnosed with congestive heart failure after presenting with similar symptoms. The patient's breathing appears labored, and you note diminished breath sounds on auscultation and dullness to percussion in the lower half of the lung fields bilaterally. A chest X-ray reveals bilateral moderate pleural effusions. Thoracentesis reveals pleural fluid with the following characteristics: (1) turbid in appearance; (2) 2500 white blood cells/microliter; (3) glucose equal to serum levels; (4) ratio of pleural fluid protein to serum protein of 0.75; and (5) ratio of pleural fluid LDH to serum LDH of 0.72. Question Based on the information above, what is the most likely etiology of the pleural effusions? Answer Choices 1 Congestive Heart Failure 2 Nephrotic Syndrome 3 Cirrhosis 4 Malignancy 5 Bacterial pneumonia

Malignancy is the correct response. A pleural effusion is an abnormal accumulation of fluid in the pleural space. Patients with pleural effusions most often report dyspnea, cough, and pleuritic chest pain. Large pleural effusions are more likely to be symptomatic than smaller effusions. Physical findings are usually absent in small effusions. Larger effusions may present with dullness to percussion and diminished or absent breath sounds over the affected area. A diagnostic thoracentesis should be performed whenever there is a new pleural effusion and no clinically apparent cause, when there is an atypical presentation, or when an effusion fails to resolve as expected. Sampling allows visualization of the fluid in addition to chemical and microbiologic analyses to help identify the etiology of the effusion. Pleural samples should be sent for measurement of protein, glucose, and LDH, in addition to total and differential white blood cell counts. These chemistry tests are used to classify effusions as a transudate or an exudate. This distinction is important because the differential diagnosis for each is entirely different. A pleural exudate is an effusion that has 1 or more of the following laboratory features: (1) ratio of pleural fluid protein to serum protein > 0.5; (2) ratio of pleural fluid LDH to serum LDH > 0.6; and (3) pleural fluid LDH greater than 2/3 the upper limit of normal serum LDH. Additionally, samples with low glucose levels usually indicate a bacterial or significant inflammatory etiology. In contrast, transudative effusions have none of these features. Transudates are also distinguished by fewer than 1000 white blood cells/microliter and a pleural glucose level equal to serum. These types of effusions occur in the setting of normal capillary integrity and also suggest the absence of local pleural disease. The characteristics of the pleural fluid from the patient in the clinical scenario presented above indicate an exudative pleural effusion. Malignancy is the most likely etiology in this patient, as a bacterial cause would typically present with a decreased glucose level in the pleural fluid compared with the serum; there is an equal glucose level in this case. Congestive heart failure accounts for >90% of transudative pleural effusions. Nephrotic syndrome and cirrhosis with ascites can also lead to transudative pleural effusions. Bacterial pneumonia and cancer are the most common causes of exudative effusions.

A 65-year-old man presents with fatigue, shortness of breath, chest pain, and weight loss. Physical exam findings include diminished breath sounds on auscultation, and there is dullness to percussion. You order a chest X-ray; it reveals a pleural-based mass. What type of lung cancer is most likely? Answer Choices 1 Adenocarcinoma 2 Squamous cell carcinoma 3 Small cell carcinoma 4 Large cell carcinoma 5 Mesothelioma

Mesothelioma The clinical picture is suggestive of a mesothelioma lung cancer. The mean age at onset of symptoms is 60 years old with a history of asbestos exposure. Common findings include weight loss, shortness of breath, fatigue, and chest pain. Other findings include diminished breath sounds, digital clubbing, and dullness to percussion. A key feature is a pleural-based mass or thickening of the pleura on imaging. Adenocarcinomas usually present as peripheral nodules or masses on imaging. Squamous cell carcinoma usually presents as central masses on imaging and are more likely to present with hemoptysis. Small cell carcinomas typically begin centrally with concomitant hilar and mediastinal abnormalities present. Large cell carcinomas on imaging present as central or peripheral masses, but they do not usually involve the pleura.

A 50-year-old woman presents with right-sided pleural effusion. Thoracentesis shows the presence of exudative serosanguineous pleural fluid and positive cytology. For what condition is this finding most typical? Answer Choices 1 Metastatic infiltrating ductal carcinoma 2 Cor pulmonale 3 Systemic lupus erythematosus 4 Staphylococcus aureus septicemia 5 Pulmonary infarction

Metastatic infiltrating ductal carcinoma A blood-tinged effusion in anyone past the age of 40 should put neoplasia into the differential diagnosis. Neoplastic pleural invasion usually involves both the visceral and the parietal surfaces and can cause a bloody pleural effusion. Breast cancers often metastasize to the lung and pleura. Carcinomas of the lung and breast are the most common primary sites from which pleural metastases arise. Stomach and ovarian carcinomas are next in frequency for the development of malignant pleural effusion. Spread of malignancy to the pleura from breast, stomach, and ovarian cancers, usually occurring indirectly from hepatic metastases. However, contiguous spread through the chest wall in breast cancer, and through the diaphragm in ovarian or stomach cancer, can occur occasionally. When malignancy is suspected, cytological examination of the fluid helps in establishing the diagnosis. Pleural biopsy, or pleural fluid immunocytochemistry, is used as an adjunct to cell morphology to aid diagnosis. Cor pulmonale results in peripheral edema including pleural effusion. These are usually serous effusions, which is transudative in nature. Systemic lupus erythematosus (SLE) can be associated with exudative pleural effusions, but they are usually serous and present bilaterally. Bacterial sepsisdue to S. aureus could lead to pleuritis with purulent exudates. Pulmonary embolism (a common cause of pulmonary infarction) results in bloody pleural effusions; however, the cytological examination will be negative.

A 4 year-old girl presents to the clinic due to severe allergic rhinitis and recurrent asthma exacerbations. Which of the following leukotriene pathway inhibitors is indicated for use in this child? A Beclomethasone B Montelukast C Salmeterol D Zafirlukast E Zileuton

Montelukast Montelukast (B) is indicated for use in children 1 year of age or older, zafirlukast (D) is indicated for children > 5 years of age, and zileuton (E) is indicated for children > 12 years of age. Beclomethasone (A) is an inhaled corticosteroid and salmeterol (C) is an inhaled long-acting beta-2 agonist.

A 22 year-old male presents with a 2-week history of an upper respiratory infection that hasn't improved after taking amoxicillin for 6 days. He notes persistent sore throat, intermittent fever, and a worsening nonproductive cough. Physical examination reveals bilateral diffuse crackles. What is the most likely causative agent of this patient's symptoms? A Streptococcal pneumoniae B Klebsiella pneumoniae C Mycoplasma pneumoniae D Pneumocystis jiroveci E Staphlococcal aureus

Mycoplasma pneumoniae C The patient presentation is consistent with atypical pneumonia most commonly caused by viruses, mycoplasma pneumonia (C), chlamydia pneumonia, or legionella. The presentation of streptococcal pneumoniae (A) is more acute with productive cough and fever being early symptoms. Klebsiella pneumoniae (B) also presents with fever and a cough classically noted to be productive of "currant jelly" sputum. Pneumocystis jiroveci (D) causes pneumonia of insidious onset in immunocompromised patients. Staphlococcal (E) pneumonia also presents more acutely with productive cough.

A 4-year-old male has been experiencing a significant cough for the last 12 to 14 days, and initial episodes of coughing are characterized as frequent outbursts of 5 to 10 spastic coughs in a row. The patient does not report any fever, but does note that the coughing is worse at night. On examination, the patient is alert, awake, and oriented. His temperature is 97.7 0 F, pulse rate is 89, respiratory rate is 25, and blood pressure is 110/56. The HEENT is unremarkable, and lung sounds are clear to auscultation. You suspect that the patient may have an acute case of pertussis. Based on the history and physical exam findings, which is the test of choice for confirming a diagnosis of pertussis? A Complete blood count (CBC) B Throat culture C Chest x-ray D Nasopharyngeal culture E Sputum gram stain and culture

Nasopharyngeal culture A special medium culture plate (such as a Bordet-Gengou agar) is required for the nasopharyngeal swab for the diagnosis of pertussis. Throat culture, chest x-rays, and complete blood counts are helpful in ruling out other disease patterns.

A 4-year-old girl, adequately immunized for her age, has a history of repeated pyogenic infections. She had normal antibody responses following childhood immunizations. A defect in what cells best explains the cause of her infections? Answer Choices 1 B-lymphocytes 2 T lymphocytes 3 Eosinophils 4 Macrophages 5 Neutrophils 6 Basophils

Neutrophils Pyogenic infections are generally caused by bacteria. The entry of bacteria into the body triggers a number of responses: activation of the complement pathway and phagocytosis by neutrophils and macrophages. Macrophages play a major role in chronic granulomatous infections, and neutrophils predominate in acute pyogenic infections. An adequate response to childhood immunizations indicates an adequate capability of antibody production, and recovery from viral infections like measles indicates appropriate cell-mediated immunity. Therefore, the defect must lie in the neutrophils.

What is the most common cause of treatment failure in tuberculosis? A Drug resistance B Noncompliance to therapy C Inappropriate selection of medication D Sepsis

Noncompliance to therapy The usual reason for failure is simply due to the patient not continuning their treatment plan, regardless of the severity of the disease. Drug resistence, while present in some cases does not preclude the patient from treatment failure. Ongoing sepsis is not a reason to have treatment failure.

A 66-year-old man presents with vomiting. He has lung cancer, but he has declined treatment. His father died of lung cancer at the age of 65. On examination, he is confused and dehydrated with a BP of 100/50 mm Hg and PR of 100 beats/minute. Laboratory analysis reveals a calcium level of 16 mg/dL. His renal function tests are normal. Question What is the most appropriate initial therapy in the management of this patient? Answer Choices 1 Normal saline 2 Prednisone 3 Calcitonin 4 Hemodialysis 5 Plicamycin

Normal Saline The clinical presentation is suggestive of a hypercalcemia of malignancy. It is most frequently associated with multiple myeloma, breast, lung, and kidney cancers, as well as T-cell lymphomas. The hypercalcemia is usually a result of increased bone resorption by osteoclasts. Patients present with anorexia, nausea, vomiting, constipation, fatigue, and headache. When the plasma calcium level rises above 12 mg/dL, they may also present with confusion, delirium, and coma. Laboratory results may also reveal an elevated blood urea nitrogen and creatinine levels. The 1st step in the management of severe hypercalcemia of malignancy, when the renal function is normal, is aggressive rehydration with intravenous normal saline while closely monitoring urine output and electrolytes. The bisphosphonate pamidronate, which acts as a bone resorption inhibitor, can then be given intravenously in addition to the normal saline. Calcitonin, another bone resorption inhibitor, is also used to manage severe hypercalcemia of malignancy when it is combined with prednisone; however, some patients are unresponsive to this combination. Hemodialysis, along with a calcium-free dialysate and peritoneal dialysis, is also used to lower the calcium levels (especially in patients with renal failure). Plicamycin, which inhibits bone resorption, is also effective in lowering plasma calcium; however, due to its potentially severe adverse effects, it should be given if there is insufficient response to the other medications.

A 3-year-old boy presents for his annual well child check. He and his family moved to the United States from Africa 4 months ago. He is a thin boy; he is in no acute distress. His heart rate and rhythm are regular; his lungs are clear, and he has no hepatosplenomegaly. Due to his recent immigration, he is given a tuberculin skin test. He has no known chronic medical conditions and no known contacts with tuberculosis disease. The test shows an induration of 6 mm. Question What is the next step in the management of this patient? Answer Choices 1 Chest X-ray and isoniazid 2 Isoniazid, rifampin, pyrazinamide, and ethambutol 3 Hospitalization for further workup 4 Normal well child care 5 Gastric aspirate sampling

Normal child well care visit For a child who has immigrated from a high-prevalence area of the world but has no close contacts with tuberculosis disease, is not immunocompromised, and has no signs or symptoms of the disease, the induration must be 10 mm or greater to be positive. Therefore, this child has a negative test and should receive normal well child care. If the test were positive, he would require a chest X-ray and further workup. If he were found to have latent infection without active disease, he would be treated with isoniazid alone. If he were found to have a pulmonary or extrapulmonary disease, except meningitis, he would be treated with isoniazid, rifampin, pyrazinamide, and ethambutol. Gastric aspirate sampling is one way of isolating the organism if a patient has active tuberculosis disease.

A 26 year-old man presents to the clinic after developing a severe productive cough, fever, chills, night sweats and weight loss. Which is the most definitive diagnostic test for determining the presence of Mycobacterium tuberculosis as the causative agent of his active pulmonary tuberculosis and yields results in less than 24 hours? A Acid-fast bacilli smear of sputum B Blood culture C Mycobacterial culture of sputum D Nucleic acid amplification of sputum E Tuberculin skin test

Nucleic acid amplification of sputum The most definitive diagnostic tests for confirmation of tuberculosis are mycobacterial culture (C) which requires at least 2 weeks to grow mycobacterium, and nucleic acid amplification (D) of a respiratory specimen that can be completed in a matter of hours. AFB smear (A) has a sensitivity of only 40-60%, but positive results help establish a presumptive diagnosis. Tuberculin skin testing (D) is effective in screening patients for latent TB. Routine blood cultures (B) are ineffective in diagnosing TB.

A 34-year-old male has a one and one-half day history of fever, chills, a non-productive cough, and malaise. He is otherwise healthy with no long-standing medical history, and is taking no chronic medications. On examination, the patient has a temperature of 101.3 0 F, BP 123/63, P 78, R 18. His HEENT reveals mild rhinorrhea, moist mucous membranes, clear lung sounds, and a regular rate and rhythm. The rapid nasal viral test for influenza B is positive. Based on this information, what is the medication treatment for this patient? A Amantadine B Oseltamavir C Ramantadine D Famciclovir E Azithromycin

Oseltamavir Oseltamivir is the best antiviral medication for the treatment of acute influenza. This medication is ideally started within the first onset of illness, usually within the first 24 to 36 hours. Amantadine and Ramantidine have been shown to not be effective, and there is growing resistance to the medication.

A 45-year-old male with asthma and diabetes is diagnosed with influenza B by nasal swab. He has been ill for one and a half days. Which of the following is indicated for treating this patient? A Acyclovir B Amantadine C Nevirapine D Oseltamivir E Zanamivir

Oseltamavir The neuraminidase inhibitors, including oseltamivir and zanamivir, are associated with a reduction in duration of illness, and secondary complications for both influenza A and B viral strains. However, zanamivir, due to the oral inhalation delivery route, is relatively contraindicated in this patient, due to his history of asthma and an associated increased risk of bronchospasm. Amantadine is inactive against influenza B, as well as certain influenza A strains. Nevirapine is an antiretroviral agent used in the treatment regimen for HIV. Acyclovir is an antiviral agent, but is not indicated for influenza.

A woman is diagnosed with primary tuberculosis. She is 5 months pregnant. What drug should be avoided in this patient? Answer Choices 1 Isoniazid 2 Rifampin 3 Ethambutol 4 Streptomycin 5 Pyrazinamide

Streptomycin Tuberculosis in pregnancy is treated with an initial regimen of isoniazid, rifampin, and ethambutol. Because the teratogenicity of pyrazinamide is not determined, pyrazinamide is added only if a resistance to the other drugs is documented/suspected, and susceptibility to pyrazinamide is likely. Streptomycin is contraindicated in pregnancy because it may cause congenital defects. Isoniazid, rifampin, and ethambutol are considered safe to use during pregnancy.

A 72-year-old man is evaluated at the bedside following hospital admission for a 1-year history of progressive dyspnea, nonproductive cough, weight loss, low-grade fevers, fatigue, and myalgias. His past medical history is remarkable for atrial fibrillation, for which he takes amiodarone, hypercholesterolemia, and recurrent urinary tract infections, for which his urologist prescribed nitrofurantoin on a chronic, prophylactic basis. He denies any cigarette use, history of murmurs or coronary artery disease, chills, fatigue, rhinitis, otalgia, chest pain, wheezing, hemoptysis, syncope, abdominal pain, rashes, peripheral edema, diaphoresis, arthralgias, vomiting, or urinary complaints. Arterial blood gas analysis demonstrated a PaO2 of 50 mmHg and pulse oximetry of SpO2 of 87%; bedside echocardiogram and electrocardiograms are unremarkable for abnormalities; a chest x-ray revealed peripheral reticular opacities at the lung bases and a generalized honeycombing pattern. Question What treatment is the most beneficial in the management of this patient? Answer Choices 1 Prednisone 2 Colchicine 3 Oxygen 4 N-acetylcysteine 5 Bosentan

Oxygen This patient's most likely diagnosis is idiopathic pulmonary fibrosis (IPF). It is defined as a specific form of chronic, progressive fibrosing interstitial pneumonia of unknown cause, primarily occurring in older adults, limited to the lungs, and associated with the histopathologic and/or radiologic pattern of usual interstitial pneumonia (UIP). Patients with hypoxemia (PaO2 < 55 mmHg or oxygen saturation as measured using pulse oximetry [SpO2] < 88%) at rest or with exercise should be prescribed oxygen therapy to maintain a saturation of at least 90% at rest, with sleep, and with exertion. Corticosteroids have not been evaluated in a randomized, placebo-controlled trial to determine their benefit in treating patients with idiopathic pulmonary fibrosis. Retrospective uncontrolled studies have reported no survival benefits. Evidence-based guidelines recommend that patients with idiopathic pulmonary fibrosis should not be treated with corticosteroid monotherapy. Colchicine has been shown to inhibit fibroblast proliferation and collagen synthesis in vitro; however, evidence-based guidelines recommend that patients with idiopathic pulmonary fibrosis should not be treated with colchicine. Evidence-based guidelines recommend that the majority of patients with IPF should not be treated with N-acetylcysteine monotherapy. Bosentan is an endothelin receptor A and B antagonist that is approved for the treatment of pulmonary hypertension. While bosentan has been shown to have antifibrotic effects in an animal model of pulmonary fibrosis, current evidence-based guidelines recommend that patients with idiopathic pulmonary fibrosis should not be treated with bosentan.

A 69-year-old man with a past medical history of obstructive sleep apnea, hypertension, and COPD presents with dyspnea on exertion that has been slowly progressive over the course of the last year. He notes impairment in climbing stairs and walking short distances. His review of systems is positive for fatigue, palpitations, intermittent retrosternal chest pain, swelling of his lower extremities, dizziness, and "feeling faint;" his associated symptoms are also known to occur upon exertion. His cardiac exam is remarkable for an oxygen saturation of 90%, accessory muscle usage, an increased pulmonic component of the second heart sound (P2), wide, inspiratory splitting of S2 over the cardiac apex, right-sided S3 and S4 gallops, a left parasternal lift, a loud diastolic murmur that increases with inspiration and diminishes with the Valsalva maneuver, prominent "A" waves in jugular venous pulsations, and increased JVD. He has an enlarged liver with hepatojugular reflux, peripheral edema, and ascites. A bedside EKG and chest x-ray were performed, which revealed the results in the image. Question What is the most appropriate intervention in the management of this patient? Answer Choices 1 Lisinopril 2 Warfarin 3 Digoxin 4 Oxygen therapy 5 Nitroglycerin

Oxygen therapy This patient has a presentation most consistent with cor pulmonale secondary to chronic obstructive pulmonary disease (COPD). The electrocardiogram demonstrates right axis deviation, right ventricular hypertrophy, and right atrial enlargement. The chest x-ray indicates cardiac enlargement, with prominence of the pulmonary artery, right atrium, and right ventricle. Treatment should center upon the underlying contributory disorder. For patients with COPD, bronchodilation and infection treatment should be considered. Oxygen therapy is of great importance in patients with underlying COPD, especially when administered on a continuous basis. Oxygen therapy relieves hypoxemic pulmonary vasoconstriction, which then improves cardiac output, lessens sympathetic vasoconstriction, alleviates tissue hypoxemia, and improves renal perfusion. Anticoagulation with warfarin is recommended in patients at high risk for thromboembolism. The evidence of benefit has not been established in patients with secondary PAH. Therefore, anticoagulation therapy may be used in patients with cor pulmonale secondary to thromboembolic phenomena and with underlying primary pulmonary artery hypertension. The use of cardiac glycosides, such as digitalis, in patients with cor pulmonale has been controversial, and the beneficial effect of these drugs is not as obvious as in the setting of left heart failure. This drug must be used cautiously and should not be used during the acute phases of respiratory insufficiency, when large fluctuations in levels of hypoxia and acidosis may occur. Patients with hypoxemia or acidosis are at increased risk of developing arrhythmias due to digitalis through different mechanisms, including sympathoadrenal stimulation. Some classes of vasodilators, such as beta agonists, nitrates, and angiotensin-converting enzyme (ACE) inhibitors, have been tried, but, in general, vasodilators have failed to show sustained benefit in patients with COPD and are not routinely used.

A 15-year-old male presents complaining of a sore throat, headache, and mild cough that started 8 days ago and has progressed to include a worsening cough and increasing fatigue. His chest x-ray reveals bilateral hilar infiltrates and a CBC is normal. Which of the following diagnostic tests will most likely confirm the suspected diagnosis? A Acid-fast bacilli smear and culture B Blood culture C PCR testing of sputum D Sputum culture E Sputum gram stain

PCR Testing of the sputum The patient most likely has atypical pneumonia secondary to mycoplasma pneumoniae which is best confirmed by PCR testing of sputum (C), oropharyngeal or nasal secretions and isn't detected through standard cultures (B, D) or staining techniques (A, E).

A 20-year-old man presents due to developing sudden onset of shortness of breath. He is experiencing chest pain that worsens with inspiration. He has no history of pulmonary disease, but he does smoke half a pack of cigarettes daily. Vital signs reveal blood pressure 130/82, heart rate 140, respirations 28, and temperature of 98.2 degrees Fahrenheit. Heart sounds are normal with no murmurs, gallops or rubs. Lung examination reveals absent breath sounds and hyperresonance to percussion on the left side. Question What is the most likely diagnosis? Answer Choices 1 Pleural effusion 2 Pneumonia 3 Lung cancer 4 Pneumothorax 5 Asthma attack

PTX Pneumothorax occurs when air accumulates in the pleural spaces, causing the lung to collapse. Pneumothorax may be associated with underlying lung disease, or it can occur spontaneously. Spontaneous pneumothorax is more common in young men with a history of smoking. The presentation is dependent on the size of the pneumothorax. The patient in this case most likely has a large pneumothorax, as he presents with chest pain, tachypnea, tachycardia, hyperresonance to percussion, and absent breath sounds on the affected side. Diagnosis can be confirmed with chest X-ray. Pleural effusion is abnormal fluid collection in the pleuritic space. Patients will complain of pleuritic chest pain, as in this case, but they will have findings of dullness to percussion on examination. Pneumonia is unlikely; the patient is not demonstrating any signs of infection. Lung cancer may be associated with dyspnea, but it typically has gradual onset. Although the patient does have a risk factor for lung cancer (history of smoking), his young age makes the diagnosis unlikely. A patient having an asthma attack may complain of dyspnea and chest pain, but he or she would likely have wheezing on examination.

Which agent is most responsible for croup infections? A Parvovirus B Adenovirus C Parainfluenza D Bocavirus E Streptococcus

Parainfluenza Laryngotracheobronchitis, or croup, is mostly caused by the parainfluenza virus. Other agents that can cause this are respiratory syncytial virus (RSV), human metapneumovirus, influenza virus, rubeola virus, adenovirus, and Mycoplasma pneumoniae. Croup, also known as laryngotracheobronchitis, is associated with upper tracheal narrowing and edema, which is visible on an anteroposterior soft tissue neck x-ray. This is termed the "steeple sign." Epiglottitis is associated with a thickened epiglottis on a lateral soft-tissue neck x-ray, termed the "thumb" sign. Foreign body aspiration and tracheal carcinoma may have x-ray findings based on the location, size, and components present. Peritonsillitis is best visualized on physical exam. If assessing for a potential peritonsillar abscess, a contrasted CT is recommended.

A 65-year-old man presents with dry mouth and difficulties rising from a chair, climbing stairs, and walking. His symptoms started about 1 year ago, are worse in the morning, and improve with exercise. He is a heavy smoker and was recently diagnosed with lung cancer. Question What is the most likely underlying mechanism of his weakness? Answer Choices 1 Infiltration of the nerves 2 Spinal cord compression 3 Spinal cord metastasis 4 Paraneoplastic 5 Nicotine poisoning

Paraneoplastic Paraneoplastic syndrome (Lambert-Eaton syndrome), resembling myasthenia gravis, occurs in some people with small cell carcinoma of the lungs. It usually manifests as progressive weakness in the large muscles. Lambert-Eaton syndrome is caused by the inhibition of voltage-gated calcium channels on the presynaptic membrane of the neuromuscular junction; this prevents the release of acetylcholine. As the muscles continue to contract, acetylcholine can build up in sufficient quantities for the strength to get better; weakness improves after repetitive muscle contraction. Although the underlying mechanism is autoimmune, Lambert-Eaton syndrome in this patient is regarded as paraneoplastic because it is a consequence of a cancer, not due to a local presence of cancer cells. Tumor infiltration is a local manifestation of a tumor associated with the production of various types of extracellular matrix-degrading enzymes. Local infiltration of the nerves excludes variegated symptoms of autonomic nervous system disturbance and fluctuating weakness in several muscle groups. Secondary spinal cord tumors usually follow hematogenous spread to the vertebral bodies, epidural expansion, and/or intramedullary metastasis. Subsequent symptoms of compression (i.e., pain, radicular, or medullar symptoms) gradually worsen, not improve with exercise. Nicotine poisoning is not likely; it is impossible to overdose on nicotine through smoking alone. Smoking causes vascular disease, cancer, lung disease, peptic ulcer, and reproductive disturbances (e.g., premature birth). Nicotine may contribute to tobacco-related disease, but direct causation has not been determined because nicotine is consumed simultaneously with a multitude of other potentially harmful substances that occur in tobacco smoke. The effects of nicotine on nerves and muscles are generally dose-dependent; they also occur in nicotine-tolerant individuals. Initially, nicotine has a short-lived stimulatory phase followed by a longer inhibitory phase which leads to a neuromuscular blockade. Neuromuscular symptoms include hypotonia, decreased deep tendon reflexes, weakness, fasciculations, and paralysis of muscles (including respiratory muscles). Cholinergic effects on the autonomic nervous system, often observed initially, include diaphoresis, salivation, lacrimation, increased bronchial secretions, miosis, and later mydriasis. Nicotine acts on the sympathetic ganglia, chemoreceptors of the aorta, and carotid bodies; it affects the adrenal medulla, releasing catecholamines.

A 21-year-old woman presents with a severe headache; it is accompanied by sweating and palpitations. She was seen by her primary care physician earlier in the day, and he admitted her from the office when her blood pressure was recorded several times at 210/98. The patient has no known medical or surgical history, and she is currently on a daily multivitamin. She states that she has been having her symptoms episodically over the last 2 weeks, but this is the first time she has been seen by a physician. The patient is found to have elevated urinary and plasma metanephrines and catecholamines. Question What antihypertensive agent should be used for this patient? Answer Choices 1 Lisinopril 2 Phenoxybenzamine 3 Metoprolol 4 Lasix 5 Hypertension should not be treated at this time

Phenoxybenzamine The correct answer is phenoxybenzamine. The patient's young age suggests that she is suffering from a form of secondary hypertension. Given her paroxysmal headaches, diaphoresis, and palpitations accompanied by hypertension, she is likely suffering from a pheochromocytoma. Pheochromocytomas are benign tumors that arise from the adrenal glands. The tumors secrete norepinephrine, which leads to hypertension, tachycardia, headache, and diaphoresis. Diagnosis is made based on elevation in urinary and plasma metanephrines and catecholamines. and it is followed by tumor location on CT or MRI. The patient requires removal of the pheochromocytoma, but preoperative control of her blood pressure is necessary. Alpha blockers, such as phenoxybenzamine or prazosin, are the preferred antihypertensive agents in pheochromocytoma. Lisinopril is an ACE inhibitor and would not be considered first-line treatment in the control of hypertension in pheochromocytoma. Metoprolol is a beta blocker. Beta blockers may only be used in the treatment of pheochromocytoma after the patient has received an alpha blocker. If a patient with pheochromocytoma receives a beta blocker as first-line treatment, the patient will develop unopposed alpha agonism, causing worsened hypertension. Lasix is a diuretic and is not recommended as first-line treatment in the control of hypertension in pheochromocytoma. Hypertension should not be treated at this time is an incorrect answer; the patient's blood pressure is severely elevated and she requires control of her blood pressure prior to the removal of her pheochromocytoma.

A 30-year-old woman presents with a 2-day history of fever, cough with sputum, and chest pain; there is also a 1-day history of frank blood in sputum (75 cc in the last 24 hours). She is a non-smoker and does not give a history of any recent inhalation exposure or illicit drug use. There is no past history of malignancy or autoimmune disease. On examination, pulse is 92/min; BP is 106/70mmHg; and temperature is 101 F. Oxygen saturation is 97%. On auscultation, S1, S2 normal, decreased breath sounds, and increased tactile vocal fremitus are noted on the right middle lobe. There is no pallor, icterus, cyanosis, edema, or lymphadenopathy. Question What is the next best step in evaluation of her hemoptysis? Answer Choices 1 Computed Tomography (CT) scan of chest 2 Sputum for Gram stain 3 Plain X-ray of chest 4 Bronchoscopy 5 Complete blood count (CBC)

Plain X-ray of chest Plain X-ray of chest is the correct answer. It is the next step in the evaluation of hemoptysis to identify the source of bleeding1. If the chest X-ray does not help identify the source of bleeding, then a CT scan of the chest should be done1. CT scan helps to identify the source of bleeding, as well as gives information on mediastinal lymhadenopathy, which may support the diagnosis of thoracic malignancy1. Both Gram stain and CBC should be obtained for complete evaluation1. Gram stain helps evaluate for the presence of infection, and CBC is used to assess platelet count and hematocrit1. These investigations should be done following chest X-ray. If all these procedures are unrevealing, then bronchoscopy should be considered1.

A 55-year-old woman was diagnosed with small cell cancer of the lung 2 months ago; she presents with increasing dysphagia, respiratory difficulties, and weakness of the upper limb. Her vital signs are: pulse 85/min, BP 120/90 mm Hg, resp. 12/min, and temp. 37.7 C. On examination, she has ptosis of both eyes, and she reports diplopia. Her pupillary responses are normal. The strength in the muscles of her arm on testing is 2/5; on repeated testing, the strength improves to 4/5. Sensation is intact in both upper limbs. What is the most appropriate treatment for her condition? Answer Choices 1 Neostigmine 2 Atropine 3 Plasmapheresis 4 Amikacin 5 Cisapride

Plasmapheresis The Lambert-Eaton syndrome is caused by antibodies to the presynaptic calcium channel, which decreases the release of acetylcholine. Repeated stimulation of the nerve ending increases the intracellular calcium concentration, which allows enough acetylcholine to be released to cause muscle contraction. Anticholinergics would not help in this condition. The treatment lies in removing the circulating antibodies using plasmapheresis or immunosuppressants, such as prednisone or azathioprine. The Lambert-Eaton (or myasthenic syndrome) may occur as a paraneoplastic syndrome of small cell lung cancer. The presentation is similar to myasthenia gravis; however, the weakness is made worse by sustained movement in myasthenia gravis. It is improved in the Lambert-Eaton syndrome. Myasthenia gravis is caused by antibodies to the acetylcholine receptor; consequently, the treatment lies in increasing the acetylcholine concentration in the synaptic cleft using an anti-cholinergic medication such as neostigmine. Amikacin is an aminoglycoside; it can cause weakness of skeletal muscles. Cisapride is a promotility agent use for dysphagia or gastro-esophageal reflux. It has been withdrawn from the US market, and it is available only to patients who meet specific criteria. Atropine is a cholinergic antagonist at muscarinic receptors and would not be useful in this case.

You are examining a 6-month-old male infant who has not received any immunizations since birth. He was born at home, and he has suffered respiratory tract infections and chronic diarrhea since birth. His parents are not related. 2 of his older brothers died of pneumonia at the age of 8 and 12 months, respectively; however, his 4-year-old sister is healthy. His parents moved to United States from an underdeveloped country 3 months ago. They are worried because 1 of their neighbors' children, who is 6 years of age, developed a pruritic skin rash and fever 2 days ago; another child, who is 6 months of age, is in the hospital because of severe vomiting and diarrhea. The parents of your patient are worried. On examination, your patient's vitals are normal; his weight is <5th percentile, his height is at the 5th percentile, and his head circumference is at the 50th percentile. Eczematous skin rash and mucocutaneous changes suggestive to a fungal infection are noted. His thyroid, heart, lungs, abdomen, genitalia, and nervous system are within normal limits; you are not able to palpate lymph nodes, and there are no signs of hepatosplenomegaly. Question What vaccine should you recommend? Answer Choices 1 Rotavirus vaccine (RV) 2 Pneumococcal vaccine (PCV13) 3 Measles, mumps, and rubella vaccine (MMR) 4 Bacille Calmette-Guerin vaccine (BCG) 5 Variccella vaccine (Var)

Pneumococcal vaccine (PCV13) Your patient suffers recurrent respiratory infections and diarrhea; he also has skin changes and a family history highly suggestive of an X-linked immune deficiency, most likely a severe combined immunodeficiency (SCID). SCID is a group of congenital diseases caused by different genetic mutations resulting in the severe deficiency of T and B- lymphocytes. Onset before age 6 months suggests a T-cell defect, as maternal antibodies are present for the 1st 6 to 9 months. Live vaccines are contraindicated in SCID because introducing an infectious agent, even a weakened one, can cause actual disease. However, pneumococcal vaccine PCV13 is considered risk-specific and is recommended in a 4-dose series at ages 2, 4, and 6 months (minimum age of 6 weeks), as well as at age 12 through 15 months. Rotavirus (RV) is a live vaccine and cannot be given to SCID patients. The measles, mumps, and rubella (MMR) vaccine is a live vaccine and cannot be given to patients with SCID. BCG, or Bacille Calmette-Guerin, is not generally recommended for use in the United States because of the low risk of infection with Mycobacterium tuberculosis, the variable effectiveness of the vaccine against adult pulmonary TB, and the potential interference with tuberculin skin test reactivity. More importantly, in regard to this case, it is a live vaccine and should not be given to any immunosuppressed person. Variccella (Var) is also a live vaccine and cannot be given to SCID patients.

A 30-year-old immunocompromised patient presents with a 2-week history of breathlessness and a nonproductive, dry cough. The patient is afebrile, pulse is 100, and BP is 110/70 mm Hg. On auscultation, scattered rales all over the chest are heard. A chest X-ray shows diffuse air space and interstitial shadowing in both lungs. The shadowing is more prevalent in the apical region. What is the most likely diagnosis? Answer Choices 1 Tuberculosis 2 Mycoplasma pneumoniae 3 Viral pneumonia 4 Bacterial pneumonia 5 Pneumocystis pneumoniae

Pneumocystis pneumoniae The patient being immunocompromised narrows the choices to Pneumocystis pneumoniae and tuberculosis; diffuse infiltrate is more common in Pneumocystis infections of the lung. The fungus Pneumocystis jirovecii causes pneumocystis pneumonia. Tuberculosis shows localized infiltrate; it is commonly seen at the apices with hilar lymphadenopathy. Bacterial pneumonia will have an acute history of 2 - 4 days with fever and a nonproductive cough. The X-ray will show lobar infiltrate; it is rarely found throughout the lung. Viral pneumonia will have fever before the onset of the respiratory symptoms. Diagnosis is usually by isolating the organisms as well as serological tests. The presence of cavities in the lung indicates bacterial causes, fungal (Mycoplasma) causes, tuberculous infection, or neoplasm.

A 25 year-old HIV positive white male presents with dyspnea and a nonproductive cough. The patient's temperature is 103.5 degrees F, respirations 40, and pulse 140. Physical exam reveals a thin, dyspneic male, height 5'10" and weight 150 pounds, in moderate respiratory distress with minimal bibasilar crackles and peripheral cyanosis. CXR reveals a diffuse interstitial infiltrate and his LDH is elevated. Your most likely diagnosis is: A Streptococcal pneumoniae B Klebsiella pneumoniae C Mycoplasma pneumoniae D Pneumocystis jiroveci E Staphlococal aureus

Pneumocystitis jiroveci D The insidious onset of atypical pneumonia symptoms in a patient with HIV is consistent with pneumocystis jiroveci (D). Streptococcal pneumoniae (A), Klebsiella pneumonia (B), and Staphlococal aureus (E) present as "typical" pneumonia with the acute onset of fever and productive cough.

A 42-year-old woman is noted to have a solitary parenchymal pulmonary nodule in the right upper lobe on a routine plain chest film. This was not evident on a chest film taken 1 year earlier. The patient has a history of invasive ductal carcinoma of the right breast; she underwent a radical mastectomy 6 years ago. At that time, no lymph nodes were involved. 2 years later, she had a chest-wall recurrence and underwent wide local excision followed by radiation therapy. She was disease free for the past 4 years, until the plain chest film confirmed the pulmonary nodule. She had no history or risk factors of HIV or tuberculosis, and there is no history of travel that involved exposure to pulmonary mycosis. The patient smokes 5 cigarettes a day. She denies hemoptysis or other pulmonic or constitutional symptoms. On physical exam, her temperature is 98 degrees Fahrenheit (37 Celsius). She has no lymphadenopathy or abnormal palpable breast masses on the left side. Her complete blood count is normal. A CT scan of the patient's chest revealed only a nodular opacity in the right upper lobe measuring 2 cm x 1.5 cm. Question What is the most appropriate next step in the patient's evaluation? Answer Choices 1 Bone scan 2 Bronchoscopy 3 Positron emission tomography (PET) scan 4 Thoracotomy 5 Transthoracic needle aspiration biopsy

Positron emission tomography (PET) scan A chest CT scan is an important part of a solitary lung nodule. It is more sensitive than plain radiography and can determine whether a nodule identified on a chest film is indeed solitary. It provides better visualization of the nodule, is more sensitive for calcification, and can be used to guide thoracic needle aspiration biopsies. After the CT of the chest, the next step would be the PET scan. This study uses uptake of 2-fluoro-dexy-D-glucose (FDG) to measure glucose metabolism. The uptake of FDG by lung tumors is greater than that of normal lung parenchyma. In a patient with a solitary nodule, the sensitivity and specificity of PET for malignancy are about 95% and 90% respectively. With nodules less than 1.5 centimeters, PET is less reliable and also produces a higher rate of false-negatives. PET is particularly useful in evaluating indeterminate lesions in patients who are a poor surgical risk. Other benefits of PET scanning are detection of occult metastases and improved staging. A CT-PET strategy has been shown to be more cost-effective and also prevents more unnecessary thoracotomies than conventional approaches. CT-PET can demonstrate nodule enhancement. PET scanning is also the preferred approach in patients who have a solitary lung nodule classified as "indeterminate" based on the initial chest film and CT scan. If the PET scan is negative, follow up with serial CT scans. A positive PET scan indicates that malignancy is likely. PET is particularly useful in evaluating lesions in patients who are a poor surgical risk. Other benefits include detection of occult metastases and improved staging. A bone scan would be useful when the patient has symptoms of bone pain, mid-thoracic pain, or night fevers. An incidental finding of a high serum calcium level in this clinical setting may also warrant a bone scan. Bronchoscopy and transthoracic needle aspiration biopsy (TNAB) are the traditional options. Bronchoscopy may be useful if the lesion is 2 centimeters or more in diameter and can be accessed via a bronchus. In other settings, the false-negative rate is high; thus, in most patients, bronchoscopy is of limited value and has a relatively low diagnostic yield. TNAB is considered when the solitary nodule is smaller than 2 centimeters. The diagnostic yield is high for peripheral lesions. A negative result on TNAB may reliably rule out pulmonary mycosis in patients who live in areas where the incidence of this infection is high. TNAB has a higher complication rates than bronchoscopy. Although thoracotomy is the most definitive way to establish a diagnosis in patients with a solitary lung nodule, operative mortality is 3% to 7%. If it is video-assisted, then there is lower perioperative morbidity and a shorter hospital stay. In 25% of cases, a video-assisted thoracoscopic procedure needs to be converted to an open thoracotomy.

A premature infant is born with the inability to produce adequate amounts of pulmonary surfactant. Which of the following is the primary function of pulmonary surfactant? A increase the solubility of carbon dioxide in the alveoli B increase the solubility of oxygen in the alveoli C prevent infectious organisms from infiltrating the alveoli D prevent the collapse of small lung alveoli E stimulate the unloading of carbon dioxide from hemoglobin

Prevent the collapse of lung alveoli Because of their small size, many lung alveoli are prone to collapse. Pulmonary surfactant contains a high concentration of amphipathic phospholipid molecules, which lowers the surface tension of alveoli. According to the law of Laplace, a reduction of surface tension reduces the collapsing pressure on small alveoli and allows them to remain open. Pulmonary surfactant production does not typically begin until the 24th week of gestation; hence, an infant born before this time is at great risk for having collapsed alveoli.

A 44-year-old healthy man, who is a non-smoker, has a 3-cm coin lesion in the right upper lobe. The lesion was revealed on his chest radiograph. The patient did not have any clinical symptoms. The fine-needle aspiration did not reveal any cells. What is the most likely tumor in this case? Answer Choices 1 Pulmonary hamartoma 2 Bronchial carcinoid 3 Mesothelioma 4 Metastatic adenocarcinoma 5 Large cell undifferentiated carcinoma

Pulmonary Hamartoma A peripheral 'coin lesion' could be a granuloma, carcinoma, or a hamartoma. The most likely malignancy in a non-smoker would be adenocarcinoma. Pulmonary hamartoma is the most common benign lung tumor, although it is a relatively uncommon lesion. It is likely an acquired lesion from peribronchial mesenchyme. The patients are generally asymptomatic. The lesion is almost always solitary and is usually peripheral; hence, it does not reveal any cells on fine-needle aspiration. Histologically, hamartomas are composed of both mesenchymal and epithelial elements. A bronchial carcinoid is (usually) a central benign tumor with some of the properties of malignant growth. Mesothelioma is a bulky pleural mass. It is rare, even in smokers with asbestos exposure. Metastatic adenocarcinoma usually produces multiple lung nodules. Large cell undifferentiated carcinoma is a large peripheral mass.

What is the most common pathogen in pediatric bronchiolitis? A Mycoplasma B Chlamydia C Parainfluenza D Respiratory syncytial virus (RSV) E Streptococcus

RSV Viruses are by far the most common reasons for bronchiolitis to occur. RSV is the clear leader in causative agents.

A 40-year-old man presents in January complaining of a 4-day history of fevers, chills, myalgias, headache, productive cough, and mild sinus congestion. He has no significant past medical history. His physical examination reveals a temperature of 102.7° F, pulse 96/min, respiratory rate 20/min, and blood pressure of 128/80. There is mild maxillary sinus tenderness. His oral cavity and oropharynx are clear. His tympanic membranes are pearly gray with normal light reflex. His chest is clear to auscultation; cardiac exam is unremarkable. What is the best course of therapy for this patient? Answer Choices 1 Admit to the hospital for antibiotics and intravenous fluids 2 Treat with oral trimethoprim/sulfamethoxazole for 7 days 3 Prescribe amantadine 4 Prescribe zanamivir 5 Recommend bed rest, analgesics, and topical decongestants only

Recommend bed rest, analgesics, and topical decongestants only The best course of treatment is supportive and symptomatic care with hydration, analgesics, and decongestants. This patient presents with classic symptoms of an influenza infection. He has no evidence of a severe bacterial infection; therefore, antibiotics are not indicated. It is important to note that the flu can cause sinus congestion as well as rhinorrhea. Although amantadine and zanamivir are approved for treatment of influenza infections, they have only been found to be helpful if treatment has begun within 48 hours of the initial symptoms. If prescribed within this period, the duration of illness can be shortened by about 1 day.

A 68-year-old man with a history of cirrhosis presents due to weight gain, increased girth, and shortness of breath. He denies fever or chills, cough, melena, hematemesis, hemoptysis, and confusion. He takes furosemide on a daily basis and has not missed any doses. On physical exam, his blood pressure is 120/80; pulse is 78, and respiratory rate is 18. He is alert and cooperative. Heart exam: regular rate and rhythm without murmurs. Lung exam: decreased breath sounds on right side. Abdominal exam: positive fluid wave, no tenderness to palpation. There is no hepatosplenomegaly appreciated. Chest X-ray shows a right-sided pleural effusion; it is free flowing, as is evidenced on a decubitus film. Question What is the etiology of his pleural effusion? Answer Choices 1 Altered permeability of pleural membrane 2 Reduced intravascular oncotic pressure 3 Increased hydrostatic pressure 4 Decreased lymphatic drainage 5 Reduced pressure in pleural space

Reduced intravascular oncotic pressure Pleural effusions, an abnormal collection of fluid in the pleural space, is related to an underlying pathology causing excess fluid production or decreased absorption. Effusions are classified as transudative or exudative depending on the characteristics of the fluid. Patients with liver disease/cirrhosis have decreased protein and albumin production, leading to decreased oncotic pressure of the intravascular fluid. This contributes to ascites and dependent edema. Peritoneal ascites may travel across the diaphragm and lead to pleural effusions (hepatic hydrothorax), most commonly on the right side. Pleural effusions caused by decreased oncotic pressure are transudative. Treatment is aimed at reducing the ascitic fluid using diuretics (loops and/or spironolactone). Severe cases may require treatment with a transjugular intrahepatic portosystemic shunt (TIPS) or liver transplant. Other causes of hypoalbuminemia, such as nephrotic syndrome, also cause pleural effusions due to reduced intravascular pressure. Altered permeability of the pleural membrane causing a pleural effusion is seen with localized inflammation, malignancy, or pulmonary embolism. Decreased lymphatic drainage is commonly seen in malignancy (lung, breast, or lymphoma). The effusion is exudative and almost always unilateral on the side with the associated pathology. Pleural effusions caused by congestive heart failure are due to increased hydrostatic pressure. They are usually bilateral and transudative. Reduced pressure in the pleural space prevents full lung expansion, allowing pleural fluid accumulation. Causes of this include atelectasis (transudative) or mesothelioma (exudative).

A 17 year-old male is training for an Ironman triathlon and notes excessive coughing, chest tightness and wheezing when running. Which of the following is the most appropriate treatments for this patient? A Albuterol B Cromolyn C Fluticasone D Ipratropium bromide E Salmeterol

Salmeterol The mainstay of treatment for exercise-induced asthma are beta-2 agonists. Due to the duration of physical activity this patient should be treated with a long-acting beta-2 agonist, salmeterol (E) instead of albuterol (A). If his symptoms are persistent then addition of a daily inhaled corticosteroid (C) is required, and daily ICS should be added if his use of salmeterol is frequent to avoid asthma-related death associated with the lone use of long-acting beta-agonists. Cromolyn (B) is an additional treatment options for exercise-induced bronchospasm, but is less commonly used compared to beta-2 agonists.

A 50-year-old man presents with a several month-history of increasing shortness of breath, fever, weight loss, and night sweats. The patient is a non-smoker who works in cotton fields. A chest X-ray reveals lymphadenopathy and reticulonodular pattern in all lung fields. A transbronchial biopsy is performed and microscopically demonstrates non-caseating granuloma. What is the most likely diagnosis? Answer Choices 1 Adenocarcinoma 2 Sarcoidosis 3 Histoplasmosis 4 Unusual interstitial pneumonitis 5 Berylliosis

Sarcoidosis Sarcoidosis is a multisystem granulomatous disease. Histologically, it is characterized by the presence of non-caseating granulomas, hence the diagnosis. The mediastinal and superficial lymph nodes, lungs, liver, spleen, skin, eyes, parotid glands, and phalangeal bones are frequently involved; however, all tissues may be involved. Sarcoidosis is not fatal unless it affects the vital organs such as the heart or the central nervous system. Calcium metabolism may be disturbed, causing hypercalcemia and (rarely) nephrocalcinosis and renal failure. 30% of the patients of sarcoidosis are asymptomatic. The disease is commonly detected by an abnormal chest radiograph, which reveals bilateral hilar lymphadenopathy in an asymptomatic patient. However, with extensive lung involvement, there may be exertional dyspnea or cough. The diagnosis can be made confidently from the clinical and radiological features. It can also be confirmed histologically from the biopsy of a superficial lymph node or skin lesion. Kveim test is a useful diagnostic procedure for sarcoidosis. The antigen (0.1 ml) obtained from human sarcoid tissue is injected intradermally and a small nodule develops 4 weeks later. If the test is positive, biopsy of the nodule reveals typical sarcoid follicle. The development of positive Kveim test is suppressed by corticosteroid therapy. Asymptomatic cases resolve spontaneously, but patients with persistent erythema nodosum, pyrexia, and arthralgias require oral corticosteroid therapy for a short duration. Symptomatic pulmonary sarcoidosis and sarcoidosis involving the eye or other vital organs usually needs to be treated with corticosteroids for several years. Adenocarcinoma will yield malignant cells, not granulomas, on biopsy. Histoplasmosis is caused by dimorphic fungi, Histoplasma capsulatum, or a variant, Histoplasma duboisii, which is found in the soil. It is caused by inhalation of the infected dust, which produces lesions similar to tuberculosis and have granulomas with central caseation. Interstitial pneumonitis is the end result of diffuse fibrosing alveolitis. Many cases are idiopathic. The end stage is a honeycomb appearance and no granulomas are seen. Berylliosis produces sarcoid-like lesions, but there would have to be a history of exposure to beryllium. Exposure to beryllium is now extremely uncommon.

A 65-year-old man presents with a 25-pound unexplained weight loss. He also has noticed a change in his usual cough. He sees his family doctor. Upon questioning for the medical history, the doctor discovers that he has a 60-pack/year history of smoking as well as dyspnea. On physical examination, he appears to have Cushing's syndrome. X-rays show a central lesion with no cavitation. A hilar mass is seen on chest X-ray. What is the most likely diagnosis? Answer Choices 1 Small cell carcinoma 2 Adenocarcinoma 3 Metastatic Wilms tumor 4 Large cell carcinoma 5 Squamous cell carcinoma

Small cell carcinoma Small cell carcinoma is most frequently a central lesion not associated with cavitation. A hilar mass is associated with small cell carcinoma. Small cell carcinoma is associated with paraneoplastic syndromes. Cushing's syndrome secondary to ectopic ACTHproduction can sometimes be seen with small cell carcinoma, as can syndrome of inappropriate antidiuretic hormone production (SIADH). An adenocarcinoma is usually seen peripherally on X-ray. An adenocarcinoma is characterized pathologically by mucus production. On the rare occasion when lung cancer occurs in somebody who has never smoked, it is most often classified as an adenocarcinoma. Adenocarcinomas may develop in areas of the lung where there is scarring or fibrosis. For example, a cause of the scarring can be a granulomatous infection, such as tuberculosis. For this reason, adenocarcinoma is sometimes referred to as "scar carcinoma". Wilms tumor is a pediatric malignancy of the kidney. It can metastasize to the lungs. Large cell carcinoma is usually seen peripherally on X-ray. Large cell carcinoma shows poorly differentiated cells. It does not show squamous features or glandular features. Large cell carcinoma does not react to mucicarmine staining. Large cell carcinoma is also called undifferentiated carcinoma or anaplastic carcinoma. The cell of origin is unknown. Squamous cell carcinoma is centrally located. Cavitation can also sometimes be seen with X-ray. Keratin formation, keratin pearl formation, and intercellular bridges can be seen with squamous cell carcinoma.

A 25-year-old male basketball player presents with acute onset shortness of breath associated with right-sided chest pain. The pain is unaffected by position and is worse with inspiration. He was grocery shopping when it started. He denies chest trauma. He had an upper respiratory infection earlier in the month that had resolved without incident. He smokes 1 pack of cigarettes per day and has no significant past medical history. On PE he is afebrile; BP is 138/80; P is 124; R is 24; and pulse oximetry is 94% on room air with mild respiratory distress. Trachea is midline. Lungs are clear to auscultation bilaterally with no wheezing or rhonchi; tactile fremitus and percussion are equal throughout posterior lung fields. Heart is tachycardic with normal S1 and S2, no murmur, rubs, or gallops. Question What is the diagnosis? Answer Choices 1 Pericarditis 2 Pulmonary embolism 3 Spontaneous pneumothorax 4 Acute chest syndrome 5 Community acquired pneumonia

Spontaneous Pneumothorax The patient has a primary spontaneous pneumothorax (PSP). There is a higher incidence in males and usually occurs between 20-40 years of age. PSP is heavily associated with smoking. Patient presentation and physical exam findings will vary depending on the size of the pneumothorax. Most commonly, patients present with unilateral chest pain, dyspnea, and cough occurring with minimal activity, and tachycardia and tachypnea. A large PSP may become a tension pneumothorax, and the patient may have a deviated trachea and no breath sounds on affected side with increased resonance to percussion with respiratory distress and shock. Imaging of choice for a PSP is a chest radiograph, usually an upright inspiratory film. A white visceral pleural line is diagnostic. Supine or lateral decubitus views can also be used. Pericarditis is a differential diagnosis, but the chest pain is usually relieved with positioning. Pulmonary embolism is another differential, but the patient does not have risk factors of hypercoagulability, venous stasis, or vascular injury. A patient with CAP would usually be febrile and complain of associated symptoms such as a productive cough and malaise. Acute chest syndrome is associated with sickle cell disease.

A 2-year-old male presents with a 10-day history of fever, cough, and decreased appetite and fluid intake. He is normally healthy. On examination, the child appears ill, has a temperature of 102.2 0 F, a pulse rate of 122, and a respiratory rate of 36. On auscultation of the lungs there are rhonchi heard on the right lung fields, as well as a small amount of wheeze. A chest x-ray is ordered, which reveals the presence of pneumatoceles. Based on these findings, what is the most likely pathogen causing this patient's infection? A Streptococcus pneumoniae B Hemophilus influenza C Staphylococcal aureus D Pseudomonas aeurginosa E Chlamydia pneumoniae

Staph Aureus Pneumatoceles, pyopneumothorax, and empyemas are frequently encountered in pediatric Staphylococcal aureus pneumonias.

A 75-year-old man was recovering from a pneumonia caused by Streptococcus pneumoniae. The patient seemed to be doing fine, but his condition suddenly deteriorated. He presents after developing a persistent fever, chills, cough, and diaphoresis. A CBC reveals leukocytosis with a left-shift. A chest X-ray demonstrates an air-fluid level in the pleural space, which is suggestive of an abscess. Question What organism caused the patient's deterioration? Answer Choices 1 Pseudomonas aeruginosa 2 Chlamydia trachomatis 3 Coccidioides immitis 4 Staphylococcus aureus 5 Legionella pneumophila

Staphylococcus Aureus The clinical picture is suggestive of a pleural empyema. Of the organisms listed, empyemas are commonly caused by Staphylococcus aureus, Strep pneumoniae, Strep pyogenes, and H. influenza in the adult patient. The other organisms listed either do not cause an empyema, or they infrequently cause an empyema.

A patient with HIV positive status, a CD4 count of 277/mcL, and is on chronic HIV medication therapy is diagnosed with pneumonia. What is the most common pathogen that would result in pneumonia for this patient? A Pneumocystosis jiroveci B Streptococcus pneumoniae C Pseudomonas aeurginosa D Klebsiella pneumoniae E Hemophilus influenzae

Strep Pneumo Due to the tight control of most of the HIV population, the most common pathogen is actually the most common community-acquired pathogen in the United States, Streptococcus pneumoniae. When the patient has lower CD4 counts and higher viral loads they are more susceptible to pathogens such as pneumocystosis jiroveci.

A 46-year-old man is hospitalized due to pneumonia. Over the course of 24 hours, he develops a very bad headache and stiff neck followed by a seizure. On physical examination, he is febrile with signs of meningeal irritation. A lumbar puncture results are as follows: TEST RESULTS and REFERENCE RANGE: CSF pressure- 250 mm H2O (70-80 mm H2O) CSF leukocytes- 8,700 cu mm (0-10 cu mm) CSF glucose- 22 mg/dL (45-80 mg/dL) CSF protein- 233 mg/dL (15-45 mg/dL) CSF gram stain- pending What organism is the most likely cause of his meningitis? Answer Choices 1 Streptococcus pneumoniae 2 Streptococcus agalactiae 3 Cryptococcus neoformans 4 Staphylococcus epidermidis 5 Staphylococcus aureus

Strep Pneumonaie The most likely pathogenic organism for bacterial meningitis depends on the age and health of the patient. Streptococcus pneumoniae is a common pathogen for meningitis. Cryptococcus neoformans is a cause of meningitis in immunocompromised patients. Staphylococcus epidermidis is a common cause of meningitis in people with CNS shunts. Staphylococcus aureus meningitis is associated with trauma, neurosurgery, or some other underlying illness or debilitating condition. Streptococcus agalactiae is a likely cause of meningitis in neonates and infants. It is the most common pathogen of neonatal meningitis because it is often found in the genital tract of women. Streptococcus agalactiae is a Gram-positive coccus.

An aspirated peanut that is causing a partial obstruction of the trachea in a child is most likely to cause which of the following physical exam findings? A Aphonia B Inability to cough C Stridor D Progressive cyanosis E Rhonchi

Stridor Incomplete airway obstruction due to a foreign body will cause turbulent air flow in the airway and an inspiratory wheeze sound, known as stridor. Attempts should be made to remove the foreign body, leaving the patient with a partial obstruction to utilize the cough reflex to remove the foreign body. If unsuccessful, or if findings such as aphonia, an inability to cough, progressive cyanosis, or unconsciousness occur, a complete obstruction is present and definitive intervention must convene. Age appropriate measures, utilizing back blows and chest compressions in infants younger than the age of one and abdominal thrusts in children over the age of one, should be attempted.

A 1-year-old female is having a 2-day history of fever (102 0 F oral), rhinorrhea, and dry cough, with a decreased appetite. The mother states that her daughter has been less active, and her fluid intake has decreased for her age. On exam, the child is non-toxic appearing, has a rectal temperature of 100.2 0 F, and has nasal flaring and a respiratory rate of 45, rhinorrhea, moist mucous membranes, and a minimal wheeze heard bilaterally. Her chest x-ray has no specific findings. What is the initial treatment of choice for this patient with these symptoms? A Antibiotics B Oxygen therapy C Supportive care D Antiviral medications E Plasmaphoresis

Supportive care The choice for treatment of acute bronchiolitis is supportive care. Oxygen therapy is only reserved for those patients who are hypoxic, and antiviral medications have not proven to be effective in shortening or eradicating the infection.

A 45-year-old man presents with significant weight loss (10 pounds in 4 months), cough with hemoptysis, and pleuritic chest pain. The chest X-ray shows ill-defined opacities in both the lungs; the opacities have a reticulonodular pattern. A transbronchial biopsy is performed, and it microscopically shows a few epithelioid cells with necrotic debris. What is the most likely diagnosis? Answer Choices 1 Aspergillosis 2 Squamous cell carcinoma 3 Pneumocystis carinii pneumonia 4 Oat cell carcinoma 5 Tuberculosis

TB The epithelioid cells suggest that it is a granuloma. The hemoptysis suggests that the granuloma has eroded enough parenchyma and involved a bronchus. A granulomatous infection along with hemoptysis is typical for tuberculosis. The history of significant weight loss and hemoptysis is also typical of tuberculosis, but it can be seen in other cases also. Aspergillosis is more likely to produce a fungus ball, which is defined on an X-ray. The inflammatory reaction would vary from acute to mixed to granulomatous. Squamous cell carcinoma is usually a central mass lesion. It is unlikely to produce epithelioid cells, which are seen in the biopsy of this patient. Pneumocystis carinii pneumonia has a granulomatous pattern in rare cases, but the granulomas are small and unlikely to produce hemoptysis. Oat cell (or small cell) carcinoma is centrally located and is unlikely to be found with epithelioid cells.

A 28-year-old man presents with acute onset of dyspnea and left-sided chest pain. He denies any trauma or previous similar complaints in the past. Chest X-ray demonstrates a visceral pleural line just under the left hilum. Question What left-sided findings would you expect on physical exam? Answer Choices 1 Positive whisper pectoriloquy 2 Egophony 3 Increased tactile fremitus 4 Hyperresonance to percussion 5 Pleural rub

The correct answer is hyperresonance to percussion over the affected area, which is the most consistent finding on physical exam for a pneumothorax. Based on history, physical exam and radiographic findings, spontaneous pneumothorax is the most likely diagnosis. Tactile fremitus is decreased over the affected area on physical exam. Whisper pectoriloquy is absent over the affected area. Egophony is heard when consolidation is present, not with a pneumothorax. A pleural rub is not typically heard with a pneumothorax, but it is heard in conditions such as pleuritis, pleural effusion, and possibly pneumonia.

A 75-year-old man presents with a 4-month history of dyspnea on exertion and a productive cough. He also unintentionally lost 10 pounds in 2 months. His past medical history is significant for coronary artery disease and a myocardial infarction. He has smoked an occasional cigar the last few years. He has been retired for 12 years, but for 30 years he worked odd jobs in the construction industry. He also helped his father in the family's car garage shop. Vital signs are normal. His physical exam is remarkable for decreased breath sounds in left lower lung fields and dullness to percussion. A chest radiograph is ordered, and it shows a left sided pleural effusion. Question What condition does this patient have? Answer Choices 1 Lung cancer 2 Congestive heart failure 3 Malignant mesothelioma 4 Pneumonia 5 Recurrent postmyocardial infarction pericarditis

The correct answer is mesothelioma. His work history predisposed him to asbestos. Mesothelioma can take more than 40 years to surface, and the most common industries that expose people to asbestos are construction, mining, brake linings, roofing, and working in shipyards. This patient worked in both construction and a car garage shop, which would have likely exposed him routinely to asbestos (1). Lung cancer is not likely as he does not have the classic risk factors for developing it. His cigar smoking history is not significant. Congestive heart failure is an incorrect choice; his work history is the most important thing to focus on in this case. There is no mention of any significant cardiac findings on exam. Also, there is a history of loss of weight, which favors mesothelioma. Pneumonia is incorrect. Even though he could have this as an underlying disease, the pleural effusion seen on the chest radiograph is indicative of malignant mesothelioma. Recurrent postmyocardial infarction pericarditis is incorrect because there was no mention of pleuritic or inspiratory chest pain.

A 54-year-old Caucasian man presents with sudden severe shortness of breath. The patient is well known to you due to a 5-year history of severe emphysema. He states that he has been a basketball player all his life; he was practicing about 1 hour prior to presentation, at which point he experienced sudden chest pain and immediate shortness of breath. It is still bothering him. He describes the chest pain in the middle of the chest, more so on the right anterior side. The patient admits to smoking 2.5 packs of cigarettes daily. The patient has had at least 5 episodes similar to this one since he was 19 years old. Physical examination reveals a tall, thin, well-developed man in moderate distress. Other abnormalities include mild tachycardia (120 beats per minute) and diminished breath sounds in the posterior right lower lobe. There is noted wheezing and crackles heard throughout the rest of the lung fields in both the inspiratory and expiratory phases of breathing. Question Based upon the most likely diagnosis at this time, what pharmaceutical option may be used to help decrease this occurrence in the future? Answer Choices 1 Inhaled corticosteroids 2 Intravenous antibiotics 3 Talc sclerotherapy 4 Inhaled short-acting β2-agonist 5 Oral antibiotics

The correct response is Talc sclerotherapy. The clinical scenario is highly suggestive of a (recurrent) secondary pneumothorax. Components that lead to this diagnosis include unilateral, sudden chest pain with dyspnea and minimal physical exam findings, in this case the mild tachycardia and diminished breath sounds. The above scenario most likely fits what is considered a secondary pneumothorax, which usually occurs as a complication of various pathologies, including COPD, asthma, cystic fibrosis, tuberculosis, Pneumocystis pneumonia, and various interstitial lung diseases. Diagnosis is usually confirmed with chest radiograph. Treatment is typically observation and potential surgical intervention after about 5 days if the pneumothorax fails to resolve itself. However, the ability of the patient to withstand a surgical intervention also must be considered. There are various thought processes in terms of management of recurrent pneumothorax especially. The correct option out of the above is the talc sclerotherapy; this is commonly reserved for patients who may not necessarily be surgical candidates. Intravenous antibiotics would be correct if the pneumothorax were actually caused by the sequelae of staphylococcal pneumonia. Oral antibiotics, short-acting β2-agonist, and inhaled corticosteroids are inappropriate in the management of a secondary pneumothorax.

A 37-year-old man with an unremarkable past medical history presents during a cold winter's day with a 10-day history of acute onset of productive cough with a moderate amount of yellow sputum. There is associated fever, shortness of breath, and malaise. He denies recent travel, sick contacts, occupational exposure, and a history of smoking or alcohol use. He denies arthralgias, chills, wheezing, abdominal pain, nausea, vomiting, diarrhea, edema, or rashes. His physical exam is remarkable for fever, tachypnea, reduced fremitus, dullness to percussion, and basilar crackles in the right lower lung field. Question What additional presentation finding would be most likely in this patient? Answer Choices 1 Chest pain precipitated by exertion and an S4 gallop 2 Unilateral, sharp inspiratory chest pain and thoracic friction rub 3 Chest pain relieved by antacids and provoked by food intake 4 A vesicular rash on the thorax preceded by paresthesias 5 Hemoptysis, tachypnea, unilateral lower extremity edema

Unilateral, sharp inspiratory chest pain and thoracic friction rub This patient's diagnosis represents a pleural effusion, a complication of bacterial pneumonia; an exudative effusion is most likely. Common findings of a pleural effusion include unilateral, sharp, or knife-like chest pain that is provoked by breathing, coughing, or movements of the trunk. Physical exam findings include dullness to percussion, a deviated trachea away from the side in large effusions, decreased to absent breath sounds, a pleural friction rub, and reduced tactile fremitus. Chest pain precipitated by exertion and an S4 gallop is consistent with angina. Chest pain relieved by antacids and provoked by food intake is likely representative of reflux esophagitis or gallbladder disease; a vesicular rash on the thorax in a dermatomal distribution, often preceded by paresthesias, is consistent with herpes zoster. Hemoptysis, tachypnea, and unilateral lower extremity edema should raise suspicion of a pulmonary embolus.

A 78-year-old male presents to the office due to increasing exertional dyspnea and cough for the past week. Physical exam reveals an S3 gallop, mild JVD, and 2+ pitting edema of the bilateral lower extremities. The patient has had mild congestive heart failure in the past. A chest x-ray reveals prominent pulmonary vasculature without any additional complications. The patient's current medications are aspirin and simvastatin daily. He has no drug allergies. Which of the following is the most appropriate initial management for this patient? A Angiotensin II receptor blocker B β blocker C Calcium channel blocker D Nitrate E Thiazide diuretic plus ACE inhibitor

Thiazide diuretic plus ACE-I ACE inhibitors are recommended, as first line treatment for symptomatic congestive heart failure, based upon clinical trials, reveal an approximately 20% reduction in CHF mortality in symptomatic heart failure patients. Diuretics provide CHF symptom improvement and promote water and sodium excretion to decrease intravascular volume. blockers and Angiotensin II receptor blockers are also beneficial for CHF patients. Nitrates are usually reserved for acute or decompensating patients. Calcium channel blockers may accelerate CHF progression and should be used with caution.

A 41-year-old woman presents due to worsening symptoms. She was diagnosed with idiopathic pulmonary hypertension about 2 years prior to presentation; she is on home oxygen therapy. She has longstanding fatigue and dyspnea, but she is now experiencing profound dyspnea with exertion; swelling in her ankles; some discomfort in her right, upper abdomen; and the inability to breathe well when lying down. She has always been thin, but her weight has increased by 10 pounds in the last month. She denies fever and chills. She recently had an electrocardiogram (ECG), but she has not seen a healthcare provider to discuss the results. The ECG report indicates peaked p waves, right axis deviation, and tall R wave in V1. Question What is the most appropriate intervention for her current condition? Answer Choices 1 Prescribe a calcium channel blocker, such as verapamil 2 Prescribe a diuretic, such as furosemide 3 Prescribe a fluoroquinolone, such as levofloxacin 4 Prescribe a lipase inhibitor, such as orlistat 5 Prescribe a thiazolidinedione, such as pioglitazone

This patient is presenting with a progression of a primary pulmonary disease (pulmonary hypertension) into cor pulmonale, which is also known as pulmonary heart disease. Cor pulmonale, when moderate to severe, will present with signs and symptoms of right heart failure, such as the severely fluid overloaded state. In addition to treatment of the underlying pulmonary disorder, cor pulmonale is treated much like right-sided heart failure is treat. It would be most appropriate to prescribe a diuretic, such as furosemide, at this time. If possible, this patient should also be referred to a cardiopulmonary specialist.

A 72-year-old man is evaluated at the bedside following hospital admission for a 1-year history of progressive dyspnea, weight loss, low-grade fevers, fatigue, and myalgias. His past medical history is remarkable for atrial fibrillation, for which he takes amiodarone, hypercholesterolemia, and recurrent urinary tract infections, for which his urologist prescribed nitrofurantoin on a chronic, prophylactic basis. He denies any cigarette use, history of murmurs or coronary artery disease, chest pain, wheezing, hemoptysis, syncope, abdominal pain, rashes, peripheral edema, diaphoresis, or vomiting. Bedside echocardiogram and electrocardiograms are unremarkable for abnormalities. A chest x-ray revealed peripheral reticular opacities at the lung bases and a generalized honeycombing pattern (refer to image). Question Which of these statements regarding the clinical presentation of this patient is correct? Answer Choices 1 Acute, severe shortness of breath is the most common presenting symptom. 2 Most patients are asymptomatic at the time of diagnosis. 3 The most common symptoms are fever and a productive cough. 4 Physical exam findings of pulmonary hypertension commonly occur. 5 The pulmonary exam routinely reveals generalized wheezing and rhonchi.

This patient's most likely diagnosis is idiopathic pulmonary fibrosis. It is critical to obtain a complete history, including medication history, social history, occupational history, exposure history, and review of systems, to ensure other causes of interstitial lung disease are excluded. Amiodarone, bleomycin, and nitrofurantoin are notable medications associated with pulmonary fibrosis. Most patients with idiopathic pulmonary fibrosis present with a gradual onset, often for longer than 6 months duration. Dyspnea, which is the most prominent symptom in idiopathic pulmonary fibrosis (IPF), usually begins insidiously and is often progressive. Approximately 5% of patients have no presenting symptoms when idiopathic pulmonary fibrosis is diagnosed. The clinical symptoms of idiopathic pulmonary fibrosis are nonspecific; symptoms often precede the diagnosis by a median of 1-2 years. Most patients present with exertional dyspnea and a nonproductive cough. Pulmonary hypertension is a common comorbidity in patients with idiopathic pulmonary fibrosis, and an estimated 20-40% of patients with idiopathic pulmonary fibrosis who are evaluated or listed for lung transplantation have pulmonary hypertension at rest. Patients may have a loud P2 component of the second heart sound, a fixed split S2, a holosystolic tricuspid regurgitation murmur, and pedal edema. As right ventricular hypertrophy ensues, a right ventricular heave may be palpated at the lower left sternal border and increased right atrial pressure may cause elevation of the jugular venous pressure. The chest radiograph lacks diagnostic specificity for idiopathic pulmonary fibrosis. In most patients with idiopathic pulmonary fibrosis, the physical examination reveals fine bibasilar inspiratory crackles (Velcro crackles). Additionally, digital clubbing is seen in 25-50% of patients with IPF.

A 3-year-old girl presents with a persistent cough. A few weeks ago, she had a low-grade fever with rhinorrhea and cough for several days. The rhinorrhea and fever resolved, but the cough has continued. It bothers her primarily during the night and when she wakes in the morning. Her medical history is significant only for mild seasonal allergies that appeared right after she turned 3. Her vaccines are current, and her parents deny recent travel. She is normally a thriving, happy child who attends preschool 3 days a week. For the past few weeks, she has been tired and fussy during the day because the cough keeps her up at night. On exam, she appears well. She is in no respiratory distress, and her lungs are clear to auscultation. Question What intervention will most likely cut down on the coughing and improve the girl's sleep? Answer Choices 1 Give diphenhydramine before bed 2 Encourage extra fluids and humidifier for the room 3 Discourage milk consumption late in the day 4 Give Robitussin as needed 5 Try an MDI with bronchodilator

Try an MDI with Bronchodilator This girl's symptoms and history of allergic rhinitis are consistent with cough-variant asthma. Respiratory symptoms are characteristically worse at night and often triggered by respiratory infections. A short-acting bronchodilator before bed and in the morning may help ease the cough and allow her to sleep better. Her primary cold symptoms have resolved, so diphenhydramine, Robitussin, and cutting down on milk are unnecessary. They are ineffective in treating a cough caused by asthma. Fluids and humidification are always a good recommendation, but in this case they will not directly cut down on the cough as much as a bronchodilator.

A 1-year-old female is having a 2-day history of fever (102 0 F oral), rhinorrhea, and dry cough, with a decreased appetite. The mother states that her daughter has been less active, and her fluid intake has decreased for her age. On exam, the child is non-toxic appearing, has a rectal temperature of 100.2 0 F, and has nasal flaring and a respiratory rate of 45, rhinorrhea, moist mucous membranes, and a minimal wheeze heard bilaterally. Her chest x-ray has no specific findings. Based on these findings, what is the initial ancillary test to confirm the diagnosis? A Acid fast bath test B Viral nasal washings C Sputum culture and sensitivity D Blood cultures E Throat culture

Viral Nasal Washings B Viral nasal washings are the best choice for determining RSV infection that causes bronchiolitis. Sputum and blood cultures do not grow the agents that would typically cause the infection to occur, and you are obtaining a sample from a different part of the respiratory tract and blood.

A 17 year-old male notes acute onset of dyspnea, excessive non-productive coughing, "wheezing" and upper chest tightness when running. Symptoms usually resolve in a few minutes and he is able to resume running. He is prescribed albuterol with no improvement. Which of the following is the most likely diagnosis in the patient? A Angioedema B Bronchiectasis C Cystic fibrosis D GERD E Vocal Cord Dysfunction

Vocal Cord Dysfunction E Vocal cord dysfunction (E) is commonly misdiagnosed as asthma or is a comorbidity in patients with asthma. The history of acute onset and rapid disappearance of symptoms is consistent with vocal cord dysfunction, and not typical of cystic fibrosis (C), or GERD (D). The presence of urticarial or swelling of the face or lips would support the life-threatening diagnosis of angioedema (A). Bronchiectasis (B) typically presents with a chronic productive cough and additional pulmonary symptoms based on the underlying cause and severity.

What is the most common pulmonary complaint of a patient with tuberculosis? A Hemoptysis B Chronic cough C Wheezing D Dyspnea E Chest pain

chronic cough Chronic cough is the most common complaint. Hemoptysis can occur but is rare, and a productive cough becomes more prevalent after the chronic cough has been established for a while.

A 60-year-old man with a history of recurrent sinus infections presents with hemoptysis and hematuria. Physical examination shows a temperature of 101 degrees Fahrenheit, a blood pressure of 145/85 mmHg, decreased breath sounds on his right lower lobe, and palpable purpura on his bilateral lower legs. No warm or swollen joints were noted. Anti-neutrophil cytoplasmic antibody Positive Anti-double-stranded DNA Negative Anti-Glomerular Basement Antibody Negative Erythrocyte sedimentation rate 50mm/hour Hemoglobin 11 g/dL Serum creatinine 1.5 mg/dl Serum potassium 5 meq/l Serum chloride 21 meq/l Urinalysis 1.020, pH 6, numerous red cells, 500 mg albumin/g creatinine, red blood cell casts Chest x-ray Right lower lobe effusion Question What is the most likely diagnosis? Answer Choices 1 Systemic lupus erythematosus 2 Wegener's Granulomatosis 3 Rheumatoid arthritis 4 Goodpasture's syndrome 5 Henoch-Schönlein Purpura (HSP)

Wegener's Granulomatosis Wegener's Granulomatosis is a small vessel vasculitis of unknown pathogenesis that causes inflammation and necrotizing granulomas in the upper and lower respiratory tract and kidneys, as well as rash; 15 - 50% of patients have cutaneous involvement (Brenner, chapter 31). Palpable reddish-purple lesions on the lower extremities should raise suspicion of vasculitis; macules and nodules, may also erupt. Some 50 - 95% of patients with Wegener's will eventually have renal involvement (Brenner), which may range from mild findings to fulminant dialysis-requiring nephritis. On renal biopsy, focal segmental necrotizing glomerulonephritis is noted; crescents may be seen. Immune staining is generally negative. Treatment involves immunosuppressive therapy, as is the case with SLE, and dialysis as needed. In some resistant cases, plasmapheresis may be helpful. Patients with Wegener's are often Anti-Neutrophil Cytoplasmic Antibody (ANCA) positive. Patients with Goodpasture's syndrome typically present with a triad of glomerulonephritis, pulmonary hemorrhage, and anti-glomerular basement membrane (GBM), type IV collagen antibodies. Many patients with Goodpasture's syndrome are ANCA positive. SLEis a systemic inflammatory disease that causes a variety of signs and symptoms affecting many organs. Renal involvement is frequent in SLE and may include a variety of glomerular diseases, causing hematuria, proteinuria, and urinary casts. Henoch-Schönlein Purpura (HSP) is a vasculitic syndrome that is seen more often in children than adults; it may cause petechiae, purpura, gastrointestinal bleeding, and hematuria/glomerulonephritis. Complement counts and anti-double-stranded DNA are normal, and elevated IgA antibody levels may be seen. Rheumatoid arthritis (RA) is an inflammatory arthritis that causes joint stiffness and swelling that takes a long time to loosen up after prolonged sitting or sleeping. Symmetrically swollen joints and rheumatoid factor may be noted, but vasculitis is uncommon in RA (Harrison's chapter 314).

An immigrant worker presents with a chronic cough, hemoptysis, and night sweats. Physical exam reveals a wasted, middle-aged man with bronchial breath sounds in the right upper lobe. The patient is asked for 2 sputum samples; 1 is sent for culture, and the other is directly examined under a microscope. What technique will be the best choice for microscopic examination for rapid initial diagnosis? Answer Choices 1 Gram stain 2 Giemsa stain 3 Ziehl-Neelsen stain 4 Direct immunofluorescence 5 Darkfield examination

Ziehl-Neelsen stain This patient has the clinical features of pulmonary tuberculosis; in the United States, TB is common in immigrants from developing countries. For rapid diagnosis of pulmonary TB, direct sputum examination with Ziehl-Neelsen stain is the best choice. The culture should be sent at the same time to identify the type of Mycobacterium and the sensitivity to the drugs. Mycobacterium are aerobic, acid-fast bacilli. They are neither Gram-positive nor Gram-negative, but stain red with Ziehl-Neelsen stain because of the N-glycolyl muramic (mycolic) acid in their cell walls. The lipid content of the cell wall is very high; therefore, Mycobacterium is hydrophobic and impermeable to basic aniline bacteriologic stains. Gram staining is the most common stain used for most of the bacteria, but it is not used for Mycobacterium. Giemsa stain is used for tissue, blood, or secretions to see the microorganisms. It is commonly used for Neisseria species, malaria, and other blood parasites. Direct immunofluorescence is useful in the identification of many microorganisms such as Bordetella pertussis and Chlamydia trachomatis. Fluorescein-labeled monoclonal or polyclonal antibodies against specific microbial antigens are added to a tissue or cell suspension; this enables it to detect a specific microorganism. Darkfield examination uses a microscope that illuminates the object over a dark background. It is used extensively to identify spirochetes, which are too thin to be seen easily by staining.

A 60-year-old woman presents with a history of persistent cough. She is confined to her bed; walking over 10 paces causes severe breathlessness. She has no energy to carry out any of her regular activities. She has never smoked, and she drinks the occasional glass of wine. On physical examination, she is found to have decreased breath sounds and dullness to percussion over her right lower thorax. Further evaluation reveals an irregular mass in the periphery of the right lung base with a right sided pleural effusion. A needle is inserted into the pleural space and divulges blood stained fluid. Question If results prove to be a malignancy, what is the most likely sub-type considering she has never smoked? Answer Choices 1 Small cell carcinoma 2 Large cell carcinoma 3 Adenocarcinoma 4 Mesothelioma 5 Squamous cell carcinoma

adenocarcinoma Adenocarcinoma is the correct answer. Adenocarcinoma accounts for about 40% of all lung cancers. While smoking is a strong risk factor, it is the most common lung malignancy in non-smokers. It is seen more commonly in women, and it starts typically at an earlier age. Formerly known as bronchoalveolar carcinoma, adenocarcinoma sometimes produces mucus or other such substances. The peripheral location is also typical, as compared to the more central squamous cell carcinoma. Small cell carcinoma is incorrect. Also known as oat cell carcinoma, it is extremely unusual in non-smokers, and it accounts for about 10% to 15% of lung cancers. They are typically central in location, and they respond poorly to treatment. Large cell carcinoma is incorrect. It accounts for about 10% to 15% of lung cancers. It can occur in any part of the lung, and it tends to grow and spread rapidly. Being more uncommon than adenocarcinoma, in this patient, the likelihood of it being a large cell tumor is lower. Mesothelioma is incorrect. Mesothelioma is a pleural malignancy associated strongly with asbestos exposure. They typically present as pleural effusions with pleural plaques. Smoking is not strongly associated with mesothelioma. Squamous cell carcinoma is incorrect. It accounts for about 25 to 30% of all cases of lung cancer, and it is strongly associated with smoking. Squamous cell carcinoma tends to be more central in location and is most commonly associated with para-neoplastic syndromes.

A 73-year-old woman presents to you for the first time for an initial history and physical. She states that over the past year she has been hospitalized twice for pneumonia that required mechanical ventilation for 2-3 weeks each episode and bacteremia requiring several weeks of antibiotics. Which of the following nail abnormalities would you most likely expect to see? Answer Choices 1 Paronychia 2 Beau's lines 3 Pitting 4 Onycholysis 5 Splinter hemorrhages

beau's lines Explanation The correct answer is Beau's lines as this condition is typically seen in patients with a history of severe, systemic illness such as the example given. Paronychia is a bacterial infection of the proximal and lateral nail folds that causes pain and inflammation, typically with pus formation. Pitting is seen in patients with a history of psoriasis and alopecia areata. Onycholysis is the separation of the nail plate from the nail bed and is more commonly seen in women with long nails. Splinter hemorrhages are a characteristic sign in patients with infectious endocarditis.

A neonate presents with meconium ileus that is successfully unobstructed. The infant returns at her 4-month appointment with signs of failure to thrive. Which of the following is the most likely diagnosis for this patient? A cystic fibrosis B Wilson disease C intussusception D volvulus

cystic fibrosis Cystic fibrosis (CF) is a major cause of gastrointestinal and pulmonary morbidity in children due to mutations in the CF genes. The mutations lead to a deficiency in cystic fibrosis transmembrane conductance regulator protein that controls movement of salt and water into and out of epithelial cells and results in production of abnormally thick mucus. About 15% of patients with CF present with meconium ileus at birth. This is typically treated with enema for disimpaction and rarely surgery. Approximately half of the infants with CF will present with failure to thrive, which is diagnosed by lack of growth for 2 consecutive months in patients younger than 6 months of age. They may also present with respiratory compromise. However, not all patients present in childhood. Diagnosis of CF is confirmed by a sweat chloride level above 60 meq/L or with genetic testing. Treatment for patients with CF is mainly symptomatic therapy for obstructions of the digestive and respiratory tract. In addition, there is pancreatic enzyme supplementation to aid in digestion and vitamin and calorie supplementation for deficiencies in the diet. Gene therapy is now being looked at for future treatment. Intussusception (telescoping of the small intestine) typically presents in an infant with paroxysmal abdominal pain, vomiting, and diarrhea that may progress into bloody stools. Volvulus is normally the result of intestinal malrotation that causes occlusion of the superior mesenteric artery and eventual bowel necrosis. Infants typically present within 3 weeks of life with bile-stained vomiting and bowel obstruction. Wilson's disease is the defect in the ability to excrete copper in the bile that results in accumulation of copper in the liver.

A 62-year-old male diabetic patient presents to your family practice office with a fever and cough, and after a physical examination you perform a chest x-ray, which reveals a right middle lobe pneumonia. You prescribe levofloxacin 750 mg PO for five days and give the patient his first dose in the office prior to sending him home. Within five minutes, he develops generalized urticaria, shortness of breath with a pronounced wheeze, and becomes hypotensive. While your staff calls 911, what is the most important agent to administer first? A cimetidine B dipenhydramine C epinephrine D methylprednisolone E none of the above

epinephrine Early recognition of an anaphylactic reaction is mandatory, since death occurs within minutes to hours after the first symptoms. Mild symptoms such as pruritus and urticaria can be controlled by administration of 0.3 to 0.5 mL of 1:1000 (1.0 mg/mL) epinephrine SC or IM, with repeated doses as required at 5- to 20-minute intervals for a severe reaction.

A 9-year-old girl presents with a sore throat. The mother states that she began to run a fever a few days ago, and she complained that her throat hurt. On physical exam, you note a red throat, a red, beefy tongue, tonsillar exudates, and swollen anterior cervical lymph nodes. You order a rapid strep test which comes back positive. It is noted in the patient's records that she has had a severe anaphylactic reaction to penicillin. What antibiotic would treat this infection while minimizing the risk of invoking an allergic reaction? Answer Choices 1 Augmentin 2 Cephalexin (Keflex) 3 Ciprofloxacin 4 Mupirocin (Bactroban) 5 Erythromycin

erythromycin The clinical picture is suggestive of a streptococcal bacterial infection. Penicillins are the 1st-line antibiotics in the treatment of strep pharyngitis. Since the patient is allergic to penicillins, erythromycin is an effective alternative and has no allergic cross-reactivity with the penicillins. Augmentin contains amoxicillin, a member of the penicillin family with allergic cross-reactivity; it is contraindicated for patients allergic to penicillin. Cephalexin, a cephalosporin, can be used to treat strep throat. However, approximately 7% of patients who are allergic to penicillin are also allergic to cephalosporins. Therefore, compared to erythromycin, use of this agent poses a slightly higher risk of causing an allergic reaction in this patient. Ciprofloxacin is effective against Gram-negative organisms. Since streptococcal species are Gram-positive, it would not be an appropriate treatment in this scenario. Mupirocin is a topical antibiotic and is not indicated in the treatment of strep throat.

A 61-year-old male presents with acute onset dyspnea, cough with thick, blood tinged sputum, and fever. He is a known, longstanding alcoholic and has Type II Diabetes, for which he takes no medication. A chest x-ray reveals a right upper lobe infiltrate with early fibrotic changes. Which of the following is an appropriate first-line medication(s) for the most likely etiologic organism? A Macrolide B Doxycycline C Second generation cephalosporin D Beta-lactam plus fluoroquinolone E Fluoroquinolone

fluorquinolone Klebsiella pneumoniae, a gram negative bacteria, is associated with pneumonia in immunocompromised patients, especially alcoholics. Community-acquired pneumonia due to Klebsiellae is associated with a broad range of presentations. This may range from mild acute bronchitis or bronchopneumonia to acute onset of high fevers, chills, and cough with blood tinged, thick sputum (often referred to as "currant jelly sputum"), and aggressive chest x-ray findings, such as cavitation and empyema. Community-acquired pneumonia empiric treatment is based upon the likely causative organisms within patient subsets. This includes appropriate selections for individuals able to be handled as outpatients, but with significant past medical history and risk factors for drug-resistant bacteria, such as alcoholism. The guidelines currently indicate that these patients should be treated with a beta-lactam plus a macrolide or doxycycline, or a fluoroquinolone, or amoxillin-clavulanate. The guidelines also include treatment recommendations for other patient subsets, and alternative medications to consider (which can be viewed in Table 251-4).

A man presents with pulmonary edema, and he is treated with intravenous furosemide and oxygen. Clinical improvement is seen. He is also taking levothyroxine (Synthroid), acyclovir, triamcinolone acetonide (Nasacort), and azelastine hydrochloride (Astelin) spray. The pulmonary congestive symptoms are alleviated, and there is a significant increase in urine output. What medicine contributed most to the client's improvement as well as the significant increase in urine output? Answer Choices 1 Levothyroxine (Synthroid) 2 Acyclovir 3 Triamcinolone acetonide (Nasacort) 4 Azelastine Hydrochloride (Astelin) 5 Furosemide

furosemide The early effect of furosemide is thought to be due to direct pulmonary venous dilation. It may also redistribute pulmonary blood flow away from fluid-filled alveoli to well-aerated alveoli. This improves blood oxygen saturation and alleviates symptoms of congestive heart failure. Taken orally, the drug has an onset of action with 1 hour, and its diuretic effect lasts for 6 to 8 hours. Administered intravenously, it has a diuretic effect beginning within minutes and lasting approximately 2 hours. Levothyroxine (Synthroid) is used for hypothyroidism. Hypothyroidism is a slowing of the thyroid gland. Symptoms can be dry hair, dry skin, and puffiness of the face or hands and feet. The client can also suffer from depression, fatigue, weight gain or loss, and not sleeping. Triamcinolone Acetonide (Nasacort) is a nasal spray often used for allergies or chronic sinusitis. It is a steroid, and it works by shrinking the mucous membranes. Azelastine hydrochloride (Astelin) is a nasal spray used to shrink the mucous membranes, enabling the sinus to drain, or for seasonal allergies. Azelastine hydrochloride exhibits histamine H1-receptor antagonist activity in isolated tissues. Acyclovir is an antiviral medicine often used to treat the symptoms of herpes (both type 1 and type 2), herpes zoster (shingles), and chicken pox (varicella).

A 50-year-old obese woman underwent a cholecystectomy and T-tube drainage of the common bile duct. On the 7th postoperative day, she develops sudden epigastric and left-sided chest pain. She is short of breath and is sweating profusely. A review of the chart shows that her temperature is between 99 F and 100 F for the last 2 days. The patient's vitals: pulse 90/min, BP 110/60 mm Hg, and RR 24/min. The ECG reading shows tachycardia, and the ventilation-perfusion lung scan shows a VQ mismatch. Question What is the initial management of this patient? Answer Choices 1 Pulmonary embolectomy 2 Urokinase therapy 3 Exploratory laparotomy and drainage of abscess 4 Heparinization 5 Inferior vena caval filter

heparinization The diagnosis is pulmonary embolism, which should always be suspected if the patient suddenly collapses 1 - 2 weeks after surgery. Pulmonary embolism develops due to blockage of blood flow to the lungs caused by venous thrombi from deep vein thrombosis passing into the pulmonary circulation. It can be prevented by mobilizing the patient early following surgery and considering prophylaxis with heparin. Classically, it presents around 10 days post-op with collapse and sudden breathlessness while straining at stool; however, it may occur after any period of immobility or with no predisposing factors. Other features include pleuritic pain, dyspnea, shock, tachypnea, cyanosis, hemoptysis, gallop rhythm, pleural rub, right ventricular S3 gallop, and loud P2. Chest X-ray is often normal and sometimes with decreased vascular markings. Diagnosis is usually made clinically and confirmed by ventilation-perfusion mismatch on lung VQ (ventilation-perfusion) scan. An isotope lung scan using the macroaggregated albumin followed by ventilation scan using 133 Xenon can provide complementary evidence that the perfusion problem is due to pulmonary emboli. Initial management of this patient is adequate heparinization. Most patients with pulmonary emboli would resolve on adequate anticoagulation. The initial dose would be given intravenously and could be up to 10,000 units. Pulmonary embolectomy is indicated in only a very small group of patients who have massive embolus and are in a critical state. Exploratory laparotomy would be fruitless, as there will be no pathology inside the abdomen because the cause of this incident lies in insufficient physical mobilization after the operation. Treatment with streptokinase or urokinase is not recommended in this postoperative patient. Vena cava filter is indicated in patients who continue to throw emboli in spite of adequate anticoagulation or in whom heparinization is not advisable.

A 72-year-old man is evaluated following admission to the hospital for a 1-year history of progressive dyspnea, nonproductive cough, weight loss, low-grade fevers, fatigue, and myalgias. His past medical history is remarkable for atrial fibrillation (for which he takes amiodarone), hypercholesterolemia, and recurrent urinary tract infections, for which his urologist prescribed nitrofurantoin on a chronic, prophylactic basis. He denies cigarette use, a history of murmurs or coronary artery disease, chills, fatigue, rhinitis, otalgia, chest pain, wheezing, hemoptysis, syncope, abdominal pain, rashes, peripheral edema, diaphoresis, arthralgias, vomiting, and urinary complaints. A bedside echocardiogram and electrocardiograms are unremarkable for abnormalities; a chest X-ray revealed peripheral reticular opacities at the lung bases and a generalized honeycombing pattern. Question What is the most likely diagnosis? Answer Choices 1 Idiopathic pulmonary fibrosis 2 Pulmonary embolism 3 Sarcoidosis 4 Goodpasture's syndrome 5 Wegener's granulomatosis

idiopathic pulmonary fibrosis This patient's most likely diagnosis is idiopathic pulmonary fibrosis. Amiodarone, bleomycin, and nitrofurantoin are notable medications associated with pulmonary fibrosis. Most patients with idiopathic pulmonary fibrosis present with a gradual onset, which is often greater than 6 months. The clinical symptoms of idiopathic pulmonary fibrosis are nonspecific; symptoms often precede the diagnosis by a median of 1 - 2 years. Most patients present with exertional dyspnea and a nonproductive cough. Associated constitutional symptoms are uncommon. The physical examination reveals fine bibasilar inspiratory crackles (Velcro crackles) and digital clubbing in 25 - 50% of cases. Typical chest X-ray findings include peripheral reticular opacities (netlike linear and curvilinear densities) that are predominantly located at the lung bases, honeycombing (coarse reticular pattern), and lower lobe volume loss. The diagnosis of pulmonary embolism almost always occurs with underlying predisposing conditions present; venous thrombosis may result from a generalized hypercoagulable state, venous endothelial injury, or local stasis (Virchow triad). Most commonly, the initial manifestations of pulmonary embolism include an abrupt dyspnea and chest pain. Tachycardia and hypoxia are the most common clinical signs. Associated manifestations include fever, hypotension, cyanosis, pleural friction rub, and findings consistent with pulmonary consolidation. ECG most often shows tachycardia and various ST-T wave abnormalities, which are not specific for PE. Common chest radiographic abnormalities include atelectasis, pleural effusion, parenchymal opacities, and elevation of a hemidiaphragm. The classic radiographic findings of pulmonary infarction include a wedge-shaped, pleura-based triangular opacity with an apex pointing toward the hilus (Hampton hump) or decreased vascularity (Westermark sign). Sarcoidosis is a multisystem inflammatory disease of unknown etiology that manifests as non-caseating granulomas, predominantly in the lungs and intrathoracic lymph nodes. The presentation commonly includes systemic complaints of fever, anorexia, and arthralgias, as well as pulmonary complaints such as dyspnea on exertion, cough, chest pain, and (rarely) hemoptysis. Extrapulmonary findings are common, and include erythema nodosum, lower-extremity panniculitis, lupus pernio, a facial violaceous rash, maculopapular plaques, granulomatous uveitis, conjunctival lesions, scleral plaques, cardiomyopathy, and cranial nerve palsies. The chest X-ray commonly reveals air trapping, lymphadenopathy, and infiltrates. Goodpasture's disease is a condition of glomerulonephritis, with or without pulmonary hemorrhage, and the presence of circulating anti-glomerular basement membrane (anti-GBM) antibodies. Constitutional symptoms, such as malaise, chills and fever, and/or arthralgias, may precede or be concurrent with pulmonary or renal manifestations. Hemoptysis, cough, dyspnea, and shortness of breath describe pulmonary involvement, while hematuria, edema, high blood pressure, and uremia signify renal affliction. Significant anemia and chest pain may also occur. Wegener granulomatosis is a rare multisystem autoimmune disease of unknown etiology; its hallmark features include necrotizing granulomatous inflammation and pauci-immune vasculitis in small- and medium-sized blood vessels. A wide spectrum of extrapulmonary manifestations, which include recurrent respiratory infection in adults and upper and lower respiratory tract problems in children, is expected. Manifestations include constitutional complaints, conjunctivitis, episcleritis, uveitis, optic nerve vasculitis, retinal artery occlusion, nasolacrimal duct occlusion, proptosis, chronic sinusitis, epistaxis, saddle nose deformity, serous otitis media, hearing loss, strawberry gingival hyperplasia, stridor, myalgias, arthritis, arthralgias, glomerulonephritis, and renal failure. Later-onset findings reveal neuropathic and cranial nerve abnormalities. Other manifestations include CNS small- to medium-sized vessel vasculitis, a palpable purpura, splanchnic vasculitis, myocardial infarction, and/or cardiac friction rubs.

Patients suffering from asthma, no matter the classification, should be prescribed what medication? Answer Choices 1 Inhaled corticosteroid 2 Inhaled long-acting β2-agonist 3 Leukotriene receptor antagonist 4 Inhaled short-acting β2-agonist 5 Omalizumab

inhaled short acting B2 agonist Inhaled short-acting β2-agonist (SABA) is the appropriate answer. SABA is considered a quick-relief medication that acts by relieving and reversing acute airflow obstruction. Whether the patient is diagnosed with intermittent or persistent asthma, SABA is the drug chosen to be used for acute symptomatic periods and is typically instructed to be used on an as needed basis. Patients are explicitly educated on the "Rule of 2's" in relation to the use of SABA: if using SABA >2 days a week, if waking up at night due to asthma or cough more than 2 times in a month, or if having to refill their prescribed SABA more than 2 times a year, this is highly suggest that there is inadequate control, and a step up treatment plan needs to be made in terms of asthma management. SABA can be administered via an inhaler or nebulizer machine. Inhaled corticosteroids, long-acting β2-agonists, and Leukotriene receptor antagonists are used in varying degrees, dosages, and combinations for long-term control (maintenance) of asthma. Maintenance medications are taken daily despite whether the patient is symptomatic or not. Depending on the medication, the route of administration could be via inhalation or an oral tablet. Omalizumab is a subcutaneous injection indicated for patients with moderate to severe persistent allergic asthma caused by year-round airborne allergens. This injection is given every 2-4 weeks in the provider's office.

A 32-year-old man presents due to occasional shortness of breath and associated cough, especially when he is working outside. He has associated chest tightness, which resolves within minutes when he sits down and rests. These symptoms occur 1-2 days a month. He is otherwise healthy and does not smoke. Blood pressure is 128/74; pulse is 76; respiration is 14; and pulse oximetry is 100% on room air. His FEV1 is 96%. Question What is the treatment of choice? Answer Choices 1 Inhaled corticosteroid 2 Inhaled short acting beta agonist 3 Inhaled long acting beta agonist 4 Inhaled anticholingeric 5 Oral beta agonist

inhaled short acting beat agonist This patient has intermittent asthma, which is defined as symptoms ≤ 2days per week, nighttime awakenings ≤ 2 times a month, and no interference with normal activities with a normal FEV1 between exacerbations. The treatment of choice is inhaled short acting beta agonist (SABA), albuterol, when needed. If the asthma occurs due to an allergen or in a predictable manner, the patient can use the SABA prior to the exposure and avoid triggers as much as possible. A patient with persistent asthma requires treatment with an inhaled corticosteroid as a controller medication. The dose (low, medium, or high) depends on the severity of the asthma and the level of asthma control. A long-acting beta agonist may be added for controller medication. The patient would continue to use a SABA as needed for acute symptoms. Inhaled anticholinergic agents are used to treat chronic obstructive disease (COPD), but may be used as an adjunctive therapy to SABA for acute asthma exacerbations. Oral beta agonists have a delayed therapeutic response and are not useful for acute attacks. Inhaled SABA is more effective.

A 68-year-old man presents to your pulmonology practice for long-standing dyspnea and non-productive cough. The patient has had the cough and progressively worsening dyspnea for about 1.5 years. He denies other symptoms, and he is not a smoker. He denies fevers, chills, and night sweats. He denies unusual travel, hobbies, and occupational exposures. The rest of his review of systems is negative. His past medical history is unremarkable. He had a workup through his family practice, and he was then sent to a cardiologist, who ruled out cardiovascular causes of his dyspnea. He was given trials of various antibiotics, inhalers, and steroids; there was no improvement in symptoms, despite good compliance. Several screening blood tests (including blood count, metabolic panel, HIV, and autoimmune markers) are done, revealing no abnormalities. A chest X-ray shows a few reticular opacities. A high-resolution computed tomography (HSCT) scan shows some scattered areas of reticular opacities and honeycombing. Your office runs a pulmonary function test (PFT), which shows some restrictive impairment and reduced perfusion of carbon monoxide. Physical exam is significant for fine inspiratory bibasilar crackles and clubbing in the fingers. An occasional dry cough is noted. Question What intervention is recommended for this patient's condition? Answer Choices 1 Avoidance of exertion 2 Daily inhaled steroids 3 Long-term oral steroid use 4 Lung transplant 5 Ventilation-perfusion scan

lung transplant This patient is presenting with idiopathic pulmonary fibrosis (IPF), and the only intervention currently associated with improvements in mortality is lung transplant. IPF is a chronic, progressive restrictive pulmonary disease of the lung parenchyma. IPF presents with exertional dyspnea and non-productive cough; it also presents with the imaging characteristics described in this patient. Avoidance of exertion may prevent the exertional dyspnea in this patient, but it is not recommended. Respiratory therapy to improve exercise tolerance is recommended in cases of IPF. Daily inhaled steroids and long-term oral steroid use are not proven as effective in IPF. The pathophysiologic basis for IPF is much more a disorder of fibroblastic disease, in which there is an abnormal accumulation of extracellular matrix, and not inflammatory in nature. Anti-inflammatory medications do not alter the course of IPF. A ventilation-perfusion scan is a test that is primarily used in the diagnosis of pulmonary embolism (PE). This patient is not presenting with a history consistent with a PE (which may be acute dyspnea and chest pain). The imaging that has already been done (high-resolution CT) is sufficient for diagnosis of IPF. No further studies are recommended. If the provider needs further confirmation, a lung biopsy should be the confirmatory test of choice.

A 72-year-old man presents with longstanding but increasing dyspnea, and a 3 day history of shortness of breath, coughing, unilateral sharp chest pain and mild temperature elevation. Pain is worse when he takes a deep inspiration and when he coughs. PMH is significant for a smoking history of 2 packs/day for 40 years equating to 80 pack years. The patient has other co-morbidities including CAD, COPD, CHF, hypertension and dyslipidemia. He denies recent travel, sick contacts, occupational exposure, drug or alcohol use. Vital signs are BP 190/96, HR 140, RR 48, 02 68% on room air. Chest examination of the left posterior chest reveals a dull percussion note, inaudible bronchovesicular breath sounds, decreased tactile fremitus, a pleural friction rub on inspiration and diminished voice sounds below the 6th intercostal space. Significant bilateral lower extremity edema is also noted. A chest x-ray reveals blunting of the costophrenic angle. Question What is the most likely diagnosis? Answer Choices 1 Pneumothorax 2 Pulmonary thromboembolism 3 Pulmonary fibrosis 4 Pleural effusion 5 Bronchiectasis

pleural effusion The signs and symptoms of the patient are consistent with pleural effusion. The key finding is blunting of the costophrenic angle on chest x-ray. Fluid in the pleural space pushes the lung upward resulting in a blunting of the costophrenic angle that is normally seen as a sharply pointed downward indentation. Results of the chest examination show decreased or absent breath sounds, dullness to percussion over affected area, decreased tactile fremitus, and friction rub. The most common cause of pleural effusion is left-ventricular failure. Pneumothorax is characterized by hyperresonance of percussion note, JVD due to compression of the superior vena cava, falling 02 saturation, and hypotension. Chest x-ray shows a visceral pleural line and/or lung retraction from the chest wall. Pulmonary thromboembolism is characterized by sudden onset dyspnea, pleuritic chest pain, hemoptysis, tachypnea, unilateral lower extremity edema, hypoxia and hypocarbia. Pulmonary fibrosis is associated with exertional dyspnea, non-productive cough and crackles heard on inspiration. CT scan shows patchy opacities at lung bases with honeycombing. Bronchiectasis is associated with dilation of the bronchi. Patients present with productive cough with yellow or green sputum, dyspnea, hemoptysis. Exam reveals rales, rhonchi, wheezes, purulent mucus. Chest x-ray reveals increased bronchovascular markings; tram lines and areas of honeycombing.

A 45-year-old man presents with a 2-day history of sharp left-sided chest pain. It is aggravated by taking a deep breath. He denies any trauma to the chest. On examination, he is febrile, dyspneic, and has rales on auscultation. Question What is the most likely cause of his chest pain? Answer Choices 1 Esophageal reflux 2 Pneumonia 3 Aortic dissection 4 Pulmonary embolism 5 Unstable angina

pneumonia Pneumonia pain is usually described as a sharp pain that is aggravated by inspiration. Other signs and symptoms of pneumonia, such as cough, fever, dyspnea, and rales on auscultation, may also be present. Esophageal reflux pain is a substernal pain that is usually described as burning. It may last for up to 1 hour. It is relieved by antacids and may be aggravated by alcohol and postprandial recumbency. Aortic dissection pain is of abrupt onset that is usually described as severe and tearing. It may radiate from the anterior chest wall posteriorly to the interscapular region. On examination, the blood pressure may be elevated, and a pericardial rub and the diastolic murmur of aortic insufficiency radiating along the right sternal border may be auscultated. Pulmonary embolism pain is usually of abrupt onset and pleuritic in nature. It has a duration of several minutes to a few hours. There may be predisposing factors, such as deep venous thrombosis or long periods of immobilization. Patients may also report hemoptysis. On examination, the patients are dyspneic and tachypneic with tachycardia. Unstable angina pain is usually described as tightness or pressure on the chest and lasts for 10 -20 minutes. It is usually left-sided or retrosternal, and it may radiate to the jaw and neck. It often occurs at rest, but it may be precipitated by physical exertion and emotional stress. On examination, a systolic murmur of mitral regurgitation may be auscultated during the pain.

A 17-year-old male arrives comatose in the emergency department where you are on duty. There is no history on your patient and no visible signs of trauma. You draw blood for gases. If the blood gas results are pH 7.6, H+ 38 nEq/L, PCO2 32 mmHg, and HCO3- 23 mEq/L, what acid-base disturbance does your patient have? Answer Choices 1 Respiratory acidosis 2 Respiratory alkalosis 3 Metabolic acidosis 4 Metabolic alkalosis 5 No disturbance, he is normal

respiratory alkalosis With respiratory acidosis, his blood pH should be below 7.4 and his pCO2 should be greater than 40 mmHg. Rarely does respiratory alkalosis occur. The underlying mechanism is increased respiration decreases CO2 in the body. Respiratory alkalosis can occur if an individual ascends to a high altitude and, in an effort to increase O2 levels in the blood, the respiratory rate increases. With metabolic acidosis, his blood pH should be below 7.4. While the patient is alkalotic, you would expect his pCO2 to increase as his respiratory system compensates for metabolic alkalosis. Normal pH is roughly 7.4.

A 50-year-old man with no significant past medical history presents for a follow-up. He notes that, over the past week, he has developed increased nonproductive cough and shortness of breath. He has had a chronically-progressive cough and shortness of breath upon exertion over the past year. His occupational history reveals coal mining for 25 years. He denies any smoking, alcohol consumption, travel history, or sick contacts. He also denies fever, chills, sore throat, otalgia, chest or abdominal pain, peripheral edema, rashes, or pruritus. A chest radiograph was performed which revealed the following imaging. Question What treatment would be most beneficial in this case? Answer Choices 1 Trexall (Methotrexate) 2 Oseltamivir (Tamiflu) 3 Amoxicillin-clavulanate (Augmentin) 4 Prednisone (Deltasone) 5 Acyclovir (Zovirax)

prednisone The correct response is prednisone (Deltasone). This patient's most likely diagnosis is silicosis. His acute manifestations signify the development of acute silicosis on top of chronic silicosis. The X-ray demonstrates miliary mottling with perihilar and mediastinal lymphadenopathy showing 'egg shell calcification'. Corticosteroids may be of benefit in acute silicosis. In chronic silicosis, they are unlikely to be of any benefit, although pulmonary function improvement was noted in 1 study. This patient is not experiencing a bacterial or viral infection; therefore, antibiotics (Augmentin) or antiviral agents (Tamiflu and Zovirax) are inappropriate. Methotrexate is indicated in the treatment of ectopic pregnancy, autoimmune disorders (such as psoriasis and rheumatoid arthritis), as second-line therapy in sarcoidosis, and several malignancies.

A 28-year-old Caucasian woman presents with worsening shortness of breath with activity. The patient states that, over the last 4 weeks, it has become increasingly difficulty for her to walk up a flight of stairs without stopping after a few steps to rest. Currently, she cannot even walk across a room without feeling out of breath. She admits to significantly increased fatigue and dizziness as well as shortness of breath. The patient has a history of systemic lupus erythematosus; she was diagnosed with the condition about 2 years ago. Physical examination reveals a young woman who appears to be her stated age, but she is in a mild amount of distress due to being short of breath while just sitting on the examination table. Accessory muscle use is present. The patient's skin has a faint bluish hue to it; her lips are also a shade of blue. Ankles reveal 2+ pitting edema. Question What is the most likely diagnosis? Answer Choices 1 Pulmonary embolism 2 Pulmonary hypertension 3 Congestive heart failure 4 Dilated cardiomyopathy 5 Pericarditis

pulmonary htn The patient above is most likely suffering from restrictive lung diseasedue to the lupus, and specifically signs and symptoms of pulmonary hypertension. Pulmonary hypertension is a lung disorder the pulmonary arteries become narrowed. As a result, the blood pressure in these arteries rises above normal levels. The high pressure strains the right ventricle of the heart, causing it to expand in size. Overworked and enlarged, the right ventricle gradually becomes weaker and loses its ability to pump enough blood to the lungs. This can lead to the development of right heart failure. Symptoms of pulmonary hypertension do not usually occur until the condition has progressed. The initial symptom is usually shortness of breath with everyday activities, such as climbing stairs. Fatigue, dizziness, and fainting spells also can be symptoms. Edema in the ankles, abdomen, or legs may be present; bluish lips may also be present. Chest pain may occur as strain on the heart increases. Causes of pulmonary hypertension may include, but are not limited, to dexfenfluramine and phentermine (fen-phen has been taken off the market, but former fen-phen users have a 23-fold increase risk of developing pulmonary hypertension, possibly years later), liver diseases, rheumatic disorders (which include systemic lupus erythematosus), other lung conditions, heart diseases, and low-oxygen conditions; it may be idiopathic in origin. Congestive heart failure is usually a disease state that initially affects the left side of the heart, causing signs and symptoms relating to that. Pulmonary embolism is incorrect; it usually presents with dyspnea and pain on exertion, possible tachypnea, and (potentially) even hemoptysis or syncope. It is also usually seen in a more acute presentation scenario. Pericarditis is incorrect, although may be extremely hard to differentiate from pathologies that cause right-sided heart failure. The cause of pericarditis is a major difference; etiologies include tuberculosis, radiation therapy, cardiac surgery, viral pericarditis, or histoplasmosis. Dilated cardiomyopathy is incorrect; this condition will also occur in the left portion of the heart initially.Causes include (but are not limited to) ETOH abuse, catecholamine excess, and myocarditis; they can also be idiopathic.

A 55-year-old man, a smoker with a 30-pack-year history of cigarette smoking, presents with a 3-month history of cough productive of blood and sputum. The patient admits a weight loss of 25 pounds over the past year. The patient also complains of diffuse 'bone' pain, abdominal pain, polydipsia, polyuria, and anxiety over the past month; all are unusual symptoms for him. The patient denies recent travel. The patient was treated in the emergency room twice over the past 2 months for nephrolithiasis. On chest radiography, a 3cm cavitary lesion with an air-fluid level is noted centrally in the left upper lung field; according to the radiologist it is considered suspicious for a pulmonary abscess. The patient also exhibits clubbing on physical examination. The patient's PPD is negative, and there is no history of anti-tuberculosis medication use in this patient's history. The patient's laboratory values are as follows: Calcium: 13.2 (Normal: 8.5-10.8 mg/dl) Phosphorus: 1.8 (Normal: 2.5-4.5 mg/dl) Urine Cyclic Adenosine Monophosphate (cAMP): 9.2 nmol/mL (Normal: 1.6-6.2 nmol/mL) Question What is the most likely diagnosis? Answer Choices 1 Reactivation of Latent Tuberculosis Infection 2 Small Cell Carcinoma 3 Squamous Cell Carcinoma 4 Bronchoalveolar Cell Carcinoma 5 Large Cell Carcinoma

squamous cell Squamous cell carcinoma is the most likely diagnosis in this patient with signs and symptoms of hypercalcemia; it is a notorious paraneoplastic syndrome in pulmonary squamous cell carcinoma due to the secretion of parathyroid hormone related peptide (PTHRP). The patient also has a strong history of cigarette smoking, clubbing on physical examination, and a centrally-located cavitary lung lesion that resembles a pulmonary abscess; all findings suggest squamous cell carcinoma. This question combines clinical features of both hyperparathroidism and squamous cell carcinoma. Hyperparathyroidism is a syndrome of hypercalcemia resulting from excessive release of parathyroid hormone and in most cases is due to adenoma in a single parathyroid gland. Parathyroid gland hypertrophy accounts for the remaining cases of hyperparathyroidism, and paraneoplastic syndromes such as those associated with squamous cell carcinoma are indeed rare. The symptoms of hyperparathyroidism are simple if you remember the mnemonic: "Painful bones (and tenderness), renal stones (nephrolithiasis), abdominal groans (abdominal pain), and psychic moans (changes in mental status)." This patient has abdominal pain, bone pain, a history of nephrolithiasis, and a recent change in mental states (new onset anxiety and depression). Combine these features with a high serum calcium, low serum phosphate, and high urinary cAMP (PTH exerts its effects though the second messenger cyclic AMP (cAMP)), and the diagnosis of hyperparathyroidism is assured. Bronchoalveolar cell carcinoma is a subtype of adenocarcinoma, and although an association with smoking has not been established, a substantial percentage of patients have a significant smoking history. The incidence of bronchoalveolar cell carcinoma is increased in patients who have underlying interstitial lung disease, parenchymal scaring, and exogenous lipoid pneumonia. Bronchoalveolar carcinoma may appear as a solitary pulmonary nodule, multiple nodules, or consolidation. Large cell pulmonary carcinomas account for only a small percentage of bronchogenic carcinomas and are also associated with cigarette smoking. The lesion occurs peripherally and grows rapidly, with early metastases and a poor outcome. Small cell carcinomas also arise centrally in peribronchial locations and infiltrate the bronchial submucosa. Small cell carcinoma of the lung (Oat Cell Carcinoma) is strongly associated with cigarette smoking. Widespread metastases occur early in the course of the disease, with common spread to mediastinal lymph nodes, liver, bones, adrenal glands, and brain. Also, several paraneoplastic syndromes are associated with oat cell carcinoma secondary to oncogenic cell production of peptide hormones. The most common paraneoplastic syndromes are the syndrome of inappropriate secretion of antidiuretic hormone (SIADH) and the syndrome of ectopic adrenocorticotropic hormone (ACTH) production, which can lead to Cushing Disease.

A 6-week-old male infant presents with a 4-day history of cough and nasal congestion. According to his mother, he occasionally has a bluish tint around his lips while sleeping. There is no history of fever. His older siblings have an upper respiratory infection. The patient's appetite has been decreased somewhat, mostly due to the copious nasal secretions; however, he has been maintaining a normal urine output. He was delivered pre-term at approximately 34 weeks gestation. He had mild respiratory distress syndrome, spending 2 days on a ventilator in the neonatal intensive care unit (NICU). He went home in 10 days and has done well since. He has had no immunizations. A physical exam reveals an infant in mild respiratory distress, respirations of 52/min, with slight intercostal retractions. Temperature is 100.2 F with a HR of 130/min. Perioral duskiness is seen. Oxygen saturation at room air is 83%, and HEENT exam is otherwise normal. His chest exam shows coarse rhonchi and expiratory wheezes. Heart rate and rhythm are regular. No murmurs appreciated. Abdomen is soft and non-tender. Neurological is intact. Chest X-ray shows mild hyper expansion, but no consolidation. Nasal swab for respiratory syncytial virus (RSV) is positive. Question What is the most appropriate treatment? Answer Choices 1 Aerosolized albuterol 2 Intravenous steroids 3 Empiric ampicillin and cefotaxime 4 Aerosolized ribavirin 5 Supplemental oxygen and supportive therapy

supp O2 and supportive treatment Respiratory syncytial virus (RSV) is the most important etiology of viral lower respiratory disease in infants and children; bronchiolitis is the most common manifestation. Infection is initiated in the upper respiratory tract and may spread to the lower tract; this causes obstruction of smaller airways by edema, necrotic tissue, and inflammatory cells. This leads to the typical findings of nasal discharge, cough, retractions, and wheezing or rhonchi. The peak incidence is during the first 2 years of life. In the United States, it occurs during the winter months in an epidemic fashion. RSV is a member of the Paramyxovirus family. Complications include apnea in the very young and premature infant, pneumonia, croup, respiratory failure, otitis media, and dehydration. The severity of bronchiolitis is diagnosed and assessed based on history and physical examination. Infants with mild bronchiolitis can be treated symptomatically at home. The following are the criteria for hospitalization in RSV bronchiolitis: Age less than 12 weeks Gestational age at birth of less than 34 weeks Cardiopulmonary disease or immunodeficiencies Wheezing and respiratory distress associated with oxygen saturation below 92 percent on room air (hypoxemia) History of significant apnea before assessment. The management in moderate-to-severe bronchiolitis involves supplemental oxygen therapy and supportive measures such as prevention of dehydration and respiratory support. Supplemental oxygen is the single most useful therapy, usually delivered via nasal prongs when oxygen saturation (SpO2) falls persistently below 90% in previously healthy infants. Mechanical ventilation may be needed for respiratory failure or severe apnea. Ribavirin, an antiviral agent, has not been shown to reduce hospital stays or mortality. The agent is very expensive and may be teratogenic in humans. It may still have a place for treatment of infants at significant risk for complications (i.e., those with cardiopulmonary disease). Bronchodilators, such as albuterol, have not been shown to be effective; however, their empirical use in the setting of the hospitalized patient is still commonly seen. Likewise, corticosteroids are not effective, and their use is contraindicated. Antibiotics, such as ampicillin and cefotaxime, are not routinely used in the majority of patients, but they are indicated in cases of clinical evidence of superimposed bacterial pneumonia or acquired nosocomial infection during the hospitalization.

A 45-year-old man presents with significant weight loss (10 pounds in 4 months), cough with hemoptysis, and pleuritic chest pain. The chest X-ray shows ill-defined opacities in both the lungs; the opacities have a reticulonodular pattern. A transbronchial biopsy is performed, and it microscopically shows a few epithelioid cells with necrotic debris. What is the most likely diagnosis? Answer Choices 1 Aspergillosis 2 Squamous cell carcinoma 3 Pneumocystis carinii pneumonia 4 Oat cell carcinoma 5 Tuberculosis

tuberculosis The epithelioid cells suggest that it is a granuloma. The hemoptysis suggests that the granuloma has eroded enough parenchyma and involved a bronchus. A granulomatous infection along with hemoptysis is typical for tuberculosis. The history of significant weight loss and hemoptysis is also typical of tuberculosis, but it can be seen in other cases also. Aspergillosis is more likely to produce a fungus ball, which is defined on an X-ray. The inflammatory reaction would vary from acute to mixed to granulomatous. Squamous cell carcinoma is usually a central mass lesion. It is unlikely to produce epithelioid cells, which are seen in the biopsy of this patient. Pneumocystis carinii pneumonia has a granulomatous pattern in rare cases, but the granulomas are small and unlikely to produce hemoptysis. Oat cell (or small cell) carcinoma is centrally located and is unlikely to be found with epithelioid cells.

A 25-year-old asymptomatic, non-smoking male presents with a 9mm solitary pulmonary nodule, with dense central calcification on routine chest x-ray. The nodule's appearance is smooth and calcified with a well-defined edge. This same lesion was present on a work physical x-ray two years ago. Which of the following is the most appropriate management step for this patient? A Positron emission tomograph to rule out malignancy B Resection of the pulmonary nodule C Thoracic magnetic resonance imaging (MRI) D Watchful waiting with serial imaging E Pulmonary function tests

watchful waiting with serial imaging Solitary pulmonary nodules are categorized based upon the probability of malignancy. This patient is under the age of 30, with previous documentation of the lesion and without evidence of growth or change, which decreases the probability of malignancy. In this case, watchful waiting is indicated. Risk factors, such as advanced age, smoking, lesion growth or appearance change, and history of prior malignancy increase malignancy probability, and indicate a need for more defined evaluation.


Related study sets

Taxes: Chap 10 Partnership taxation

View Set

Six essential elements of geography

View Set

NTR 102 Chapter 19, alcohol, Gateway fall '18

View Set

Chapter 25: Negotiable Instruments

View Set

What kind of data is this? Level of Measurement

View Set